Sie sind auf Seite 1von 200

Prof.

Henry de Holanda Campos


Reitor da Universidade Federal do Cear
Prof. Jos Arimatia Dantas Lopes
Reitor da Universidade Federal do Piau
Incio Francisco de Assis Nunes Arruda
Secretrio da Cincia, Tecnologia e Educao Superior do Cear
Profa. Mrcia Maria Tavares Machado
Pr-Reitora de Extenso da UFC
Prof. Miguel Ferreira Cavalcante Filho
Pr-Reitor de Extenso da UFPI
Prof. Robrio Fernandes Alves de Oliveira
Presidente da Associao Brasileira de Qumica
Prof. Srgio Maia Melo
Coordenador do Programa Nacional Olimpadas de Qumica
Prof. Jos Arimatia Dantas Lopes
Vice-coordenador do Programa Nacional Olimpadas de Qumica e
Coordenador da equipe de medidas educacionais
Prof. Cristiano de Almeida Cardoso Marcelino Jr.
Coordenador da equipe de elaborao dos exames experimentais (vdeos)
Prof. Francisco Dantas Filho
Coordenador da equipe de elaborao dos exames tericos

ISSN: 1809-2012

Imprensa Universitria
Universidade Federal do Cear
Organizao de originais:
Prof. Srgio Melo
Capa:
Maherle
Editorao e Projeto Grfico:
Maherle/Srgio Melo
2015 Programa Nacional Olimpadas de Qumica
Lanamento em 26.11.2015 por ocasio da solenidade de encerramento dos eventos: XXI Olimpada Norte/Nordeste de Qumica, Olimpada Brasileira de Qumica Jnior e Olimpada Brasileira de Qumica - 2015
Tiragem: 15.000 exemplares.
Distribuio gratuita

Qumica 2015
Parte 1
Calendrio ........................................................................................ 4
Mensagem do Reitor da UFC aos participantes da OBQ ......... 5
Opinio.............................................................................................. 7

Sumrio

XXI Olimpada Norte/Nordeste


Exames .............................................................................................. 9
Soluoes escolhidas ...................................................................... 15
Resultados....................................................................................... 23
VIII OBQ Jnior
Exames Fase I ................................................................................. 25
Exames Fase II ................................................................................ 34
Solues escolhidas ...................................................................... 38
Resultados....................................................................................... 41
Destaques de 8o. ano ................................................................... 39
Destaques das escolas pblicas ................................................. 40

OBQ 2015
Fase III Modalidade A .................................................................... 46
Fase III Modalidade B .................................................................... 59
Solues escolhidas ...................................................................... 73
Resultados - Modalidade A .......................................................... 84
Resultados - Modalidade B .......................................................... 86
Parte 2
PROCESSO SELETIVO .................................................................. 98
OBQ Fase IV.................................................................................... 88
OBQ FASE V ................................................................................... 91
OBQ FASE VI .................................................................................. 92
47th International Chemistry Olympiad
Exame Terico ...................................................................104
Exame Prtico ....................................................................131
XX Olimpada Iberoamericana de Qumica
Exame Prtico ....................................................................156
Exame Terico ...................................................................165
Destaques Olmpicos ..................................................................185
Depoimentos ................................................................................192
Consideraes Finais ..................................................................195
Endereos dos Cooordenadores ..............................................197

Calendrio 2015
Data
27/01/2015

Hora
15h

01/03/2015 a
14/03/2015

8h-12h e
14h-18h

18/04/2015
30/04/2015

9:00h
23h

23/05/2015

14h

Aps
28/06/2015
01/06 a
08/08/2015
08/08/2015

23h
on line

20 a
29/07/2015
01 a
22/08/2015

Atividade
Fase IV da OBQ-2014 (Exame sobre tcnicas laboratoriais com o
objetivo de selecionar a equipe que representar o Brasil em Azerbaijo e em Teresina).
Curso de Aprofundamento e Excelncia (Fase V) para os 15 estudantes selecionados no exame de conhecimentos de laboratrio.
Ministrado pela UFRN.
Exames da Olimpada Brasileira de Qumica - 2014 Fase VI
Divulgao dos nomes dos quatro estudantes que representaro
o Brasil nas competies internacionais.
XXI Olimpada Norte/Nordeste de Qumica - XXI ONNeQ. Cinquenta estudantes por estado. Inscries restritas aos coordenadores-estaduais.
Divulgao de resultados da XXI Olimpada Norte/Nordeste de
Qumica.
Inscries para a VIII Olimpada Brasileira de Qumica Jnior. Escolas inscrevem seus alunos de 8 e 9 anos do ensino fundamental.
Exames da VIII Olimpada Brasileira de Qumica Jnior - OBQjr,
para estudantes de 8 e 9 anos. (Fase I). Lanar as notas at
21/08/2015.
47 Olimpada Internacional de Qumica, Baku - Azerbaijo. http://
icho2015.msu.az/
Inscries para a Olimpada Brasileira de Qumica - 2015. Vinte
e cinco estudantes por estado na modalidade A (penltima srie
do ensino mdio ou srie anterior), 25 estudantes por estado na
modalidade B (3a srie).
Exames da Olimpada Brasileira de Qumica - 2015 - Fase III - Modalidades A e B. Questes analtico-expositivas.
20a Olimpada Ibero-americana de Qumica, Teresina- PI.

29/08/2015

14h

05/09 a
14/09/2015
19/09/2015

14h

Exames da VIII OBQjr. (Fase II). (OBS: s sero recebidos os exames da 2 fase se postados nos Correios at 08.10).
Semana Nacional de Cincia e Tecnologia

23h

Divulgao dos resultados da OBQ-2015, a partir de 04/10/2015.


(48h para recurso do gabarito)
Divulgao de resultados da VIII Olimpada Brasileira de Qumica
Jnior - VIII OBQjr, a partir de 25/10/2015.
Reunio do Conselho de coordenadores, em Fortaleza.

19 a
25/10/2015
Aps
04/10/2015
Aps
25/10/2015
26 e
27/11/2015
26/11/2015

Atividade

23h
8:30h a
18:00h
19:30h

Solenidade Nacional de Encerramento e Premiao das Olimpadas de Qumica, em Fortaleza. Theatro Jos de Alencar.

Olimpada Brasileira de Qumica - 2015

Mensagem

Dr. Henry de Holanda Campos

Mensagem do Reitor da Universidade


Federal do Cear aos participantes das
Olimpadas de Qumica
A Qumica est mais presente em nossas vidas
do que costumamos imaginar. Alimentos e medicamentos, combustveis e fertilizantes, tintas e cosmticos, em quase tudo o que passa pelas nossas
mos, na sociedade moderna, h uma contribuio
dos estudos e pesquisas do profissional da Qumica. Da sua importncia na construo de um modelo saudvel de sociedade, onde se invista na qualidade de vida, sem descuidar da responsabilidade
social e ambiental.
Hoje, aps experimentar extraordinrio desenvolvimento terico e metodolgico, a cincia de Lavoisier busca os caminhos para seguir avanando,
atravs das grandes vertentes em que se desdobrou, no sculo XX, para melhor explicar o ambiente que nos cerca: Qumica Orgnica, Qumica Inorgnica, Bioqumica, Fsico-Qumica e Qumica Analtica. To complexa quanto
interdependente, ela tornou-se, verdadeiramente, uma Cincia Central, estabelecendo conexes naturais com a Fsica, a Geologia, a Biologia e outras
reas do saber.
claro que esse universo merece toda a ateno dos que trabalham com
a Educao em nosso pas. O surgimento das Olimpadas de Qumica, em
1986, coloca-se como um marco no ensino dessa disciplina, na medida em
que passou a incentivar os estudantes e suas escolas a se engajarem numa
salutar competio de conhecimentos, cujos benefcios se projetam por todo
o sistema, despertando vocaes e motivando, de uma forma natural, o estudo e a pesquisa. No por acaso que o Programa Nacional Olimpadas de
Qumica, criado em 1995 pela Universidade Federal do Cear e outras 17
instituies federais de ensino, se expandiu com o passar dos anos, atraindo
hoje milhares de participantes em todos os estados brasileiros.
As Olimpadas se orientam por uma srie de valores, que incluem o reconhecimento do potencial criativo dos jovens e da educao como base da
cidadania, assim como o incentivo tica, responsabilidade social, honestidade e ao trabalho em equipe. Esses postulados se tornam cada vez mais
importantes diante dos novos desafios que se colocam para os profissionais
da Qumica, confrontados que so, diariamente, pela necessidade de desenI

Olimpada Brasileira de Qumica - 2015

Mensagem

Dr. Henry de Holanda Campos

volverem tecnologias limpas, que respeitei o meio ambiente, que levem a


uma explorao racional dos recursos naturais e que coloquem em primeiro
plano o ser humano e os interesses coletivos.
Ao externar meu convencimento de que as Olimpadas de Qumica tm
cumprido seu papel, ajudando a formar profissionais no apenas competentes, mas tambm conscientes de sua misso, transmito a todos os participantes dessa contenda da inteligncia os votos de muito sucesso em seus
projetos, augurando que o trabalho agora realizado consolide seu interesse
pela Qumica e os ajude a se firmarem, futuramente, na seara profissional.
Henry de Holanda Campos
Reitor da Universidade Federal do Cear-UFC

O cientista no o homem que fornece


as verdadeiras respostas;
quem faz as verdadeiras perguntas
Claude Lvy-Straus
antroplogo franco belga, (1908-2009)

Olimpada Brasileira de Qumica - 2015

Opinio

Antnio Gis

Fuga de professores
Maioria dos alunos que ingressam em licenciaturas de fsica,
biologia, matemtica e qumica no conclui o curso
Ao investigar a base de dados do Censo da Educao Superior no Brasil,
a pesquisadora Rachel Pereira Rabelo descobriu um dado indito e preocupante: dos alunos que ingressaram em 2009 nos cursos de licenciatura em fsica, biologia, matemtica e qumica, apenas uma minoria consegue concluir
o curso. A situao mais preocupante est em fsica, onde somente um em
cada cinco (21%) estudantes obtm o diploma. Em matemtica e qumica, a
relao de apenas um em cada trs universitrios (34% em ambos os cursos). Em biologia, a taxa de 43%.
Os dados constam de sua dissertao de mestrado na Escola Nacional
de Cincias Estatsticas do IBGE. Rachel, que tambm servidora do Inep
(instituto vinculado ao MEC responsvel pelas avaliaes e censos educacionais), conseguiu trabalhar pela primeira vez com dados longitudinais dos
estudantes, ou seja, pde acompanhar at 2013 a trajetria de uma mesma
gerao de alunos que ingressou nesses cursos cinco anos antes. Isso s foi
possvel porque, a partir de 2009, os dados do censo permitiram identificar
cada universitrio em todo o seu percurso acadmico.
Alm de uma minoria de alunos conseguir se diplomar nessas licenciaturas, a tese mostra que, desses poucos que se formam, a maioria, uma vez
no mercado de trabalho, acaba desistindo da profisso e migra para outras
ocupaes, possivelmente em busca de melhores salrios.
O olhar de Rachel para essas quatro licenciaturas se justifica porque elas
esto entre as de maior dficit de professores em sala de aula. A tese mostra
que somente 20% dos profissionais que do aulas de fsica no pas possuem
formao adequada para esta disciplina. Esses percentuais so um pouco
maiores nos casos de qumica (35%), biologia (52%) e matemtica (64%).
Outro dado que revela o tamanho do problema nessas reas o percentual de alunos na educao bsica que no tiveram professores nessas
disciplinas. Em 2013, de acordo com o Censo Escolar, um tero das turmas
do ensino mdio no tiveram docentes de biologia (33%), qumica (35%) ou
fsica (36%) para dar aulas. Em matemtica, a proporo foi de 25%.
Ou seja, uma parcela nada desprezvel de nossos jovens tm formao
precria nessas disciplinas porque sequer havia professor com titulao adequada para dar aulas.
I

Olimpada Brasileira de Qumica - 2015

Opinio

Antnio Gis

O problema no novo, e, nos ltimos anos, o Ministrio da Educao


tentou combat-lo estimulando a criao de novos cursos ou tentando atrair
mais alunos para essas reas de formao de professor, oferecendo bolsas
e outros incentivos tanto para professores j em atuao, mas com formao
inadequada, quanto para novos alunos que pretendem seguir a carreira docente.
Esse esforo, at agora, tem se mostrado insuficiente para dar conta do
desafio. Com base nas taxas de ingresso, concluso, e em componentes demogrficos e do mercado de trabalho, Rachel fez projees do nmero de
professores em sala de aula at 2028. Elas indicam que o problema mais
preocupante em fsica e matemtica, pois, mesmo no cenrio mais otimista,
a estimativa de que chegaremos em 2028 com um nmero menor de professores at do que o verificado hoje nessas duas reas.
Ao fim, a concluso do estudo de que seria muito mais eficiente desenvolver polticas para evitar a evaso nesses cursos do que priorizar a ampliao do nmero de vagas. como se estivssemos insistindo em colocar mais
gua numa banheira com vazamentos por todos os lados.
Antnio Gis
Antnio Gois jornalista e colunista do GLOBO,
especializado em educao.
Publicado em 20.04.2015 Jornal O Globo.

"Seja qual for o seu sonho - comece.


Ousadia tem genialidade, poder e magia."
Goethe

Olimpada Brasileira de Qumica - 2015

XXI ONNeQ

Apresentao

XXI Olimpada
Norte-Nordeste de Qumica
23/05/2015

LUZ, CINCIA E VIDA


Luz, cincia e vida o tema da 12 Semana Nacional de Cincia e
Tecnologia - SNCT 2015 e baseia-se na deciso da Assembleia Geral das
Naes Unidas, que proclamou 2015 como o Ano Internacional da Luz, com
objetivo de celebrar a luz como matria da cincia e do desenvolvimento
tecnolgico.
Hoje, nas grandes cidades, o excesso de iluminao polui o cu, mas por
outro lado, ainda existem muitos lugares no mundo onde a escurido da noite quebrada apenas pelas luzes das lamparinas a querosene, que comprometem a sade de quem as utiliza, fazendo com que a luz esteja ligada de
forma visceral vida na terra e ao caminho da humanidade.
As plantas, quando expostas luz do sol, realizam o processo da fotossntese, de extrema importncia para a manuteno do equilbrio biolgico
nos diversos ecossistemas do planeta. Todas as reaes que ocorrem durante
este processo so essencialmente qumicas.
Da mesma forma, no desenvolvimento econmico e tecnolgico, a indstria qumica transforma as matrias-primas presentes na natureza em produtos teis ao homem, desde a fabricao de bens como computadores e
automveis, at itens como plsticos, vidros, papel e tintas que tambm so
resultados de transformaes qumicas.
Assim, o desafio atual socializar o conhecimento da cincia Qumica, a
fim de desmistific-la como vil nos principais impactos das atividades humanas, e promover o aperfeioamento das indstrias qumicas, de forma a assegurar a sustentabilidade ambiental, econmica e social de seus processos
e produtos, bem como contribuir para a permanente melhoria da qualidade
de vida da sociedade.
A Comisso
I

Olimpada Brasileira de Qumica - 2015

XXI ONNeQ

Exames

XXI Olimpada
Norte-Nordeste de Qumica
23/05/2015
QUESTO 1
Em qumica muito comum nos depararmos com substncias que aparentemente no tm aplicaes no cotidiano e por isso acabam no recebendo a devida importncia dos estudantes. Um exemplo disso so as substncias COCl2 (cloreto de carbonila ou fosgnio), SOCl2 (cloreto de tionila), o
SO2Cl2 (cloreto de sulfurila) e o POCl3 (cloreto de fosforila). Essas substncias so extremamente importantes na sntese orgnica devido ao arranjo
espacial de seus tomos que apresentam um centro eletroflico e grupos
abandonadores. So extremamente perigosos devido sua facilidade de
reao com a gua, que gera gs clordrico. Sobre essas substncias e suas
reaes, responda:
a) Em cada uma das substncias acima h ligao dupla. Explique, com
base na carga formal dos tomos, por que essa ligao prevalece sobre a
ligao coordenada nas molculas de SOCl2 e POCl3.
b) Apresente as frmulas estruturais planas de todas as substncias citadas
no enunciado acima e suas respectivas geometrias moleculares.
c) Escreva as equaes qumicas balanceadas da reao do COCl2 e do
SOCl2 com a gua, respectivamente.
d) O cloreto de sulfurila forma-se atravs da reao entre o cloreto de tionila
com oxignio atmico. Escreva a reao e identifique as espcies reagentes que se comportam como cido ou base de Lewis.
e) Classifique as substncias citadas no enunciado acima quanto sua polaridade, justificando em funo do momento dipolar resultante.

10

Olimpada Brasileira de Qumica - 2015

XXI ONNeQ

Exames

QUESTO 2
A caracterizao de uma substncia requer a determinao da frmula qumica seguida da frmula estrutural que pode ser feita por diversas tcnicas
analticas, desde as mais clssicas, como a gravimetria, e as especificas, como
a espectroscopia. A decomposio dos organismos acompanhada pela
formao de substncias de odores indesejveis, como a putrescina. Em uma
anlise elementar de 0,5000 g dessa substncia foi estimada 272,70 mg de
carbono e 68,18 mg de hidrognio, e o restante corresponde massa de nitrognio. A massa molar determinada experimentalmente para a putrescina
88 g mol-1.
Responda aos itens a seguir:
a) Escreva a frmula emprica e frmula molecular da putrescina.
b) A putrescina uma diamina terminal, com base nessas informaes apresente o nome sistemtico e sua frmula estrutural.
c) Determine o nmero de molculas de putrescina na amostra analisada.
d) Determine o nmero de tomos de nitrognio na amostra analisada.
e) O odor desagradvel exalado pela putrescina pode ser neutralizado, ou
pelo menos reduzido, quando em contato com substncias de carter
cido. Escreva a equao da neutralizao da putrescina com cido clordrico.

QUESTO 3
A soldagem o procedimento de fixao de materiais metlicos com aplicao de calor ou presso. O tipo de solda que visa unir os materiais por adio de calor denominado soldagem por fuso, pois se baseia na fuso das
partes metlicas a serem unidas. Para isso, faz-se uso de um maarico, que
um equipamento capaz de produzir chama pela queima de um combustvel.
Suponha que, em um dia de sol, com temperatura prxima dos 30C, um mecnico deseja efetuar uma solda em um objeto de ferro e, para isso, ele precisa fundir cerca de 100 g, referentes ponta do objeto metlico. Assim, ele
utilizou um maarico tendo o acetileno, C2H2, como combustvel. Para efeito
de clculos considere os seguintes dados:
Entalpia de formao do dixido de carbono, fHo(CO2, g)
I

394 kJ mol-1

Olimpada Brasileira de Qumica - 2015

11

XXI ONNeQ

Exames

Entalpia de formao da gua, fHo(H2O, l)


286 kJ mol-1
Entalpia de formao do acetileno, fHo(C2H2, g)
+227 kJ mol-1
-1
-1
Calor especfico do ferro, c(Fe)
0,46 J g C
Calor de latente de fuso do ferro, L(Fe)
268 J g-1
Temperatura de fuso do ferro, T(Fe) = 1535 C
Pede-se:
a) Represente os orbitais hibridos e p puros do carbono no acetileno.
b) Represente a reao qumica resultante da queima do acetileno (combusto completa).
c) Qual a quantidade mnima de calor necessria para fundir os 100 g de
ferro?
d) Calcule a entalpia de combusto do acetileno.
e) Determine a quantidade mnima de massa de acetileno gasta na soldagem.

QUESTO 4
A reao de fuso nuclear que ocorre no Sol produz luz e calor que, ao atin-

que ocorregir
noaSol
produz luzterrestre,
e calor que,
ao atingir a a vida na Terra. Um processo natural
superfcie
movimenta
na
Terra.
Um
processo
natural
que
se
utiliza
da no
energia
que sedeutiliza
da energia
disponibilizada
sol que,
reao
de fotossntese,
to 4 - A reao
fuso nuclear
que ocorre
Sol produz pelo
luz e calor
ao atingir
a

fcie terrestre, movimenta a vida na Terra. Um processo


natural
se utiliza
da energia
ossntese,
, em
queque
o gs
oxignio
liberado enuanto
que
o
carbono

assimilado
para
produo
de
quanto
que
o
carbono

assimilado
para
produo
de
biomassa.
nibilizada pelo sol reao de fotossntese,
, Mas, o horimrdios,
dominou
uso
do
fogo
e
vem
desenvolvendo
mem,
desde
dos
primrdios,
dominou
uso
do
fogo
e
vem
desenvolvendo
ue o gs oxignio liberado enquanto que o carbono assimilado para produo de
e energia
para
atender
as dos
suas
necessidades
maisuso
de
usoprimrdios,
de
vriasdominou
fontes
de energia
atender as suas necesassa.
Mas,tecnologias
o homem,
desde
do fogo e para
vem desenvolvendo
de
energia
fotovoltaica,
ainda
de
elevado
custo
de
sidades
mais
bsicas.
Como
exemplo,
a
tecnologia
de
energia
fotovoltaica,
logias de uso de vrias fontes de energia para atender as suas necessidades
mais
m
energia
eltrica
atravs
de
clulas
fotovoltaicas
com
ainda
de
elevado
custo
de
instalao,
que
converte
energia
solar
as. Como exemplo, a tecnologia de energia fotovoltaica, ainda de elevado custo deem energia
ontes que
de energia
tambm
pela
energia
eltrica
atravs
desupridas
clulas
fotovoltaicas
com
base fotovoltaicas
no princpio
fotoeltrico.
ao,
converteso
energia
solar
em
energia
eltrica
atravs
de clulas
com
hdrica.
Com
base
no
supracitado
texto,
responda
s
Outras
fontes
de
energia
so
tambm
supridas
pela
energia
solar,
no princpio fotoeltrico. Outras fontes de energia so tambm supridas pela energia entre elas

elicae ea ahdrica.
hdrica.
no supracitado
texto, s
responda
entre elasa aenergia
energia elica
ComCom
basebase
no supracitado
texto, responda
seguintes perguntas:
ntes perguntas:
a) ocorre
Descreva
a principal
nuclear que
na superfcie
do Sol.

reao de fuso nuclear que ocorre na superfcie

do Sol.

escreva a principal reao de fuso nuclear que ocorre na superfcie do Sol.


combusto da glicose a 25C 2808 kJ mol1. Com
b) Sabe-se que a entalpia padro de combusto da glicose
a estime
a entalpia
de padro
formao
10 glicose
g de a 25C 2808 kJ mol1. Com
1 produzirda
abe-se
que
a entalpia
depara
combusto

a 25C
2808 kJ mol . Com base na reao de fotossntese acima esti-1
-1
kJ mol
e fHode
(CO
kJ mol
. formao
2, g) = -395 acima
ase
na reao
fotossntese
a entalpia
de formao
me
a entalpia
deestime
para
produzirpara
10produzir
g de 10
C6gHde
O . Dados:
12 6
o
-1
o
-1
6H12O6. Dados: fH (H2O, ) = -286 kJ mol e fH (CO2, g) = -395 kJ mol .
dispostos 30 painis retangulares de clulas de Si de

2
produz
em
mdia
Considerando
que
uma
rea
1 mdispostos
uma
estao
energia
solardeBrasileira
so
painis
retangulares de clulas de Si de
I
Olimpada
de Qumica 30
- 2015
I
12de
-1
J.s
.
Qual
a
quantidade
de
energia
emitida
pelo
Sol
menso de 0,92 m 2,00 m cada. Considerando que
uma rea de 1 m2 produz em mdia

XXI ONNeQ

Exames

c) Numa estao de energia solar so dispostos 30 painis retangulares de


clulas de Si de dimenso de 0,92 m x 2,00 m cada. Considerando que
uma rea de 1 m2 produz em mdia 100 W de potncia e que 1W = 1 J.s-1.
Qual a quantidade de energia emitida pelo Sol durante 5 horas em um
dia ensolarado.
d) Com base no estudo dos gases como est relacionada a energia elica
para produo de energia eltrica?
e) A energia hdrica o aproveitamento dos movimentos das guas de rios
com desnveis naturais ou artificiais que movimentam as turbinas para
produo de energia eltrica. A frmula da potncia instalada de uma
hidreltrica (P) P=
, onde a densidade de gua (kg m3), Q
vazo de gua (m3 s1), h a altura da coluna dgua (m), g a acelerao
da gravidade ( 10 m s2) e o rendimento do sistema (valor relativo).
Agora, estime o volume mdio de gua por segundo da usina hidreltrica de Belo Monte em construo no rio Xingu, no Par, considerando 125
metros de altura de queda dgua para gerar 11,25 mil megawatts, com
90 % de eficincia e densidade da gua de 103 kg m-3. Dados: 1 W = 1 J
s1 = 1 kg m2 s3.

QUESTO 5
O gs acetileno muito utilizado em oficinas de automveis, maaricos para
solda,
amadurecimento
artificial
de frutas
etc.
utilizado
como
ma- para
Questo
5 - O gs acetileno
muito
utilizado
emTambm
oficinas de
automveis,
maaricos
tria-prima
para a obteno
substncias.
Pode
ser obtido para a
solda, amadurecimento
artificialde
de diversas
frutas etc.outras
Tambm
utilizado como
matria-prima
a partir
do de
carbonato
de clcio,
principal
componente
calcrio,
utilizando
obteno
diversas outras
substncias.
Pode
ser obtido ado
partir
do carbonato
de clcio,
o principal
coque (carvo)
como
dos reagentes.
A seguir,
apresentada
uma
secomponente
do um
calcrio,
utilizando o coque
(carvo)
como um dos
reagentes.
A
quncia
deapresentada
reaes, mostrando
a obteno
domostrando
acetilenoaeobteno
sua transformao
seguir,
uma sequncia
de reaes,
do acetileno e sua
em
substnciasem
relacionadas:
transformao
substncias relacionadas:
(I)

CaCO3

(II)

CaO

(III)

CaC2

(IV)

3A

(V)

calor

3C

CaO + CO2
forno
eltrico

CaC2

2H-O-H

+
A

CO

Ca(OH) 2

Olimpada Brasileira de Qumica - 2015


+ I H-Cl

p, T
catalisador

H3C-Cl

AlCl3

13

transformao em substncias relacionadas:


calor

CaO + CO2

(I)

CaCO3

(II)

CaO
+ 3C
XXI
ONNeQ
eltrico

(III)

CaC2

(IV)

3A

forno

2H-O-H

H3C-Cl

3HNO 3

(VII)

Exames

CO

Ca(OH) 2

H-Cl

H-O-H

p, T

+
B

catalisador

(V)
(VI)

CaC2

AlCl3

H2SO4

O
.

AlCl3

Cl

H-Cl

De acordo com as reaes acima, responda aos itens abaixo:

De acordo com as reaes acima, responda aos itens abaixo:


Determine a afrmula
estrutural
e o nome
das substncias
A, B, C e D. A,
a)a) Determine
frmula
estrutural
e o sistemtico
nome sistemtico
das substncias
B, C e D.
b) Numa amostra de calcrio foi extrada a massa de 750 g de carbonato de clcio. Essa

b) Numa
amostra de calcrio foi extrada a massa de 750 g de carbonato de
massa foi utilizada para a produo de acetileno, com rendimento de 80 %. Calcule a
clcio.
Essa massa foi utilizada para a produo de acetileno, com rendimassa de acetileno produzida.
mento de 80 %. Calcule a massa de acetileno produzida.
Qual
ocupado,
nas CNTP,
pelo acetileno
produzidoproduzido
no item (b)? no item
c) c) Qual
oovolume
volume
ocupado,
nas CNTP,
pelo acetileno
(b)?
d) O cloreto de alumnio, usado como catalisador nas reaes (V) e (VII), considerado cido

d) O cloreto de alumnio, usado como catalisador nas reaes (V) e (VII),


ou base de Lewis? Justifique.
considerado cido ou base de Lewis? Justifique.

Identifique qual
dos
produtos
acimaacima
utilizado
na fabricao
de explosivos
e descreva sua
e)e) Identifique
qual
dos
produtos
utilizado
na fabricao
de explosireao
de decomposio.
vos
e descreva
sua reao de decomposio.

"Quando Deus mudar o itinerrio, no se frustre,


Ele sabe o que h depois da curva".
Desconhecido

14

Olimpada Brasileira de Qumica - 2015

MELHORES RESPOSTAS ONNEQ 2015

XXI ONNeQ

Solues Escolhidas

Questo
01 Sigilo 76
NOTA: 10,0
pontos 2015
MELHORES
RESPOSTAS
ONNEQ
MELHORES RESPOSTAS ONNEQ 2015

XXI Olimpada
Natal
Colgio Cincias Aplicadas
RN
de Qumica
Questo 01 Sigilo 76 Norte-Nordeste
NOTA: 10,0 pontos
Breno Carvalho Cirne
De Simas

Questo 01 Sigilo 76 NOTA: 10,0 pontos

MELHORES RESPOSTAS
Breno Carvalho Cirne
Natal
Colgio Cincias Aplicadas
De
Simas
Breno
Carvalho Cirne
RN Colgio Cincias Aplicadas
Natal de Lewis
a) Frmulas
De Simas
RN
Questo 01

SOCl2

Resoluo de Breno Carvalho Cirne De


(II)
RN
(I) Colgio Cincias Aplicadas
Cl Simas,
Cl
a) Frmulas de Lewis
S
O
Cl
Cl S O
a)a)
Frmulas
de Lewis
Frmulas
de Lewis
ou
SOCl2
SOCl
2
SOCl
Na estrutura I, as cargas formais dos ligantes so todas iguais a 0, enquanto que
2
(I)
Cl
(II)
Cl
(I)
Cl
estrutura
II
temos
uma carga fomal +1 para
oCl
S
O
Cl
Senxofre
O (II)e -1 para o oxignio. Dessa fo
Cl
ou
Cl IS aOmais estvel por
S O
estrutura
ou apresentarClmenor soma dos mdulos das cargas form
Na estrutura I, as cargas formais dos ligantes so todas iguais a 0, enquanto que na
estrutura I,
as cargas
formais
dosdos
ligantes
so todas
iguais
a 0,
enquanto
na
NaNaestrutura
as
cargas
formais
so
todas
iguais
a 0,que
enquanto
estrutura
II temosI,uma
carga fomal
+1 para
o ligantes
enxofre e -1
para
o oxignio.
Dessa
forma, a
estrutura
temos POCl
uma carga
fomaluma
+1 para
o enxofre
-1 para
o oxignio.
Dessa
forma,
que naII estrutura
II3 temos
carga
fomale +1
para
o enxofre
e -1
paraa o
estrutura
I

a
mais
estvel
por
apresentar
menor
soma
dos
mdulos
das
cargas
formais:
oxignio.
forma,
estrutura
IO asoma
mais
estvel
por
menor
estrutura
I a Dessa
mais estvel
poraapresentar
menor
mdulos
dasapresentar
cargas formais:
(IV)
(III)dos
O
soma dos mdulos das cargas formais:

Cl P Cl
Cl

Cl P Cl
Cl

POCl
3
POCl
POCl3 3

ou
O
(IV)
(III)
O
Da mesma
forma, a estrutura III possui O
todos os
ligantes com carga formal nula
O
(IV)
(III)
Cl P Cl
Cl P Cl
P Cl
que IV
o fsforo com carga formal +1
o Cl
oxignio com carga -1, logo III uma
Cl ePCl
Cltem
Cl
ou
Cl
Cl
mais estvel, o que justifica
ou a prevalncia da dupla.
Da
mesma
forma,
a
estrutura
III
possui
todos
os ligantes
com com
cargacarga
formal nula, enquanto
DaDa
mesma
estruturaIIIIIIpossui
possui
todos
os ligantes
nula,
mesmaforma,
forma, aa estrutura
todos
os ligantes
com carga
formalformal
nula, enquanto
que
IV tem o fsforo
formalcom
+1 e carga
o oxignio
com+1
carga
logo III com
uma estrutura
enquanto
que IVcom
temcarga
o fsforo
formal
e o-1,oxignio
carga
b) com
que
tem III
o fsforo
carga formal
+1 e o oxignio
com carga
-1, logo III uma
estrutura
-1,IV
logo
uma
estrutura
mais estvel,
o que justifica
a prevalncia
da dupla.
mais
estvel,
oque
justifica
a prevalncia
da dupla.
mais estvel, o queSOCl
justifica
a prevalncia da dupla.
2:

b)

Cl S

b)
b)
SOCl2:
SOCl2:
Cl S
Cl S
Cl
Cl
POCl3:
POCl3:
O
O
Cl P
Cl P
Cl
Cl

Cl
O
O

Cl
Cl

Cl

POCl3:O
O
OS
Cl S Cl
Cl
ClCl P Cl
Cl
O:
SO2Cl
COCl
2:O
2
P
Cl P
Cl

(geometria piramidal)
Cl

(geometria piramidal)
(geometria
O piramidal)
Cl (geometria tetradrica)
P
Cl
Cl

Cl (geometria tetradrica)
Cl (geometria
I
Olimpada Brasileira de Qumica - 2015
tetradrica)
Cl
Cl

15

XXI ONNeQ

Solues Escolhidas

c) 1COCl2(g) + 1H2O(l) 2HCl(g) + 1CO2(g)


1SOCl2(g) + 1H2O(l) 2HCl(g) + 1SO2(g)
d) SOCl2(g) + O(g) SO2Cl2(g)
O oxignio age como cido de Lewis (receptor de par isolado) e o SOCl2
como base de Lewis (doador de par isolado)
e) Todas as substncia apresentadas apresentam desigual distribuio de
cargas em suas estruturas, e tm, por isso vetor momento dipolar eltrico no-nulo. Logo, so polares.

Questo 02
Resoluo de Seon Augusto De Souza Ferreira, Colgio Militar BA
a) Para facilitar os clculos, pode-se tratar de uma amostra hipottica de 1
g, multiplicando os valores experimentais por 2:
1g: 545,4 mg C
136,6 mg H
1g (545,4+136,36) = 318,24 mg N
Dividindo-se pelas respectivas massas molares:
545,4 mg C = 45,45 mmol C => 500/11 mmol C
136,6 mg H = 136,36 mmol H => 1500/11 mmol H
318,24 mg N = 22,73 mmol N => 250/11 mmol N
Dividindo pela menor frao:
Proporo: 2C : 6H : 1N => logo, a frmula emprica (C2H6N)
Como a frmula emprica tem massa molecular de 44 g/mol, duas vezes ela
dar a putrescina. Assim, a frmula molecular da putrescina 2x(C2H6N) =
C4H12N2

16

Olimpada Brasileira de Qumica - 2015

Proporo: 2C : 6H : 1N => logo, a frmula emprica (C2H6N)


Como a frmula emprica tem massa molecular de 44 g/mol, duas vezes ela dar a
putrescina. Assim, a frmula molecular da putrescina 2x(C2H6N) = C4H12N2
Solues
Escolhidas
XXI ONNeQ
b)

b)
NH2

NH2

Butan-1,4-diamina (1,4-butandiamina)

c) O nmero de Butan-1,4-diamina
mols de molculas na amostra
pode ser encontrado atravs da massa
(1,4-butandiamina)
molar: de mols de molculas na amostra pode ser encontrado atrab) O nmero
N=m/MM
n=0,5/ 88g/mol .: n=0,0057mol
vs da
massa=>molar:
23

1mol=6,02x10 molculas
N=m/MM
=> n=0,5/ 88g/mol .: n=0,0057mol

Np = 0,0057 mol X 6,02x1023 molculas/mol= 3,43 x 1021 molculas

1mol=6,02x1023 molculas

d) 1 molcula de putrescina = 2 tomos de nitrognio

23
21
21
21
Np =N=2
0,0057
X 6,02x10
x 3,43x10
= 6,86x10molculas/mol=
tomos de nitrognio3,43 x 10 molculas
Np = 2 mol

1 e)
molcula de putrescina = 2 tomos de nitrognio
N=2 Np = 2 x 3,43x1021 = 6,86x1021 tomos de nitrognio
NH2

NH2

+ 2HCl

Cl

NH3

Cl

NH3

Questao 3

03carbono no acetileno.
a) Represente os orbitais hibridos eQuesto
p puros do
Resposta:
Resoluo sugerida
No acetileno o carbono apresenta hibridizao sp, ou seja, tem dois orbitais
Represente
orbitais
hibridos
ppuros.
puros Estes
do carbono
acetileno.
hbridosos
sp
de dois
orbitaise p
orbitais no
podem
ser assim
representados:
Resposta:
i. os orbitais hbridos sp e formam as
Noii.acetileno
apresenta
hibridizao
sp, ou seja, tem dois orbi(pi)
os orbitais o
pcarbono
puros formam
as ligaes
tais hbridos sp de dois orbitais p puros. Estes orbitais podem ser assim
representados:
A molcula de etino (acetileno) pode ser assim representada:
i. as ligaes entre os orbitais hbridos
i - os
orbitais hbridos sp e formam as ligaes (sigma)
frente e tem superfcie de contato maior;

ii - os
pppuros
as ligaes
(pi) se ligam em paralelo, a
ii. orbitais
os orbitais
puros formam
formam ligaes
; os
orbitais
superfcie
de contato
menor;
portanto,
as ligaes
so mais fracas, mais
A molcula
de etino
(acetileno)
pode
ser assim
representada:
fceis de serem rompidas.

as ligaes entre os orbitais hbridos so as ligaes ; os orbitais se ligam


de frente e tem superfcie de contato maior;
os orbitais p puros formam ligaes ; os orbitais se ligam em paralelo, a
superfcie de contato menor; portanto, as ligaes so mais fracas, mais
fceis de serem rompidas.

Olimpada Brasileira de Qumica - 2015

17

XXI ONNeQ

Solues Escolhidas

b) Equaciona-se
Equaciona-se a
a queima
queima do
do
do acetileno:
acetileno:
b)
b) Equaciona-se
Equaciona-seaaqueima
queima doacetileno:
acetileno:
b) Equaciona-se a queima do acetileno:

necessria
fuso dos
dos100
100ggde
deferro:
ferro:
c) Calcula-se
Calcula-seaaaenergia
energia necessria
necessria para fuso
100
ferro:
c)
c) Calcula-se
Calcula-se aenergia
energia necessriapara
para fusodos
dos 100ggde
de ferro:
c) Calcula-se a energia necessria para fuso dos 100 g de ferro:

d) Calcula-se
Calcula-se aa entalpia
entalpia de
de combusto:
combusto:
d)
d)Calcula-se
Calcula-sea aentalpia
entalpiade
decombusto:
combusto:
d) Calcula-se a entalpia
combusto:

e) Determina-se
Determina-se ento
ento oo nmero
nmero de
de mols
mols ee aa massa
massa de
de acetileno
acetileno gasta
gasta
e)
de mols
molseeaa massa
massa de
deacetileno
acetilenogasta
gasta
e)Determina-se
Determina-seento
entooo nmero
nmero de
e) Determina-se------ento o nmero de mols e a massa de acetileno gasta
-------------------------------------------

Questo 0404Questo
Questo0404Questo
a) 18
A reao
fuso
nuclear
do- Sol
Sol proposta
sugure que
que se inicia
inicia com dois
dois ncleos
I de
Olimpada
Brasileira
de Qumica
2015
I
a)
a) AAreao
reaode
defuso
fusonuclear
nucleardo
do Solproposta
propostasugure
sugure quese
se iniciacom
com doisncleos
ncleos
a) atmicos
A
reao hidrognios
de
fuso nuclear
do
Sol
proposta
sugure
que
se
inicia
com
dois
ncleos
atmicos
hidrognios
com
sucessivas
reaes
dando
origem
ao
hlio,
conforme
com sucessivas reaes dando origem ao hlio,
conforme

XXI ONNeQ

Solues Escolhidas

Questo 04
Resoluo sugerida
a) A reao de fuso nuclear do Sol proposta sugure que se inicia com
1a reao:
dois ncleos atmicos
hidrognios com sucessivas reaes dando ori1a reao:
gem ao hlio, conforme
mecanismo abaixo.
aa reao:
11a reao:
11a areao:
reao:
reao:
11aa reao:
Reao
Global
1a reao:
Reao
Global
Reao Global
Global
Reao
Reao
Global
b) A partir dos dados fornecidos pode-se calcular a entalpia padro molar de
b) formao
A partir dos
dados fornecidos pode-se calcular a entalpia padro molar de
dados
glicose:
A partir
dadosfornecidos
fornecidos
pode-secalcular
calcularaaa entalpia
entalpia padro
padro
molar
b) b)
A partir
partir
dos
dados
fornecidos
pode-se
calcular
entalpia
padro molar
molar de
de
b)
A
dos
dados
pode-se
formao
da glicose:
de
formao
da
glicose:
b) Aformao
partir
dos
dados
fornecidos
pode-se
calcular
a
entalpia
padro
molar
de
formao da
da glicose:
glicose:
formao da glicose:

E para 10 g de glicose a entalpia ou calor :


E para 10 g de glicose a entalpia ou calor :
E para
para 10
10 gg de
de glicose
glicose aa entalpia
entalpia ou
ou calor
calor :
:
E
E para 10 g de glicose a entalpia ou calor :
c)
c) c)
A rea
painis

A rea
totaltotal
dos dos
painis

c) A rea total dos painis


c)
c) A
A rea
rea total
total dos
dos painis
painis
a potncia
total
gerada
painis
AEErea
total
dos
painis
pelos
totalgerada
gerada
pelospainis
painis
E aa potncia
potncia total
pelos

E aa potncia
potncia total
total gerada
gerada pelos
pelos painis
painis
E
E a potncia total gerada pelos painis

Assim, pode-se calcular a energia emitida pelo Sol em megajoules (1 W = 1 J s1):


Assim, pode-se calcular a energia emitida pelo Sol em megajoules (1 W = 1 J s1):
1
Assim,
pode-se
calcular
a energia
emitida
pelo
em megajoules
W
Assim, pode-se
pode-se
calcular
energia
emitida
pelo Sol
Sol
em Sol
megajoules
(1 W
W == 11 JJ(1
Assim,
calcular
aa energia
emitida
pelo
em
megajoules
(1
ss1
):): =
1
Assim, pode-se calcular a energia emitida pelo Sol em megajoules (1 W = 1 J s ):

A energia elica o aproveitamento da energia cintica contida nas massas de


A energia
elica
o aproveitamento
energia
contida
nas massas
de
ar
(misturaelica
de gases)
em movimento da
(vento)
quecintica
promove
a rotao
de hlices
A
oooaproveitamento
da
energia
cintica
contida
nas massas
massas
Aenergia
energia
elica
aproveitamento
da
energia
cintica
contida
nas
massas
de
A
energia
elica
aproveitamento
da
energia
cintica
contida
nas
de
ar
(mistura
de
gases)
em
movimento
(vento)
que
promove
a
rotao
de
hlices
a gerao
de
eletricidade.
Os movimentos
do ar ocorrem
praticamente
pela
Apara
energia
elica

o
aproveitamento
da
energia
cintica
contida
nas
massas
de
ar (mistura
(mistura
de gases)
gases)
em movimento
movimento
(vento) que
que
promove
rotao de
de hlices
hlices
ar
de
em
(vento)
aa rotao
para
a gerao
de aquece
eletricidade.
Os movimentos
do promove
araumenta
ocorrem
pela
solar
a atmosfera,
ou
apraticamente
dos
I melhor,
Olimpada
Brasileira
de Qumica
- temperatura
2015
Ide hlices
arenergia
(mistura
de que
gases)
em movimento
(vento)
quedo
promove
a rotao
19pela
para
a
gerao
de
eletricidade.
Os
movimentos
do
ar
ocorrem
praticamente
pela
para
a
gerao
de
eletricidade.
Os
movimentos
ar
ocorrem
praticamente
energia
solar
que
aquece
a
atmosfera,
ou
melhor,
aumenta
a
temperatura
dos
gases
atmosfricos
(energia Os
trmica)
que do
aumenta
proporcionalmente
ao
para
a gerao
de eletricidade.
movimentos
ar ocorrem
praticamente pela

Assim, pode-se calcular a energia emitida pelo Sol em megajoules (1 W = 1 J s1):

XXI ONNeQ

Solues Escolhidas
A energia elica o aproveitamento da energia cintica contida nas massas de
ar (mistura de gases) em movimento (vento) que promove a rotao de hlices
de ar
(mistura de
deeletricidade.
gases) em movimento
(vento)
que
promove
a rotao pela
de
para
a gerao
Os movimentos
do ar
ocorrem
praticamente
hlices para a gerao de eletricidade. Os movimentos do ar ocorrem praenergia solar que aquece a atmosfera, ou melhor, aumenta a temperatura dos
ticamente pela energia solar que aquece a atmosfera, ou melhor, aumenta
gases
atmosfricos
(energia
trmica) (energia
que aumenta
proporcionalmente
a temperatura
dos gases
atmosfricos
trmica)
que aumenta pro-ao
quadrado
da
velocidade
mdia
desses
gases
(energia
cintica).
E como
uma
porcionalmente ao quadrado da velocidade mdia desses gases
(energia
mistura
de gases
comporta-se
como
fluido,
a relao trmica
cintica
cintica).
E como
uma mistura
deum
gases
comporta-se
como eum
fluido,est
a
relao trmica
e cintica
correlacionada
a teoria
cintica
correlacionada
entre
a teoriaest
cintica
dos gases entre
e lei dos
gases
ideais, dos
pela
gases e lei
dospor
gases ideais, pela expresso dada por
expresso
dada

onde: a energia cintica mdia das molculas de gs


onde:
a massa do gs

a energia cintica23
mdia1das molculas de gs

Boltzmann
(1,38
10 J K )
Ec aconstante
energiade
cintica
mdia
a massa
dodas
gsmolculas de gs

a
temperatura
termodinmica.
m a massa do gs
constante de Boltzmann (1,38 1023 J K1)
k constante de Boltzmann (1,38 x 1023 J K1)
a temperatura termodinmica.
d)T a temperatura termodinmica.

Rearranjando a frmula para calcular a vazo mdia temos:


d)
c) RearranjandoRearranjando
a frmula para
calcular
vazo amdia
a frmula
paraacalcular
vazotemos:
mdia temos:

Questo 05 Sigilo 183 NOTA: 10,0 pontos


Eric Pereira Queiroz
Questo
05 Sigilo
Belm
Ideal183 NOTA: 10,0 pontos
Moreira
Eric Pereira Queiroz
Moreira

Questo 05
Belm

Ideal

Resoluo de Eric Pereira Queiroz Moreira, Colgio Ideal - PA


a)a)
A

a)

A CH
3
H-C

B
O2N

H-C

C-H

metilbenzeno
(tolueno)
Etino

NO 2

CH3

C-H

Etino

CH3

O2N

CH3
NO 2

CH3

CH

NO 2
2,4,6-trinitrotolueno
metilbenzeno

1-fenilbutan-1ona
NO
2

2,4,6-trinitrotolueno

1-fenilbutan-1ona

I
Olimpada Brasileira de Qumica - 2015 (tolueno)
I
b) De acordo com os coeficientes estequiomtricos das reaes I, II e III a proporo

20

XXI ONNeQ

Solues Escolhidas

b) De acordo com os coeficientes estequiomtricos das reaes I, II e III a


proporo molar do acetileno e do clcio de 1:1, logo o n de acetileno de 0,75 mol x 10, porm esse resultado para 100%. Assim feita
a proporo de 100% para 0,75 e 80% para x, onde x 0,6 mol, logo 0,6
mol de acetileno dividido para 26g/ mol de clcio onde o resultado
15,6 gramas e multiplicando por 10 o resultado 156g.
CaO

CaCO3
100%

-----

0,75

80%

-----

x
X= 15,6 g x 100 = 156g
X= 15,6 g x 100 = 156g

X= 15,6 g x 100 = 156g

c)
1 mol

c) c)

c)
mol --------22,4
LL
11mol
22,46mol
1
mol
----6mol --------6mol
V
6mol
-----

-----

22,4 L

-----

22,4 L

d) O AlCl3 um cido de Lewis, isso observado na sua estrutura de Lewi

d) O AlCl3 um cido de Lewis, isso observado na sua estrutura de Lewis,


observa
o Al
possui
6 eltrons
sua
camada
de
d) Opois
AlClse
um cidoque
de
Lewis,
isso
observado
na em
suaem
estrutura
de Lewis,
pois seassim ele po
3 observa
o Alque
possui
6 eltrons
sua
camada
devalncia,
valncia,
um6 par
de
eltrons
o camada
que
caracteriza
os cidos
Lewis.
assim ele
receber
um
par
de
eltrons
o que
caracteriza
osreceber
cidos
observa
que opode
Al possui
eltrons
em sua
de valncia,
assimdeele
pode
d) O AlCl3 um cido de Lewis, isso observado na sua estrutura de Lewi
de Lewis.
um par de eltrons o que caracteriza os cidos de Lewis.

observa
o Al possui
6 como
eltrons
em2,4,6-trinitrotolueno.
sua acamada
de valncia, assim ele pod
e) A substncia
usada
como
explosivo
aexplosivo
(C),
e) que
A substncia
usada
(C), 2,4,6-trinitrotolueno.
parcomo
de eltrons
o que
caracteriza
os cidos de Lewis.
e) A substnciaum
usada
explosivo
a (C),
2,4,6-trinitrotolueno.
e) A substncia usada como explosivo a (C), 2,4,6-trinitrotolueno.
Questo 05 Sigilo 267 NOTA: 10,0 pontos

De Souza
Questo 05 Sigilo 267Gustavo
NOTA:
10,0 pontos
Medeiros

Gustavo De Souza
Medeiros

Colgio Motivo

Recife
Colgio Motivo

Recife

Questo 05 Sigilo 267 NOTA: 10,0 pontos


Gustavo De Souza
Medeiros
a)

Colgio Motivo
Recife

Olimpada Brasileira de Qumica - 2015

21
D

Questo 05 Sigilo 267 NOTA: 10,0 pontos

XXI ONNeQ

Gustavo De Souza
Medeiros

Solues Escolhidas

Colgio Motivo

Recife

Questo 06
Resoluo sugerida Gustavo De Souza Medeiros. Colgio Motivo - PE
a)a)
A

B
CH3

H-C

CH3
H3 C

NO 2

O
CH3

C-H

NO 23 produziu 1 mol de C2H2. Como o


b) De acordo com as reaes v-se que 1 mol de CaCO

produzira 0,8 mols de C1-fenilbutan-1ona


rendimento
1mol de CaCO3 2,4,6-trinitrobenzeno
2H2. Logo 7,5 mol de
Etino de 80%,
metilbenzeno
b) De acordo com as reaes v-se que 1 mol de CaCO3 produziu 1 mol de C2H2. Como o
de C2H2, massa molar de C2H2 26 g consequentemente 6 mol
CaCO3 produzira 6 mols(toluene)
rendimento de 80%, 1mol de CaCO3 produzira 0,8 mols de C2H2. Logo 7,5 mol de
sero
156g.
b)
De3 acordo
as de
reaes
v-se molar
que 1demol
CaCO3 produziu 1 mol
produziracom
6 mols
C2H2, massa
C2Hde
6 mol
CaCO
2 26 g consequentemente
de C2H2. Como o rendimento de 80%, 1mol de CaCO3 produzira 0,8
sero
156g.
mols
de C2H2. Logo 7,5 mol de CaCO3 produzira 6 mols de C2H2, massa
c)
molar de C2H2 26 g consequentemente 6 mol sero 156g.
1 mol
----22,4 L
c) c)
6 mol
----1 mol
----22,4 L
6 mol

-----

d) O AlCl3 um cido de Lewis, pois ele age como um receptor de eltron e se torna o
Oion
AlCl
um
cido de Lewis, pois ele age como um receptor de eltron e
3
AlCl4
4d) torna
O AlCl
um
cido
de Lewis, pois ele age como um receptor de eltron e se torna o
se
o 3ion
AlCl
ion AlCl4
e) A substncia usada como explosivo a (C), 2,4,6-trinitrotolueno. O reagente de entalpia
d) A substncia usada como explosivo a (C), 2,4,6-trinitrotolueno. O reamuito gente
alta decompem-se
em vrias
estveis de
a reao estmuito
deusada
entalpia
altasubstancia
decompem-se
emmodo
vriasque
substancia
e) A substncia
comomuito
explosivo
a (C), 2,4,6-trinitrotolueno.
O reagente de
entalpia
veis de
que a reao muito exotrmica e at explosiva.
exotrmica
e atmodo
explosiva.
muito alta decompem-se em vrias substancia estveis de modo que a reao muito
exotrmica e at explosiva.

22

Olimpada Brasileira de Qumica - 2015

XXI ONNeQ

Resultados

XXI Olimpada
Norte/Nordeste
de Qumica 2015
RESULTADOS
Nome
Seon Augusto de Souza Ferreira
Renner Leite Lucena
Tiago de Sousa Viana
Gabriel Ferreira Gomes Amgarten
Giovanni Elson Rafael de Souza
Lucas Bastos Oliveira
Matheus Fortunato J.Barros
Davi Oliveira Arago
William Chaves Lima
Breno Carvalho Cirne de Simas
Joo Guilherme Madeira Arajo
Eric Pereira Queiroz Moreira
Joo Martins Cortez Filho
Gabriel Henrique Cabezas Assis
Carlos A. C. de Vasconcelos Filho
Francisco Luiz Isael Junyor
Flvia Pineiro Nery
Gustavo Milito S. do Nascimento
Ana Anglica Luz Pereira
Dayanne Rolim Carvalho
Matheus Cardoso Arago
Gabriel Barbosa Meireles

Cidade
UF
Escola
OURO
Salvador
BA
Colgio Militar
Fortaleza
CE
Farias Brito
Fortaleza
CE
Master Bezerra
Fortaleza
CE
Ari de S
Fortaleza
CE
Farias Brito
Fortaleza
CE
Master Sul
Salvador
BA
Col. Integral
PRATA
Fortaleza
CE
Colgio Militar
Fortaleza
CE
Farias Brito
Natal
RN Cincias Aplicadas
Fortaleza
CE
Farias Brito
Belm
PA
Ideal
Teresina
Pi
Dom Barreto
Fortaleza
CE
7 de Setembro
Maranguape CE
Farias Brito
Fortaleza
CE
Farias Brito
Salvador
BA
Antnio Vieira
Manaus
AM Colgio Militar
Teresina
Pi
Dom Barreto
Fortaleza
CE
Farias Brito
Fortaleza
CE
Ari de S
Fortaleza
CE
Antares

Nota Escore
45,50
43,25
43,00
42,50
42,00
42,00
41,75

100,0
95,05
94,51
93,41
92,31
92,31
91,76

41,25
41,00
40,50
40,00
40,00
40,00
39,75
39,50
39,50
39,25
39,00
38,75
38,50
38,50
38,45

90,66
90,11
89,01
87,91
87,91
87,91
87,36
86,81
86,81
86,26
85,71
85,16
84,62
84,62
84,51

Olimpada Brasileira de Qumica - 2015

23

XXI ONNeQ

Joo Guilherme Porto Santos


Breno Maia Baptista
Italo Lesione de Paiva Rocha
Eduardo Nunes Velloso
Italo Rennan Lima Silva
Svio de Oliveira Brilhante
Gustavo Manfio Leme de Campos
Joo Lus Sousa Guedes Alcoforado
Amanda Camelo Paulino
Marcelo Lotufo Manzano
Marcelo Arajo Guanabara
Carolina de Sousa Sampaio
Felipe Vieira Coimbra
Pedro Teotnio de Sousa
Isabel Maria Oliveira Macdo Lima
Juan Freire Dantas Galvo
Ana Lusa Vieira Ferreira
Cefas Vieira da S. Almeida Ferreira

Resultados

BRONZE
Aracaju
SE
Fortaleza
CE
Fortaleza
CE
Natal
RN
Fortaleza
CE
Fortaleza
CE
Fortaleza
CE
Recife
PE
Fortaleza
CE
Palmas
TO
Fortaleza
CE
Salvador
BA
Teresina
Pi
Fortaleza
CE
Teresina
Pi
Fortaleza
CE
Caucaia
CE
Teresina
Pi

Colgio Amadeus
Farias Brito
Master Bezerra
Salesiano S. Jos
7 de Setembro
Farias Brito
Farias Brito
Marista So Luis
Ari de S
Colgio Olimpo
7 de Setembro
Colgio Anchieta
Dom Barreto
Ari de S
Dom Barreto
Farias Brito
Colgio Master
Colgio Lavoisier

38,00
37,75
37,75
37,50
37,25
37,25
37,00
37,00
36,75
36,75
36,50
36,25
36,00
35,75
35,75
35,25
35,00
34,95

83,52
82,97
82,97
82,42
81,87
81,87
81,32
81,32
80,77
80,77
80,22
79,67
79,12
78,57
78,57
77,47
76,92
76,81

A lista de alunos agraciados com MENO HONROSA


e DEMAIS CLASSIFICADOS na XXI Olimpada Norte/Nordeste de Qumica
encontra-se disponvel em www.obquimica.org.

"Abandonar uma iluso nos deixa mais sbios


que encontrar uma verdade"
Ludwig Born

24

Olimpada Brasileira de Qumica - 2015

VIII OBQ Jnior

Fase I

VIII Olimpada Brasileira


de Qumica Jnior - Fase I
8 e 9 anos do Ensino Fundamental

INSTRUES
1. A prova consta de 20 (vinte) questes objetivas, cada uma contendo quatro alternativas,
das quais voc deve assinalar apenas uma.
2. A prova tem durao de 3 horas.
3. Voc receber o gabarito aps 1 hora do incio da prova, para registrar as suas opes de
respostas.
Boa prova!
A imagem abaixo ser utilizada nas questes 1 e 2.

Bolbo
Filamento
Haste

Base

01 Em qual dos componentes desse tipo de lmpada existe uma maior


quantidade de silcio?
A) Base

B) Bulbo

C) Filamento

D) Haste

02 Um gs inerte deve ser adicionado no interior dessa lmpada para


evitar a presena de oxignio. Entre as opes abaixo, que gs teria
essa indicao?
A) Argnio

B) Fsforo

C) Sdio

D) Urnio
I

Olimpada Brasileira de Qumica - 2015

25

VIII OBQ Jnior

Fase I

03 Uma estudante recebeu um bilhete contendo a seguinte composio:


52

95

Te

Am

Telurio

Americio

Oxgeno

Considerando essas informaes e as configuraes eletrnicas dos elementos qumicos utilizados (O: 1s2 2s2 2p4; Te: [Kr] 4d10 5s2 5p4;
Am: [Rn] 5f7 7s2), quem elaborou essa composio escolheu dois elementos
da tabela peridica que
A) se localizam no mesmo perodo.
B) se encontram num mesmo grupo.
C) so considerados gases nobres.
D) so metais gasosos e radioativos.
04 A ferrugem, xido de ferro, o resultado da reao do ferro com o
oxignio, quando esse metal est em contato com a gua e/ou com
umidade do ar. Em ambientes midos, isso ocorre numa esponja de
ao (l de ao-carbono) e num prego, por exemplo.
Uma substncia simples que participa do fenmeno qumico descrito acima
o (a)
A) ao.

B) gua.

C) ferrugem.

D) oxignio.

05 Um processo de separao de uma mistura


ilustrado abaixo.
Considerando que houve adequao da tcnica
utilizada nesse procedimento, no bquer acima
do funil h uma
A) soluo.
B) substncia pura, apenas.
C) mistura homognea.
D) mistura heterognea.

26

Olimpada Brasileira de Qumica - 2015

VIII OBQ Jnior

Fase I

06 Um material produzido a partir de matria orgnica, na ausncia de


oxignio, tem a seguinte composio qumica mdia:
Gs
(Frmula qumica)
CH4
CO2
N2
H2
O2

Teor em volume (%)


55 - 75
25 - 45
0-3
0-2
0 - 0,1

H2S

0-1

Essas especificaes so correspondentes ao


A) ar comprimido.
C) gs de cozinha.

B) biogs.
D) gs para bales de festas.

07 Analise a imagem mostrada abaixo.

http://bioquimicaufal.blogspot.com.br/

Ela ilustra o incio da dissoluo de


A) vinagre em gua. B) acar em etanol.
C) um sal na gua.

D) uma moeda em um cido.

Olimpada Brasileira de Qumica - 2015

27

VIII OBQ Jnior

Fase I

08 O melado um lquido xaroposo obtido pelo aquecimento do caldo de cana (Saccharum officinarum) para retirada da gua, ou a partir
da rapadura, por processos tecnolgicos adequados.
Adaptado de http://www.anvisa.gov.br/anvisalegis/resol/12_78_melaco.htm

A preparao do melado envolve uma operao de


A) destilao simples

B) destilao fracionada

C) evaporao

D) fuso

09 Em muitas culturas h registros de diferentes objetos produzidos para atividades similares. Um exemplo desses aparatos tecnolgicos mostrado na figura abaixo.
Esse equipamento um(a)
A) centrfuga.

B) destilador.

C) nebulizador.

D) panela de presso.

Fonte: http://elprofe-sabe.blogspot.com.
br/2013_04_01_archive.html

Leia o texto abaixo. Ele ser utilizado nas questes 10 e 11.


O Mg o oitavo elemento qumico em abundncia na crosta terrestre. Industrialmente, o magnsio metlico tem sido obtido a partir do processamento
do MgCl2 fundido. Por sua vez, o MgCl2 tem sido obtido de jazidas salinas
ou a partir da gua do mar. Um dos produtos de magnsio mais conhecido o leite de magnsia, a suspenso aquosa de hidrxido de magnsio
(Mg(OH)2), que usada como anti-cido. Nos vegetais, o Mg est presente na
clorofila (C55H72O5N4Mg), substncia essencial para a fotossntese.
Adaptado de http://qnesc.sbq.org.br/online/qnesc12/v12a11.pdf

10 Em que momento o elemento qumico destacado no texto acima


est presente na forma de uma substncia simples?
A) Na clorofila
B) Nas jazidas salinas
C) No processamento do MgCl2 fundido
D) Na composio do leite de magnsia

28

Olimpada Brasileira de Qumica - 2015

VIII OBQ Jnior

Fase I

11 A suspenso usada como anti-cido uma


A) substncia simples

B) substncia pura

C) mistura heterognea D) mistura homognea


12 O recipiente ao lado foi projetado para armazenar
dois produtos alimentcios, que possuem propriedades diferentes. Imiscveis, um deles uma soluo, o
outro uma mistura que contm vrios constituintes.
A fase menos densa e a mais densa so ocupadas, respectivamente, por:
A) azeite de oliva e vinagre.
C) vinho e vinagre.

B) vinagre e azeite de oliva.


D) vinagre e vinho.

Fonte: carbonodesign.com.br

13 Uma teoria bem aceita defende que os elementos qumicos naturais


foram produzidos por um processo denominado nucleossntese. A
partir do elemento mais leve e mais abundante do universo, outros
elementos mais pesados so formados no interior das estrelas por
processos conhecidos como fuso ou fisso nuclear.
De acordo com essa teoria, a nucleossntese ocorre a partir do
A) Au.

B) H.

C) Na.

D) O.

14 difcil de estimar a importncia da tabela peridica. Uma das suas


principais qualidades a de prever a propriedade dos elementos
qumicos e dos compostos que vo se formar por suas combinaes.
Tal previsibilidade o reflexo dos elementos estarem agrupados na
tabela no s pela ordem crescente de seus nmeros atmicos, mas
tambm em funo de sua
_______ I _______. Esta, por sua vez, est no fundamento das ______II ______.
Ourides Santin Filho
Fonte: http://www.clickciencia.ufscar.br/portal/edicao24/

Olimpada Brasileira de Qumica - 2015

29

VIII OBQ Jnior

Fase I

O texto completado de forma correta quando I e II correspondem, respectivamente,


A) densidade e cores das substncias.
B) distribuio eletrnica e ligaes qumicas.
C) dureza e solubilidades dos compostos.
D) ocorrncia e propriedades macroscpicas.
15 Os grficos abaixo trazem uma distribuio da mdia de abundncia
de alguns elementos qumicos em dois sistemas, I e II.

II

Adaptado de: http://www.quimlab.com.br/guiadoselementos/

Os sistemas I e II correspondem, respectivamente,


A) ao e ar.
C) ar e ao.

B) corpo humano e crosta terrestre.


D) crosta terrestre e corpo humano.

16 Qual o mineral composto apenas por carbono e que possui dureza


10 na Escala de Mohs?
A) Bauxita

B) Diamante C) Gipsita D) Quartzo

17 Uma Agncia de Saneamento encaminhou para a percia um cilindro


contendo uma substncia gasosa. Ela utilizada nas Estaes de Tratamento de gua para matar as bactrias que porventura estejam
presentes na gua. Um problema na vlvula do cilindro provocou o
vazamento do produto, que tambm muito txico e corrosivo.
A substncia em questo o
B) H2

A) Cl2

30

C) N2

Olimpada Brasileira de Qumica - 2015

D) SO2

VIII OBQ Jnior

Fase I

Leia o texto abaixo. Ele ser utilizado nas questes 18 e 19.


O esquema abaixo indica parte do processo de decomposio de matria
orgnica em um aqurio.

Restos de comida, folhas, fezes e urina levam produo de uma substncia


(I), que extremamente txica para os peixes. No ciclo do nitrognio, sob a
ao de microorganismos, ela convertida em outros compostos, tambm
txicos para os peixes. Mesmo com a absoro de uma dessas espcies qumicas por vegetais aquticos, esse processo exige a troca peridica da gua.
18 A substncia (I) formada no aqurio a(o)
A) amnia. B) gs carbnico. C) metano. D) oznio.
19 A espcie qumica oxigenada absorvida pelos vegetais do aqurio
ilustrado acima
A) molecular. B) gasosa. C) inica. D) metlica.

Olimpada Brasileira de Qumica - 2015

31

VIII OBQ Jnior

Fase I

20 A imagem abaixo ilustra uma das fontes responsveis por cerca de


20% das emisses globais de carbono para atmosfera.

Fonte: http://www.ipam.org.br

Essa quantidade de carbono produzido se associa s


A) nuvens de oznio.
B) reciclagens de resduos.
C) mudanas de uso do solo.
D) queimas de combustveis fsseis.

Gabarito
1

10

11

12

13

14

15

16

17

18

19

20

32

Olimpada Brasileira de Qumica - 2015

VIII
VII OBQ Jnior

Fase III

VIII Olimpada Brasileira


de Qumica Jnior - Fase II
8 e 9 anos do Ensino Fundamental
INSTRUES
1. A prova consta de 13(treze) questes, 10 (dez) questes do tipo mltipla escolha (mximo 40 pontos) e 3 (trs) questes analtico-expositivas (mximo 60 pontos).
2. Para responder as questes de mltipla escolha, identifique APENAS UMA NICA alternativa correta e marque a letra correspondente no gabarito existente na Folha de Respostas.
3. Para responder as questes analtico-expositivas, utilize APENAS o espao destinado para
cada uma das trs questes na Folha de Respostas.
4. A prova tem durao de 3 horas.
5. Voc receber a Folha de Respostas aps 1 hora do incio da prova, para registrar as suas
opes.
Boa prova!

QUESTES DE MLTIPLA ESCOLHA


Leia o texto abaixo. Ele ser utilizado nas questes 1, 2 e 3.
O hlio-3 um istopo no-radioativo do hlio (configurao eletrnica: 1s2).
Usado na fuso nuclear, ele considerado um combustvel ideal por ser potente, no-poluente e pela sua baixssima gerao de resduos radioativos.
Por causa de sua escassez na Terra, mas abundante na Lua, alguns pases tm
se interessado em construir uma base nesse satlite, para extra-lo em escala
industrial. O procedimento envolveria o aquecimento do solo lunar, fazendo-o escapar das rochas, seguindo-se pela utilizao de outro processo para
poder colet-lo.

Adaptado de http://noticias.uol.com.br/

Olimpada Brasileira de Qumica - 2015

33

VIII OBQ Jnior

Fase II

01 Uma caracterstica dos ncleos de cada tomo do combustvel citado


no texto acima a presena de:
A) dois prtons e um nutron.
B) trs prtons e um nutron.
C) dois prtons e trs nutrons.
D) trs prtons e dois nutrons.
02 Entre os processos apresentados a seguir, qual seria o mais indicado
para recolher e armazenar o combustvel citado no texto?
A) Calefao

B) Fuso

C) Liquefao

D) Sublimao

03 A obteno desse combustvel a partir do solo lunar seria um processo basicamente


A) biolgico.

B) fsico.

C) qumico.

04 Diferentes artistas produziram leituras a


partir de caractersticas dos elementos
qumicos. O autor da obra ao lado lembra
que um desses elementos costumava ser
utilizado para dar cor em alguns tipos de
vidros. Fonte: https://www.flickr.com/
photos/
Outro aspecto que integra a obra a lembrana
do formato de um(a)
A) rvore metlica.
B) luminria de neon.
C) cogumelo de uma exploso atmica.
D) trofu coberto por um banho metlico.

34

Olimpada Brasileira de Qumica - 2015

D) metereolgico.

VIII
VII OBQ Jnior

Fase III

05 Analise a imagem abaixo.


Plataforma Bioqumica
Resduo

Intermedirios
GLICDICOS
(ACARES)

Energia

Plataforma Termoqumica

Intermedirios
GASOSOS ( CO +H2) OU
LQUIDOS (bio-leo)

BIOMASSA
Calor

Fonte: http://www.ladebio.org.br/

Esse tipo de processo est voltado a gerar produtos (I) como


A) biocombustveis
C) material radioativo

B) gs natural
D) querosene

06 O quadro abaixo traz informaes nutricionais de uma poro de


100g de um alimento comercializado por uma rede de fastfood.

Valor Energtico
Carboidratos
Protenas
Gorduras Totais
Gorduras Saturadas
Gorduras Trans
Colesterol
Fibra Alimentar
Clcio
Sdio

Quantidade por poro


48,4 kcal = 203 kj
3,3 g
4,5 g
1,95 g
1,50 g
0g
12 mg
0g
210 mg
63 mg

% VD (*)
2,4 %
1,2 %
5,8 %
3,2 %
7,5 %
---3,7 %
0%
21 %
2,8 %

Essas informaes correspondem a uma poro de


A) batata frita.

B) brigadeiro.

C) coalhada fresca.

D) hambrguer.

Olimpada Brasileira de Qumica - 2015

35

VIII OBQ Jnior

Fase II

07 Misturas de cobre com estanho, aps serem processadas, produzem


um material que ao ser trabalhado confere beleza e resistncia a diversos tipos de objetos. Por isso esse material foi utilizado por muito
tempo na produo de esculturas, armas, utenslios de cozinha e conexes hidrulicas.
Uma etapa determinante na obteno desse material a
A) destilao. B) ebulio. C) fuso. D) calefao.
Leia o texto abaixo. Ele ser utilizado nas questes 8 e 9.
Determinado produto utiliza o hidrognio como principal fonte de combustvel. Ao contrrio dos modelos mais comuns no mercado (como os que utilizam ltio), os resduos gerados pela utilizao desse produto so apenas
energia trmica, calor e gua, eliminada como vapor. Alm disso, esse tipo
de produto armazena mais energia em um espao menor que aquele exigido pelos modelos base de ltio.
Adaptado de: http://veja.abril.com.br/noticia/ciencia

08 O produto enfatizado no texto um(a)


A) foguete

B) caldeira industrial.

C) tanque de armazenamento

D) bateria de clula de combustvel.

09 Considerando os conceitos da qumica verde, esse produto comercialmente


A) inadequado, porque polui muito.
B) adequado, porque utiliza uma substncia inerte.
C) adequado, pois o combustvel e os resduos gerados so pouco agressivos ao meio ambiente.
D) inadequado, pois muito baixa a eficincia atmica do seu processo para
gerar energia.
10 Aproveitando a disputa dos Jogos Pan-Americanos, em Toronto, um
professor fez uma analogia entre um dos tipos de modelos atmicos
e o lanamento do martelo. Nesse desporto, o atleta lana uma esfe-

36

Olimpada Brasileira de Qumica - 2015

VIII OBQ Jnior

Fase II

ra, que est ligada a um cabo de arame preso a uma ala, a manopla.
O atleta segura a manopla com as duas mos, mantendo os ps imveis, e gira a esfera sobre a sua cabea. Depois, gira sobre o prprio
corpo, mantendo a trajetria circular j iniciada. Ao final do ltimo
giro, bloqueia o movimento do seu corpo, fazendo uma alavanca e
lanando a bola.
Considerando o movimento anterior ao lanamento da bola, qual o modelo atmico mais apropriado a esse tipo de analogia?
A) Bohr B) Dalton C) Rutherford

D) Thomson

QUESTES ANALTICO-EXPOSITIVAS

Entre as opes listadas abaixo, indique a que


representa o processo que ocorre com esse
material e justifique a sua escolha.

Massa

11 O grfico ao lado representa as variaes das massas de um material slido,


quando foi molhado e exposto ao ar.

I. Uma pedra de gelo seco


II. Uma palha de ao usada em limpeza

Tempo

III. Uma pepita de ouro, retirada do um rio

12 Quando quatro sais de quatro metais diferentes queimaram, eles


emitiram chamas de cores diferentes.
Explique essa diferena de colorao nas chamas.
13 Algumas pessoas utilizam alimentos/produtos alimentcios para retirar o odor de cheiro de peixe das mos. E funciona! Indique um produto utilizado com essa finalidade e explique porque ele tem essa
eficcia.

Olimpada Brasileira de Qumica - 2015

37

Resoluo de Gabriel Marcos Pereira Barbosa, 9 Ano, Colgio Bernoulli - MG


II- Uma palha de ao usada em limpeza.
A palha de ao constituda principalmente de ferro. Ao ser molhada e exposta ao ar,

VIII OBQ Jnior

Fase II

esta passa a sofrer uma reao qumica denominada oxidao. De acordo com a lei de
Lavoisier a massa dos reagentes deve ser igual a massa dos produtos. Porm a palha

de ao 11
ao -ser
colocada
ar, faz
comPereira
que esta
se encontre
um sistema
aberto,
Questao
Resoluo
deno
Gabriel
Marcos
Barbosa,
9 Ano,em
Colgio
Bernoulli
- MG
utilizando o oxignio presente no sistema (ar). O ferro, ao reagir com a gua e com o
II- Uma
palha de ao usada em limpeza.
oxignio forma o xido de ferro. Como o oxignio possui massa, este ligado ao ferro

A palha de ao constituda principalmente de ferro. Ao ser molhada e exposta ao ar, esta passa a
a palha
ao ficar
mais pesada/aumenta
massa com
da palha.
sofrerfazuma
reaodequmica
denominada
oxidao. Deaacordo
a lei de Lavoisier a massa dos reagentes deve ser igual a massa dos produtos. Porm a palha de ao ao ser colocada no ar, faz com que
esta se encontre em um sistema aberto, utilizando o oxignio presente no sistema (ar). O ferro, ao
reagir com a gua e com o oxignio forma o xido de ferro. Como o oxignio possui massa, este ligado
ao ferro faz a palha de ao ficar mais pesada/aumenta a massa da palha.
Questao 12

Resoluo de Juliano Pinto Ribeiro Filho, 9 Ano, Colgio Mdulo Juazeiro do


Norte
CE 12 - Resoluo de Juliano Pinto Ribeiro Filho, 9 Ano, Colgio Mdulo
Questao

Juazeiro do Norte CE

A queima desse material, vai gerar excitao nos eltrons dos tomos fazendo
com que
eles
pulem as
da eletrosfera.
Quando
esse eltron
chegar
a ltima
A queima
desse
material,
vaicamadas
gerar excitao
nos eltrons
dos tomos
fazendo
com que
eles pulem
camada,daoeletrosfera.
mesmo voltar
a camada
de origem,
nessa camada,
volta queo mesmo
ocorre avoltar
liberao
as camadas
Quando
esse eltron
chegarealtima
a camada

de origem, e nessa volta que ocorre a liberao de ftons. Esses ftons tem um comprimento de
pela cor
dada
chama.
E como cada
tem uma
liberao
de ftons
ondaluz,
() responsvel
que vai determinar
a cor
luz, responsvel
pelatomo
cor da chama.
E como
cada tomo
tem uma
diferente;
o
comprimento
de
onda
ir
variar
de
tomo
para
tomo,
assim
como
a corcomo a
liberao de ftons diferente; o comprimento de onda ir variar de tomo para tomo, assim
daluz
luzemitida.
emitida.
cor da
Esse comprimento
de onda podede
seronda
calculado
equao depela
Rydenberg:
Esse comprimento
podepela
ser calculado
equao de Rydenberg:

= comprimento de onda

R = constante de Rydenberg

N1 e N2 = camadas

R = constante de Rydenberg
e N2 = camadas
N1Questao
13 - Resoluo de Joo Antnio Abdalla Pinheiro, 9 Ano, Colgio Militar - RJ

Quando no preparo do peixe, que um alimento que libera um odor desagradvel devido amnia
que a matria orgnica morta (o peixe) libera.
Podemos utilizar produtos alimentcios como sumo do limo ou vinagre para neutralizar a amnia
que o peixe morto libera, consequentemente neutralizando o mau odor proveniente da mesma. Tal
neutralizao ocorre devido reao qumica que aconteceria entre o cido actico do vinagre e amnia, que resultaria em sal + H2O.

38

Olimpada Brasileira de Qumica - 2015

VIII OBQ Jnior

Destaques

VIII Olimpada Brasileira


de Qumica Jnior
Destaques de estudantes de 8 ano
do ensino fundamental
Alunos de 8o. ano que participaram da VIII Olimpada Brasileira de Qumica Jnior foram avaliados juntamente
com os de 9 ano com idntico exame.
Dentro deste grupo tiveram elevado desempenho os estudantes abaixo relacionados.
Relao dos estudantes de 8 ano que mais se destacaram com medalhas na VIII Olimpada Brasileira de Qumica Jnior
NOME
Jeickson Eduardo M. de Souza Filho
Guilherme Henrique Chaves dos Santos
Anna Victria Capelato Spadaccia
Pedro Burlacchini Sanches Marinho
Helosa Benevides da Silva
Letcia Corra Consolaro
Joo Gabriel de Oliveira Santos
Joo Vitor Lustosa Lima
Marina Lopes Machado
Felipe Pinto Coelho Nuti
Felipe Barros Mendes
Larissa Morelli Ramos
Jos Vitor Alves Mesquita
Tamie Lusa Caram Tominaga
Pedro Tonini Rosenberg Schneider

UF
RN
SP
SP
BA
RN
SP
SP
CE
SP
SP
CE
SP
DF
SP
ES

Vincius Jos Menezes Pereira


Vtor Gabriel Medeiros

ANO
ESCOLA
CIDADE
8 ano
Mater Christi
Mossor
8 ano
Liceu Albert Sabin
Ribeiro Preto
8 ano
Etapa
Valinhos
8 ano
Anchieta
Salvador
8 ano
Mater Christi
Mossor
8 ano
Objetivo - Cantareira
So Paulo
8 ano
Alpha Lumen
So Jos dos Campos
8 ano
Master
Fortaleza
8 ano
Emlio Ribas
Pindamonhangaba
8 ano
Liceu Albert Sabin
Ribeiro Preto
8 ano
Ari de S Cavalcante
Fortaleza
8 ano
Petrpolis
S. Bernardo do Campo
8 ano
Olimpo
Braslia
8 ano
Albert Sabin
So Paulo
8 ano Sagrado Corao de Maria
Vitria
8 ano
GGE
Recife
8 ano
GGE
Recife

Jlia Alves de S

8 ano

CE

Teleyos

Fortaleza

PE
PE

Demais estudantes de 8o. ano que se destacaram na


VIII Olimpada Brasileira de Qumica Jnior encontram-se em www.obquimica.org

Olimpada Brasileira de Qumica - 2015

39

VIII OBQ Jnior

Destaques

VIII Olimpada Brasileira


de Qumica Jnior
Destaques das Escolas Pblicas
Estudantes de escolas pblicas que se sobressaram na VIII Olimpada Brasileira de Qumica Jnior
Pela primeira vez alunos de escolas pblicas que participaram da Olimpada Brasileira de Qumica Jnior e apresentaram elevado desempenho recebem especial homenagem. Dentre todos participantes
desse grupo tiveram elevado desempenho os estudantes abaixo relacionados.
Os cinco primeiros estudantes da relao tero seus nomes gravados no trofu destinado a essa modalidade de premiao. Aps quatro anos o trofu ser entregue escola com maior nmero de alunos gravados em sua base.
Nome do aluno
Joo Vctor de Melo F. de Queiroz
Jocelito Pessotto Junior
Miguel Vieira de Almeida
Antonio Francisco C. de Sales
Mariana Bigolin Groff
Valderi da Silva Jnior
Graziely Sousa Oliveira
Francisco Manoel P. M. A. de Carvalho
Thain Moreira Cornlio
Nayze Firmino Cavalcante
Manoel Victor Freires Vieira
Stphanie Fernandes
Gustavo Mariano Barros
Jardison Rocha Silva
Joo Guilherme V. Saraiva
Cristovan Maciel Teixeira
Veronica Hollmann Mingates Silva
Allison Junior Graa Francelino
Pedro Henrique de Matos Arajo
Raiane Felicio Coelho
Karla Alexsandra de Souza Joriatti
Ana Carolina Lima Moraes

40

Escola do Aluno
ANO Cidade da Escola
E.M. Maria Inez Q. Rodrigues
9 Patos de Minas
E.M.E.F. Afonso Balestrin
9 Taquaruu do Sul
Col. da Polcia Militar do Cear
9 Fortaleza
E.M. Prof. Valter Alencar
9 Teresina
E.E.E.M. Cardeal Roncalli
9 Frederico Westphalen
9 Teresina
E. T. M. Nossa Sra da Paz
C.I.F. Otvio Rodrigues Cavalcante 9 Novo Oriente
E. M. Prof. Oflio Leito
9 Teresina
E. E. Dep. Benedito Matarazzo
9 S. Jos dos Campos
E. T. M. Nossa Sra. da Paz
8 Teresina
E.E.F Francisco Rufino
9 Novo Oriente
E. M. Prof. Edgar Castanheira
9 Joinville
E.M Jos Mokarzel
9 Itapeva
E.M. Mons. Mateus Rufino
8 Teresina
E. M. Gen. Joo Mendona Lima
9 Rio de Janeiro
E.E.F Francisco Severi
9 Novo Oriente
E.M. No Fortes
9 Teresina
E.E.E.F. Monte Alegre
9 Alvorada do Oeste
E. M. Prof. Doriol Beato
9 Conselheiro Lafaiete
E.E.F. Francisco Rufino
9 Novo Oriente
E. M. Prof. Edgar M. Castanheira
9 Joinville
E.M. Prof. Valter Alencar
9 Teresina

Olimpada Brasileira de Qumica - 2015

UF
MG
RS
CE
PI
RS
PI
CE
PI
SP
PI
CE
SC
SP
PI
RJ
CE
PI
RO
MG
CE
SC
PI

VIII OBQ Jnior

Resultados

VIII Olimpada Brasileira


de Qumica Jnior 2015
RESULTADO
UF Cidade da Escola

SP
SC

Valinhos
Jaragu do Sul

BA

Salvador

SP

Campinas

UF Cidade da Escola

BA
SP
BA
CE
SP
BA
SP
TO
CE
PI
CE
PR
RN

Escola do Aluno

Ano

NOTA

Nome do aluno

OURO
Etapa
9
Joo Vitor Chau Bernardino
Bom Jesus Divina 9
Eduardo Augusto Pereira Fischer
Providncia
Anchieta
9
Matheus Vincius de Mendona
Fontoura
Notre Dame
9
Gustavo de Souza dos Reis
Escola do Aluno

Ano

100,0
97,95
95,86
95,08

Nome do aluno

PRATA
Salvador
Anchieta
9 Giovanna M. M. Cavalcante B. de Sousa
So Paulo
Etapa
9
Alice Maria Gallian Augusto
Salvador
Anchieta
9
Maria Clara Souza de Freitas
Fortaleza
Ari de S
9
Ana Myllena Freitas Campos
Santos
Centro Educacional 9
Bruno Carvalho Brando
Objetivo
Feira de Santana
Joo Paulo I
9
Amanda Passos Portugal
So Paulo
Etapa
9 Alexandre Parizotto Moreira da Silva
Palmas
Olimpo
9
Cssio Peres Ribeiro
Fortaleza
Ari de S
9
Samara Cavalcante Lemos
Teresina
Cev Colgio
9
Jos Antenor de Castro Neiva Neto
Fortaleza
Farias Brito
9
rica Batalha Gomes
Londrina
Marista
9
Guilherme Akira Demenech Mori
Mossor
Mater Christi
8 Jeickson Eduardo M. de Souza Filho

UF

Cidade da Escola

TO
BA
BA
CE
CE
ES
MG
MG

Palmas
Salvador
Salvador
Fortaleza
Fortaleza
Vitria
Belo Horizonte
Patos de Minas

Escola do Aluno

Ano

Olimpada Brasileira de Qumica - 2015

93,26
91,18
90,13
89,61
89,35
88,57
88,05
88,05
87,53
85,70
85,44
85,44
85,44
NOTA

Nome do aluno

BRONZE
Olimpo
9
Gustavo Gomes Santiago
Anchieta
9
Lucas Navarro Ferreira
Sartre COC
9
Otto Cruz Fernandes
Ari de S
9
Pedro Henrique Silva de Oliveira
Farias Brito
9
Gabriel Medeiros Lopes
Charles Darwin
9
Enrico Vescovi Salles
Bernoulli
9
Gabriel Marcos Pereira Barbosa
E.M. Maria Inez Q. 9
Joo Vctor de Melo Ferreira de
Rodrigues
Queiroz
I

NOTA

84,40
83,36
83,36
83,36
83,36
83,36
83,36
83,36
I

41

VIII OBQ Jnior


CE
PE
PI
CE
SE
RN
SP
SP
CE
DF
SP
MG
BA
GO
PI
SP
PA
PI
RS
BA
MG
SP
SP
BA
CE
GO
PB
SP
SP
BA
DF
CE
PB
BA
BA
CE
PE
BA
BA

42

Resultados

J. do Norte
Recife
Teresina
Fortaleza
Aracaju
Mossor
Ribeiro Preto
So Paulo
Fortaleza
Braslia
Valinhos
Ipatinga
Salvador
Goinia
Teresina

Modulo / Objetivo
GGE
Dom Barreto
Ari de S
Cemaster
Mater Christi
Liceu Albert Sabin
Objetivo Cantareira
Ari de S
Colgio Militar
Etapa
So Francisco Xavier
Sartre COC
Degraus
Esc. Popular Madre
Villac
So Paulo
Bandeirantes
Abaetetuba
Inst. Nsa Sra dos
Anjos
Teresina
Esc. Popular Madre
Villac
Taquaruu do Sul
E.M.E.F. Afonso
Balestrin
Salvador
Anchieta
Juiz de Fora
Colgio Militar
So Paulo
Albert Sabin
Valinhos
Etapa
Salvador
Sartre COC
Fortaleza
7 de Setembro
Goinia
Prevest
Campina Grande
Motiva
So Paulo
Objetivo Marqus
Valinhos
Etapa
Salvador
Sartre COC
Braslia
Leonardo da Vinci
Fortaleza
Dulia Bringel
Campina Grande
Autntico
Salvador
Anchieta
Salvador
Anchieta
Fortaleza
Colgio Militar
Recife
GGE
Salvador
Anchieta
Salvador
Anchieta
I

Olimpada Brasileira de Qumica - 2015

9
9
9
9
9
9
8
9
9
9
9
9
9
9
9

Juliano Pinto Ribeiro Filho


Rafael Figueiredo Ribeiro
Ana Beatriz Ramos Milhome
Guilherme Martins Oliveira
Pedro Franca de Figueiredo
Lawrence Francisco Martins de Melo
Guilherme Henrique C.dos Santos
Carmella Suemi Gofinet Pasoto
Fernando Silveira Fernandes
Igor de S. Xavier da Silva
Thiago Felipe de Oliveira Santos
Lucas Augusto da Silva Gonalves
Leonardo Silva Sora Ferreira
Gabriel Mazur
Fabrcio Cardoso Ferreira

82,84
82,58
82,58
82,32
82,32
81,80
81,80
81,80
81,28
81,28
81,28
81,02
80,76
80,76
80,76

9
9

Lvia Gondo
Manoel Neri Batista Neto

80,76
80,23

Kauan Vaz do Nascimento

80,23

Jocelito Pessotto Junior

80,23

9
Camille Arago Rossetti
9 Helena Delgado Malvaccini Mendes
9
Clara Zioli da Igreja
8
Anna Victria Capelato Spadaccia
9
Tiago Domingos Almeida Souza
9
Lus Eduardo Matoso Vieira
9
Letcia Mello Oliveira de Souza
9
Mrmon Lima dos Santos
9 Victor Alexandre dos Santos Valsecchi
9
Mathias Stahl Kavai
9
Caio Pereira de Oliveira
9
Joo Tito do Nascimento Silva
9
Danielle dos Santos Silva
9
Maria Eduarda de Azevedo Silva
9
Brenda de Sousa Bastos Barbosa
8 Pedro Burlacchini Sanches Marinho
9
Estevo da Silva Neto
9
Vincius Jos Menezes Pereira
9
Milena Fernandes de Oliveira
9
Roni Barreto da Silva

79,97
79,71
79,71
79,71
79,19
79,19
79,19
79,19
79,19
79,19
78,93
78,41
78,15
78,15
77,63
77,63
77,63
77,63
77,11
77,11

VIII OBQ Jnior

Resultados

BA
BA
CE

Salvador
Salvador
Fortaleza

CE
RN
SP

Fortaleza
Mossor
So Paulo

SP
PE
BA
MG
PI
RJ
SP
SP
MG
CE
CE
CE
DF
SP
SP
PE
SP
MG
BA
CE

Colgio Militar
Mater Christi
Santo Antnio de
Lisboa
So Paulo
Escola Walter Belian
Recife
GGE
Salvador
Anchieta
Belo Horizonte
Magnum Cidade
Nova
Teresina
Dom Barreto
Rio de Janeiro
Colgio Militar
So Paulo
Objetivo Cantareira
So Paulo
Vital Brazil
Belo Horizonte
Magnum Cidade
Nova
Fortaleza
Ari de S
Fortaleza
Master Sul
Fortaleza
Farias Brito
Braslia
Leonardo da Vinci
So Bernardo do
C. E. F. Salvador
Campo
Arena
So Paulo
Albert Sabin
So Jos do
Educandrio SEI
Belmonte
So Paulo
Escola Walter Belian
Belo Horizonte
Bernoulli
Salvador
Anchieta
Fortaleza
7 de Setembro

CE
CE
PI

Fortaleza
Fortaleza
Parnaba

RJ
SP

Rio de Janeiro
So Jos do Rio
Preto
So Paulo
So Paulo

SP
SP

Sartre COC
Sartre COC
Ari de S

Farias Brito
Farias Brito
Col. Nsa Sra das
Graas
Colgio Militar
Coeso

9
9
9

77,11
77,11
77,11

9
8
9

Ana Luisa de Aguiar Almeida Silva


Marianna Gomes da Silva
Marianna de Alencar Cavalcante
Viana
Camilo Rodrigues Vasconcelos
Helosa Benevides da Silva
Giulia Baffini Santana

9
9
9
9

Isis Zurita dos Santos


Bruno Estandislau Lins de Carvalho
Gabriella Sales de Macdo
Pedro Augusto Batista Corra

77,11
77,11
76,59
76,59

9
9
8
9
9

Edilana Soares Luz


Bruno Gabriel Motta Rodrigues
Letcia Corra Consolaro
Aline Yumi Higa
Giovanna Morais Simes Alves

76,59
76,59
76,59
76,59
76,33

9
9
9
9
9

Matthew Fantini
Thiago da Costa Gadelha
Matheus Sousa Rgo da Silva
Joo Pedro Boechat Guimares
Lucas Almeida Garcia de Oliveira

76,07
76,07
76,07
76,07
76,07

9
9

Leia Costanzi Strauss


Anna Paula Silva Pires

76,07
75,81

9
9
9
9

75,55
75,28
75,02
75,02

9
9
9

Ana Laura Davoli Rodrigues da Silva


Lucas Brum Monteiro Candido
Amanda Souza Rodrigues
Felipe Salim H. Buhamara A. N.
Gurjo
Arthur Lopes da Silva Gomes
Levy Bruno do Nascimento Batista
Ana Clarisse Morais Brito

75,02
75,02
75,02

9
9

Joo Antonio Abdalla Pinheiro


Artur Merino Loes

75,02
75,02

Kevin Taiko Onishi


Vincius Vellardi Janoti

75,02
75,02

Etapa
9
Escola Walter Belian 9
I

Olimpada Brasileira de Qumica - 2015

77,11
77,11
77,11

43

VIII OBQ Jnior


SP
PE
PE
DF
PE
CE
SP
CE
CE
CE
GO
PR
SP
SP
SP
BA
MG
BA
CE
CE
CE
MG
RN
SP
SP
PE
AL
CE
SP
SP
SP
PE
AM
CE
CE
DF
PI
SP
CE

44

Resultados

Valinhos
Recife

Etapa
Col. de Aplicao
UFPE
Recife
Col. de Aplicao
UFPE
Braslia
Leonardo da Vinci
Limoeiro
3 Milnio
Fortaleza
Ari de S
Boituva
Anglo de Boituva
Fortaleza
Ari de S
Fortaleza
Farias Brito
Juazeiro do Norte Modulo / Objetivo
Goinia
Olimpo Go
Londrina
Marista de Londrina
Ribeiro Preto
SEBCOC
So Jos dos
Alpha Lumen
Campos
So Paulo
Albert Sabin
Salvador
Sartre COC
Juiz de Fora
Colgio Militar
Salvador
Anchieta
Fortaleza
Col. da PM do Cear
Fortaleza
Master
Fortaleza
Farias Brito
Belo Horizonte
Colgio Militar
Mossor
Mater Christi
Pindamonhangaba
Emlio Ribas
Ribeiro Preto
Liceu Albert Sabin
Recife
Esc. de Aplicao do
Recife
Macei
Contato
Fortaleza
Ari de S
So B. do Campo
Petrpolis
So Paulo
Bandeirantes
So Paulo
Objetivo Paulista
Recife
GGE
Manaus
Adalberto Valle
Fortaleza
Master
Fortaleza
Santa Isabel
Braslia
Olimpo
Teresina
Esc. Cidado Cidad
So Paulo
Albert Sabin
Fortaleza
Christus
I

Olimpada Brasileira de Qumica - 2015

9
9

Victria de Paula Severino


Jlia Cibely da Silva Souza

75,02
75,02

Renato Henrique Alpes Sampaio

75,02

9
Luciana Yumi Miura
9
Joo Gabriel de Lima Raulinho
9 Marcelo Hipplyto de Sandes Peixoto
9
Lucas Alessio Lopez
9 Francisco Moiss Aquino Teodosio
9
Letcia Silva Pinto
9
Alexandre Marques Cabral
9
Pedro Wichtendal Villar
9
Jamil Soubhia Khoury
9
Enzo Luz da Silva
8
Joo Gabriel de Oliveira Santos

74,76
74,76
74,50
74,50
73,98
73,98
73,98
73,98
73,98
73,98
73,98

9
9
9
9
9
8
9
9
9
8
8
9

Gisele Panegassi Rodrigues


Diego Falco Arajo
Ricardo Teles Toledo
Andr Devay Torres Gomes
Miguel Vieira de Almeida
Joo Vitor Lustosa Lima
Juan Sousa Lopes
Tamirys Martins Souza
Sarah Ketley Vieira de Almeida
Marina Lopes Machado
Felipe Pinto Coelho Nuti
Ana Ceclia Interaminense de Farias

73,98
73,46
73,20
72,94
72,94
72,94
72,94
72,94
72,94
72,94
72,94
72,94

9
8
8
9
9
8
9
9
9
8
9
8
9

Hermann S. B. Lima B. de Gusmo


Felipe Barros Mendes
Larissa Morelli Ramos
Erica Huang
Caque da Silva Corra
Victor Gabriel Medeiros
Maria Beatriz Oliva Pinto
Carlos Henricco Rabelo de Queiroz
Joo Vctor Moreira Pimentel
Jos Vitor Alves Mesquita
Thallysson P. de Oliveira M. Moura
Tamie Lusa Caram Tominaga
Lus Fernando Bastos Rgo

72,42
72,42
72,42
72,42
72,42
72,42
72,16
71,90
71,90
71,90
71,90
71,90
71,38

VIII OBQ Jnior

Resultados

CE
MG
PI

Fortaleza
Belo Horizonte
Teresina

PR

Guarapuava

SP
MG
CE
CE
CE
CE
CE
CE
ES
MG
PE
PI
PI

So Paulo
Juiz de Fora
Fortaleza
Fortaleza
Fortaleza
Fortaleza
Fortaleza
Sobral
Vitria
Ipatinga
Caruaru
Teresina
Teresina

PR
SP
PE
CE
CE
CE
SP
PI

Cascavel
So Paulo
Recife
Fortaleza
Fortaleza
Fortaleza
So Paulo
Teresina

RS

Frederico
Westphalen

Farias Brito
Colgio Militar
E.M. Prof. Valter
Alencar
Imperatriz Dona
Leopoldina
Albert Sabin
Colgio Militar
Ari de S
Ari de S
Dulia Bringel
Santa Ceclia
Farias Brito
Santana
S. Corao de Maria
So Francisco Xavier
Diocesano
Lettera
Educ. Sta Maria
Goretti
Marista de Cascavel
Objetivo Paulista
Saber Viver
Teleyos
Ari de S
Teleyos
Bandeirantes
Sagr. Corao de
Jesus
E.E.E.M. Cardeal
Roncalli

9
9
9

Ana Beatriz dos Santos Diniz


71,38
Flvia de Aguiar Mouro Leite
71,38
Antonio Francisco Carvalho de Sales 71,38

Israel Henrique Ferreira

71,38

9
9
9
9
9
9
9
9
8
9
9
9
9

Lucas Rodrigues Camargo


Rodrigo Aleixo Pereira de Souza
Fabiano Lucas Dias Arajo
Mariana Melo de Medeiros
Carlos Ludvick Silva Rodrigues
Isabela Arago Colares
Vincius de Galiza Vieira
Vincius Dilamrio Ferreira da Ponte
Pedro Tonini Rosenberg Schneider
Klaide Lopes de Sena
Vitor Dias Oliveira de Santana
Amanda Silva Oliveira
Trique Muriel Modesto de Brito

71,38
71,12
70,86
70,86
70,86
70,86
70,86
70,86
70,86
70,86
70,86
70,86
70,86

9
9
9
8
9
9
9
9

Joo Gabriel Ribeiro Kopf


Dbora Sayuri Takagui Fernandez
Joo Antnio Arajo Lopes Lima
Jlia Alves de S
Luana Sales de Miranda Gomes
Larissa Sousa Rodrigues
Pedro Anequini Nogueira
Bruno Eduardo Gomes Pacheco

70,86
70,86
70,86
70,60
70,34
70,34
70,34
70,07

Mariana Bigolin Groff

70,07

Demais estudantes de 8o. ano que se destacaram na


VIII Olimpada Brasileira de Qumica Jnior encontram-se em www.obquimica.org

Olimpada Brasileira de Qumica - 2015

45

VIII OBQ Jnior

Apresentao

Olimpada Brasileira
de Qumica Modalidade A
(1 e 2 SRIES) - 29/08/2015

LUZ, CINCIA E VIDA


Luz, cincia e vida o tema da 12 Semana Nacional de Cincia e Tecnologia - SNCT 2015 e baseia-se na deciso da Assembleia Geral das Naes
Unidas, que proclamou 2015 como o Ano Internacional da Luz, com objetivo
de celebrar a luz como matria da cincia e do desenvolvimento tecnolgico.
Em termos tecnolgicos, as lmpadas de LED que consomem menos
energia, a grande quantidade de dados transmitidos em alta velocidade via
cabos de fibra ptica e a obteno de energia solar, so alguns exemplos
de como a luz est ligada de forma visceral vida na terra e ao caminho da
humanidade.
A comodidade gerada por todo desenvolvimento cientfico e tecnolgico concede ao cotidiano, de grande parte da sociedade, energia eltrica,
gua tratada, transportes, alimentos, alm de diversos produtos teis ao homem, como plsticos, vidros, papel e tintas, que so resultados de processos
e transformaes qumicas.
Dessa forma, o desafio do ensino de qumica est em harmonizar o desenvolvimento tecnolgico e a conservao do meio ambiente, fazendo com
que os futuros consumidores, empresrios e cientistas tenham uma nova forma de pensar sobre este modo de vida.
Alm disso, socializar o conhecimento da cincia Qumica, a fim de desmistific-la como vil nos principais impactos das atividades humanas, e promover o aperfeioamento das indstrias qumicas, de forma a assegurar a
sustentabilidade ambiental, econmica e social de seus processos e produtos, bem como contribuir para a permanente melhoria da qualidade de vida
da sociedade.
A Comisso

46

Olimpada Brasileira de Qumica - 2015

OBQ 2015

Questes Mltipla Escolha

QUESTES
MLTIPLA ESCOLHA

Questo 1 Quando pequenas quantidades de certas substncias so aquecidas, atravs de uma tcnica chamada teste da chama, elas emitem luz, visvel ou no. Por exemplo, o cloreto de sdio, quando aquecido, emite uma luz
amarela, caracterstica do sdio. Outros sais apresentam as mesmas caractersticas quando aquecidos, porm, com cores distintas. Por exemplo, o cloreto de clcio apresenta colorao vermelha; o cloreto de potssio, violeta; o
cloreto de brio, verde.
Devido ao fato de cada um dos tomos de metais citados emitir radiao
em comprimento de onda caracterstico (luz de cor especfica), o teste de
chama pode, ento, ser utilizado para a identificao destes elementos, teste
este baseado na Teoria Atmica. Para o teste de chama acima descrito correto afirmar que:
a) As cores so explicadas porque existe diferena de energia entre
nveis eletrnicos e, ao aquecer as substncias, ocorre excitao
eletrnica. O eltron, ao retornar sua orbita original, emite energia
na forma de luz visvel.
b) As cores que surgem no aquecimento so devidas a transies eletrnicas. Quando os eltrons so excitados, eles saltam de suas orbitas originais, liberando energia, na forma de luz visvel.
c) As cores observadas esto de acordo com a Teoria Atmica de Rutherford, cientista que estudou as rbitas eletrnicas.
d) As diferentes cores observadas devem-se ao nmero de nutrons
no ncleo de cada tomo, conforme estudos de Niels Bohr.
d) Somente os elementos dos subgrupos 1 e 2 apresentam essas propriedades, que foram previstas pelo qumico russo D. Mendeleiev.
Questo 2 As reaes qumicas so o corao da qumica. Compreender
a ocorrncia e os mecanismos das reaes qumicas permite ainda o entendimento de muitos processos que ocorrem em nossas vidas, como o metabolismo, a ao de medicamentos, o cozimento de alimentos, entre tantos
I

Olimpada Brasileira de Qumica - 2015

47

Modalidade A

Questes Mltipla Escolha

outros exemplos (Rosa, M. I. F. P. S.; Schnetzler, R. P. O Conceito de Transformao Qumica.


Qumica Nova na Escola, n. 8, 1998). Ao aplicar as reaes qumicas para quatro
metais distintos (A, B, C e D) foram obtidos os seguintes resultados.

I. Apenas B e C reagem com HCl 0,5 mol L-1 para produzir H2 no estado gasoso.
II. Quando o metal B adicionado a solues que contm os ons dos
outros metais, so formados A, C e D metlicos.
III. A reage com HNO3 6 mol L-1, mas D no reage.
Com base nas informaes acima,disponha os metais em ordem crescente como agentes redutores.
a) D < A < C < B
b) D < C < A < B
c) B < A < D < C
d) A < D < B < C
e) B < A < C < D
Questo 3 As substncias de valncia mista so aquelas que contm ons
em mais de um estado de oxidao formal, em uma mesma unidade molecular, que lhes atribuem propriedades supramoleculares originais com aplicabilidade em diversas reas: converso de energia, novos materiais, catlise e
eletrnica molecular, entre outros. O carter de valncia mista , na verdade,
responsvel pela colorao de vrios minerais bem conhecidos (Rocha, R. C.;

Toma, H. E. Transferncia de eltrons em sistemas inorgnicos de valncia mista. Qumica Nova,


v. 25, n. 4, p. 624-638, 2002). O elemento ferro forma um sulfeto com a frmula

aproximada Fe7S8 (mineral pirrotita). Suponha que o estado de oxidao do


enxofre -2 e que os tomos de ferro, ambos existem, se encontram nos
estados de oxidao +2 e +3. Qual a proporo de Fe2+ para Fe3+ na substncia?
a) 1,00

b) 1,33

c) 0,75

d) 0,40

e) 2,50

Questo 4 As transformaes qumicas so representadas por equaes


qumicas em que as substncias que sofrem transformao os reagentes
so escritas no lado esquerdo e as substncias formadas os produtos
aparecem no lado direito. As equaes qumicas devem ser balanceadas
de acordo com as leis ponderais, principalmente na lei da conservao das

48

Olimpada Brasileira de Qumica - 2015

Modalidade A

Questes Mltipla Escolha

d) um grama de cloro ocupa


d) 1,96
um grama
x 10 3dem3cloro
, 1,114575
ocupa x1,96
105xPa10e 31700
m3, 1,114575
K . Quaisxos
105 Pa e 17

massasvalores
e na lei
quais/o
nmero
de , supondo
de das
Kp, Kpropores
valores
de(ou
K/p,2 definidas),
afixas
reao
Kx2(g)
e KC paranas
a, reao
supondo
x e KC para
2comportamento
2(g)
tomos de cada tipo de elemento tem de ser igual nos reagentes e nos proideal?
ideal?
dutos, bem como as quantidades de cargas. Quando
a equao
5
5

Paa1600
0,071.
os valoresPa 0,071.
e) O grau de dissociao doe)cloro
O grau
a 1600
de dissociao
K e 1,01325x10
do cloro
K Quais
e 1,01325x10

- , supondo
de3(aq)
Kp, K
de
e KC para+a___H
reao
,+K___OH
comportamento
(g) 6]3- + ___NH
ideal?
, supondo
___NO
+x___Al(s)
O(K2)p(g)
(aq)a reao
___[A
(g) comportame
x e KC para
2(OH)
2
3

f) Uma clula eletroqumicaf) dada


Uma por:
clula
Pb/Pb
eletroqumica
dada
por:2 Pb/Pb
2
2(m)/Hg
2

2(m)/Hg2

4
1
equilibrada
corretamente
com
os
menores
coeficientes
de1,45x10
inteiros,
de 0,5357
V a 25 C e aumenta
de 0,5357
com
aVtemperatura
a 25 C
e aumenta
na razocom
de
anmeros
temperatura
na
V.K
. razo de 1,45x
qual a soma dos coeficientes dos reagentes e dos produtos, respectivamente.

f1) escreva
e a reao
reao
total de eletrodo e a reao total
a) 48 ef1)
11escreva ab)reao
48 ede
10eletrodo
c)
50 ea 10
d) 50 e 11
e) 49 e 11
f2) Calcule as propriedadesf2)
termodinmicas
Calcule as propriedades termodinmicas
rev

rev

Questo 5 As titulaes esto entre os procedimentos analticos mais


exatos. Em uma titulao, o analito reage com um reagente padronizado
(o titulante) em uma reao de estequiometria conhecida, em que a quantidade de titulante variada at que a equivalncia qumica seja atingida,
sendo esta equivalncia verificada pela mudana de cor de um indicador
PROBLEMA
PROBLEMA
6
ou pela
mudana6 na resposta
de um instrumento.
A quantidade do reagente padronizado necessria para atingir a equivalncia qumica relacionaO prmio
Nobel de qumica
prmio
E. J. Corey
Nobel(E.deJ.D.
qumica
Corey
; E.
Richard
J. Corey
D. (E.
Balanson.
J. Corey
J. ;Am.
Richard D. B
da com
a quantidade
de Oanalito
(Skoog,
A. Fundamentos
de
Qumica
Analtica,
8
e.,
Thomson,
2010,
1026
p).
Para
exemplificar,
foram
titulados
Chem. Soc., 1974, 96, 65166517)
Chem. Soc.,
usou1974,
um conjunto
96, 65166517)
de reaes
usou
nasumetapas
conjunto
de sntese
de reaes nas
2+
3+
25,0do
mL
de
uma
soluo
que
contm
ons
de
Fe
e
de
Fe
com
23,0
mL
alcalide poranterina. Adosntese
alcalide
da poranterina
poranterina.ocorre
A sntese
de acordo
da poranterina
com as seguintes
ocorre de acordo c
de KMnO4 0,0200 mol L-1 (em cido sulfrico diludo). Como resultado, todos etapas:
os ons de Fe2+ foram etapas:
oxidados para ons Fe3+. Em seguida, uma nova
alquota de 25,0 mL da soluo foi tratada com Zn metlico para converter
todos os ons de Fe3+ em ons de Fe2+. Finalmente, a soluo que contm
apenas os ons de Fe2+ consumiu 40,0 mL do mesmo titulante (soluo de
KMnO4) para a oxidao para Fe3+. Calcule as concentraes molares de
Fe2+ e de Fe3+ na soluo original. A equao inica simplificada
MnO4+5Fe2+ + 8h+

Mn2+ + 5Fe3+ + 4H2O

[Fe2+]/mol L-1 [Fe3+]/mol L-1


a) 0,0680
b) 0,0920
c) 0,0680
d) 0,0920
e) 0,0460

0,0920
0,0680
0,0460
0,0340
0,0340
I

Olimpada Brasileira de Qumica - 2015

49

Modalidade A

Questes Mltipla Escolha

Questo 6 O calor especfico pode ser definido como a quantidade de calor que um grama de determinado material deve ganhar ou perder para que
sua temperatura varie em um grau Celsius. O calor especfico dos metais
baixo quando comparado a materiais como argila ou pedra, todos materiais
usados na fabricao de panelas. Isso significa que, considerando panelas de
mesma massa, necessrio fornecer menos calor para o metal do que para a
argila para fazer com que ele atinja a temperatura de cozimento (Mortimer, E.
F.; Amaral, L. O. F. Calor e temperatura no ensino da termoqumica. Qumica
Nova na Escola, n. 7, 1998). Nesse contexto, e para efeito de comparao, so
dados abaixo os calores especficos de trs metais.
Metal
Calor especfico, J g

-1 o

-1

Fe

Pb

Zn

0,470

0,130

0,388

Se 1,00 g de cada metal aquecido a 100 oC e se adicionar 10,00 g de H2O


a 25,0 oC, qual ser a ordem das temperaturas das misturas finais a partir do
menor para o maior?
a) Fe < Zn < Pb
d) Fe < Pb < Zn

b) Pb < Zn < Fe
e) Pb < Fe < Zn

c) Zn < Fe < Pb

Questo 7 - O sangue humano um lquido ligeiramente bsico, tamponado por processos metablicos que mantm o pH entre 7,35-7,45. Para controlar o pH do sangue, o corpo usa inicialmente o sistema cido carbnico/
bicarbonato, conforme mostrado abaixo:
CO2 + H2O

H2CO3

H+ + HCO3

Se o pH sobe acima da faixa normal, a condio chamada de alcalose, cujo


valor limite de sobrevivncia por tempo reduzido 7,8. Quando o pH do
sangue est abaixo da faixa normal, a condio chamada de acidose e o
valor limite de sobrevivncia por tempo reduzido 7,0.
Sobre esse sistema-tampo so feitas as seguintes afirmaes:
I.

Respirando mais rpido e profundamente aumentamos a quantidade de


CO2 exalado e, assim, a concentrao de cido carbnico no sangue decresce, favorecendo a alcalose.

II. A inalao excessiva de fumaa aumenta a concentrao de CO2 no sangue, favorecendo a acidose.

50

Olimpada Brasileira de Qumica - 2015

Questes Mltipla Escolha

Modalidade A

III. O aumento da concentrao dos ons bicarbonato no sangue provoca


um aumento de pH, favorecendo a alcalose.
IV. A liberao excessiva de cido lctico durante a realizao de exerccios
fsicos pesados, provoca um aumento da concentrao hidrogeninica
no sangue, favorecendo a acidose.
Esto corretas:
a) Todas as alternativas
b) Somente I, II e III
c) Somente II, III e IV
d) Somente II e III
e) Somente I e IV

Questo 8 O equilbrio entre uma substncia slida e seus ons hidratados em


soluo fornece um exemplo de equilbrio heterogneo. A extenso do equilbrio na qual a reao de dissoluo ocorre expressa pela ordem de grandeza de sua constante de equilbrio, conhecida como constante do produto de
3
K
. Considerando-se
o1,96
conhecimento
algumas
regras
gerais
ps
grama de solubilidade,
clorod)ocupa
um grama
1,96
xde10cloro
m3ocupa
, 1,114575
x 10 53 Pa
m3e,de
1,114575
1700
K . xQuais
105 Pa
os
e 1700 K .
de precipitao e os equilbrios de solubilidade, em contraste, podemos fazer
ores de Kp,suposies
valores
de
Kp, Ksobre
Kx e K
a reao
para de
a reao
supondo
/2 2(g)comportamento
, supondo
quantitativas
quanto
certa, substncia
se dissolver
ou for- compo
C para
x /e2 K
C2(g)
mar precipitado. Assim, ao misturar 15,0 mL de 0,0040 mol L-1 de nitrato de
al?
chumboideal?
(II) com 15,0 mL de3cloreto
de sdio 0,0040
mol L-1, resultar:
3
5

5 x 10 Pa e 1700 K 5. Quais os
um
de cloro
ocupa
1,96
x 10K em1,01325x10
, 1,114575
Pa K0,071.
Quais os valores
Pa 0,071. Quais o
raugrama
de dissociao
e) Odo
grau
cloro
de
adissociao
1600
do
cloro a 1600
e 1,01325x10
valores
Kp, Kxa de
ereao
KKCPbC
parae(g)
aKreao
, supondo
comportamento
2+(g)
3 2+
32 , supondo
3K =
Pb
2C1,96
1,7x10
K
Kx edeKde
para
a/(aq)
reao
(g)comportamento
supondo
ideal?
ideaK
C para
p, K
x 22(s)
2(aq)
mp,grama
cloro d)
ocupa
um
grama
1,96
xCde
10
cloro
m
, ocupa
1,114575
xx10
105 Pa
me3,,1700
1,114575
K -5. comportamento
Quais
x 105 os
Pa e 1700
ps
deal?
a cluladeeletroqumica
dada
clulapor:
eletroqumica
Pb/Pb
dada
por:
Pb/Pb
(m)/Hg
2 /222 2(g)
2(m)/Hg,2 supondo
2
alores
Kp, Kf)x Uma
e Kvalores
K
,5 supondo
comportamento
com
C para adereao
p, 3K/
x 23e 2K
2(g)
C para2 a reao
5 10 Pa e 1700 K . Quais os
m
grama
de
cloro
m
,
1,114575
x
3ocupa 1,96 x 10
4,25
1,62
4
1 valores
4
Pa

0,071.
Quais
os
O0,5357
grau de
dissociao
do
cloro
a
1600
K
e
1,01325x10
Dado:
3 e 3aumenta
5 1,45x10 na
V ade
25cloro
C ede
aumenta
0,5357
com
V ax25
a10temperatura
C
na com
razo
de
temperatura
. de 1,45x10
V.K
deal?
ideal?1,96
um
grama
ocupa
m , 1,114575
x a10
Pa e 1700V.K
Krazo
. Quais
os
alores
de
K
,
K
e
K
para
a
reao
/
(g)
p
x
C
2
2
2+ , supondo comportamento
a)C para
Um slido
PbC 2(g)
ir precipitar, esupondo
ons Pb comportamento
em excesso iro
permanecer
de Kp, Kx e K
a reao
ideal?
5
Pa, supondo
0,071.
Quais
os valores
Pa 0,071. Quai
grau dede
e)eletrodo
O
grau
clorodea2reao
dissociao
1600 Kde
1,01325x10
do
cloro a5 1600
K
e 1,01325x10
valores
Kp, em
Kxdef1)
esoluo.
Kdo
/e2eletrodo
comportamento
C paraeaareao
2(g) e a reao
escreva
adissociao
reao
escreva
total
total
deal?
Uma clula eletroqumica dada por: Pb/Pb 2
2- (m)/Hg2
2
b)C para
Um slido
ir
ons C 25em
excesso iro
permanecer
em
e Kp, Kx e K
de KPbC
a reao
e Kprecipitar
,esupondo
(g)
comportamento
, supondo
ideal? comportamento
i
p, Kx2(g)
C para a reao
deal?
4 os valores
1
Pa de0,071.
Quais
Ode
grau
deasdissociao
do
cloroasa2com
1600
K e 1,01325x10
soluo.
0,5357
Vpropriedades
a 25
C
aumenta
a temperatura
na razo
1,45x10
V.K
.
Calcule
f2)eCalcule
termodinmicas
propriedades
termodinmicas
rev
rev
5
clula eletroqumica
f) Uma
dada
clula
por:
eletroqumica
Pb/Pb
dada 2por:
(m)/Hg
2
2
2 2Quais os2(m)/Hg
2
2
PaPb/Pb
0,071.
Oma
do cloro
1600
K e 1,01325x10
c) para
Um slido
PbC a(g)
ir
precipitar
meio aquoso.
e grau
K , Kde edissociao
K
a reao
,em
supondo
comportamento
ideal?valores
p

22

4
1
+
e 0,5357
areao
25Um
Cslido
aumenta
de
0,5357
V
aprecipitar
a25
temperatura
Ctotal
e aumenta
na Na
razo
com
adetemperatura
1,45x10
nairo
razo
de 1,45x10
V.K
. perma1)
escreva
de
eletrodo
e2(g)
a reao
d)
PbC com
ir
ons
e NO
em excesso
de
, Kx V
eaeletroqumica
K
ae reao
comportamento
ideal?
C para
2
maKpclula
dada
por:
Pb/Pb 2,esupondo
2(m)/Hg23
2
necer em soluo.
4
Uma
clulaV
dada
por:
2
2
0,5357
apropriedades
25 Cdee f1)
aumenta
aPb/Pb
temperatura
na razo
de 21,45x10
V.K 1.
1)e) Calcule
escreva
aeletroqumica
reao
eletrodo
escreva
ecom
aareao
reao
total
de eletrodo
e2(m)/Hg
a reao
total
as
termodinmicas

e) Uma soluo lmpida sem precipitado.

rev

de 0,5357 V a 25 C e aumenta com a temperatura na razo de 1,45x10 4 V.K 1.


1)
escrevaas
a reao
def2)
eletrodo
e aasreao
total termodinmicas
) Calcule
propriedades
Calcule
termodinmicas
propriedades
I
Olimpada Brasileira
rev de Qumica - 2015 I
BLEMA
PROBLEMA
1) escreva6 a reao
de eletrodo 6e a reao total

rev
51

Modalidade A

Questes Mltipla Escolha

Questo 9 O dispositivo experimental usado para produzir eletricidade,


a partir de uma reao espontnea, designado por clula galvnica ou
clula voltaica, em homenagem aos cientistas italianos Luigi Galvani e Alessandro Volta, que construram os primeiros prottipos do dispositivo. O
potencial da clula para a clula voltaica esquematizada abaixo de 0,109
V, sob condies padro, 1 mol L-1 de Ni2+(aq) e 1 mol L-1 Pb2+(aq). Que alterao nesta clula poderia causar um aumento na diferena de potencial
entre os eletrodos?

a)
b)
c)
d)
e)

Adicionar mais soluo de 1 mol L-1 de Pb2+ a essa semiclula.


Usar um eletrodo de Ni com maior massa.
Adicionar 50 mL de uma soluo 1 mol L-1 de NaCl para precipitar PbCl2.
Diluir com H2O a soluo de 1 mol.L-1 de Ni2+.
Usar um eltrodo de Pb com maior massa.

Questo 10 A qumica nuclear no pouco tempo de


histria da humanidade polmica e controversa,
mas inegvel a sua importncia no nosso cotidiano.
Como a meia-vida de qualquer nucldeo constante,
a meia-vida pode servir como um relgio nuclear para
determinar as idades de diferentes materiais. O 14C,
por exemplo, tem sido usado para determinar a idade
de materiais orgnicos (Figura 1). O procedimento
baseado na formao de 14C por captura de nutrons
na atmosfera superior:
N+ n

52

Olimpada Brasileira de Qumica - 2015

C+ p

Figura 1. O manto de Turin


declarado como sendo a
mortalha de Jesus Cristo foi
datado com 14C entre 1260
e 1390 d.C.

Modalidade A

Questes Mltipla Escolha

Essa reao fornece uma fonte de pequena, mas razoavelmente constante.


O 14C radioativo, sofrendo decaimento beta com meia-vida de 5.715 anos:
C

N +

(Brown, T. L. Qumica, a cincia central. 9 ed., Pearson Education do Brasil Ltda., 2014, 972p.).

Todas as seguintes sentenas abaixo so verdadeiras para o mtodo de datao por 14C, EXCETO:
a) A proporo de 14C/12C a mesma em organismos vivos terrestres como
na atmosfera.
b)

C sofre -decaimento para produzir 14N.

14

c) O teor de 14C de um organismo mantido constante durante sua vida e


inicia decrscimo depois de sua morte.
d) A datao por carbono igualmente til para as amostras que tem milhes de anos de idade, como para as amostras que tem cerca de 10.000
anos de idade.
e) A proporo de 14C/12C pode ser usada para datar uma amostra de um
organismo morto.

Olimpada Brasileira de Qumica - 2015

53

Modalidade A

Questes Analtico-Expositivas

QUESTES
ANALTICO-EXPOSITIVAS

Questo 11 A gua de um reservatrio de 500 m3 foi analisada quanto


ao teor de cloreto, considerando-se o padro de potabilidade estabelecido
pela Portaria N 2.914/2011 da ANVISA/MS, de limite mximo de 250 mg L1
para este parmetro. Dessa forma, durante a titulao, uma amostra de 100,0
mL de gua consumiu 11,5 mL de soluo de AgNO3 0,1 mol L1, somente
para reagir com os ons cloretos. Sabendo que esse reservatrio dever receber mais 400 m3 de gua, com teor de cloretos de 105 mg L1, responda:
a) Qual a massa de cloreto na amostra titulada?
b) Qual a concentrao inicial de cloretos no tanque?
c) Ao final da adio de gua no reservatrio, o teor de cloretos ir atender
a legislao vigente?
d) A fim de manter o teor de cloretos dentro do valor mximo permitido
pela portaria, qual o volume de gua livre de cloretos que poderia ser
adicionada ao reservatrio?
e) Qual a massa de sal de cozinha, em quilogramas, que poderia ser produzida com a massa de cloreto presente no reservatrio aps o recebimento dos 400 m3 de gua contendo 105mg.L1 de cloretos?
Questo 12 - As lmpadas incandescentes ou de filamento transformam
energia eltrica em energia luminosa e trmica, mas, progressivamente, esto sendo substitudas por outras de menor consumo, pois perdem em calor a maior parte da energia que consomem, e transformam em iluminao
apenas 5 % desta. Essas lmpadas utilizam um filamento de tungstnio que,
quando percorrido por uma corrente eltrica, torna-se incandescente, produzindo luz. Uma lmpada de 60 W, submetida a uma diferena de potencial
de 220 V, ligada quatro horas diariamente durante um ms em um cmodo
onde h uma pequena planta. Essa planta consegue aproveitar cerca de 10%
da energia luminosa que a atinge para a realizao da fotossntese. A partir
do exposto, responda:

54

Olimpada Brasileira de Qumica - 2015

Modalidade A

Questes Analtico-Expositivas

Dados:
o Entalpia de formao do dixido de carbono:94 kcal mol1
o Entalpia de formao da gua: 58 kcal mol1
o Entalpia de formao da glicose: 242 kcal mol1
o 1 cal
4J
o e = 1,6 x 1019 C
a) Qual a energia absorvida pela planta nesse perodo?
b) Qual o nmero de mols de gs oxignio gerado?
c) Quantos litros, aproximadamente, de oxignio so gerados, sendo que a
sala tem uma temperatura mdia de 104 F e presso de 1 atm?
d) Sabendo que um adulto consome em mdia 3 L de oxignio por minuto, quantas plantas, iguais a essa e recebendo energia nas mesmas propores, seriam necessrias para suprir esse consumo no perodo de 30
dias?
e) Quantos eltrons atravessaram a lmpada nesse perodo?
Questo 13 - O cido lctico, CH3CH(OH)COOH, recebeu esse nome porque est presente no leite azedo de gosto desagradvel como um produto
de ao bacteriana. tambm responsvel pela irritabilidade nos msculos
depois de exerccio vigoroso.
Dados: 10-3,85

1,4 x 10-4; 10-10,15

7,1x10-11

2,65. 10-2,7 = 1,6x10-3

a) A adio de hidrxido de sdio para reduzir a acidez causada pelo cido lctico formado pela ao de microrganismos no leite comercial para
consumo humano crime de adulterao de produtos alimentcios (art.
272 do Cdigo Penal). Considere uma concentrao de 1,8 g L1 de cido
lctico em um lote de 500 L de leite. Qual o volume necessrio para neutralizar completamente todo o cido contido nesse lote, sabendo que a
concentrao do hidrxido de sdio 0,5 mol L1.
b) O pKa do cido lctico 3,85. Compare esse valor com o valor para o
cido propinico (CH3CH2COO, pKa = 4,89) e explique a diferena.
c) Calcule a concentrao de on lactato em uma soluo de 0,050
mol.L1 de cido lctico.

Olimpada Brasileira de Qumica - 2015

55

Modalidade A

Questes Analtico-Expositivas

d) Quando o lactato de sdio, (CH3CH(OH)COO)Na, misturado com


uma soluo de cobre (II), possvel obter um sal slido de lactato de
cobre (II) como um hidrato de azul-esverdeado, (CH3CH(OH)COO)2Cu.
xH2O. A anlise elementar do slido nos diz que ele contm 22,9 % de Cu
e 26,0 % de C em massa. Qual o valor de x para o hidrato.
e) A constante de dissociao cida para o on Cu2+(aq) 1,0 x 10-8. Com
base nesse valor, determine se uma soluo de lactato de cobre (II) ser
cida, bsica ou neutra. Justifique sua resposta.
Questo 14 - O cido propanoico, CH3CH2COOH, um cido carboxlico
que reage com a gua de acordo com a equao abaixo.
H3C CH2 COOH (aq) + H20 (l) H3C CH2 COO- (aq) + H30+ (aq)
A 25 C o pH de uma amostra de 50,0 mL de CH3CH2COOH 0,20 mol L-1
2,79.
a) Identifique o par cido-base conjugado de Bronsted-Lowry na reao.
Rotule claramente qual o cido e o qual a base.
Determine o valor de Ka para o cido a 25 C.
a) Para cada uma das seguintes afirmaes, determinar se a afirmao
verdadeira ou falsa. Em cada caso, explicar o raciocnio que suporta a sua
resposta.
I.

O pH de uma soluo preparada pela mistura de 50,0 mL da amostra de


CH3CH2COOH 0,20 mol L-1 com uma amostra de 50,0 mL de NaOH 0,20
mol L-1 7,00.

II. Se o pH de uma soluo de cido clordrico o mesmo que o pH de uma


soluo de cido propanoico, em seguida, a concentrao molar da soluo de cido clordrico deve ser menor do que a concentrao molar da
soluo do cido propanoico.
b) Um estudante recebe a tarefa de determinar a concentrao de uma soluo de cido propanoico. Uma soluo NaOH 0,173 mol L-1 est disponvel para usar como titulante. O estudante utiliza uma pipeta volumtrica
de 25,00 mL para transferir a soluo de cido propanoico a um erlenmeyer limpo e seco. Aps a adio de um indicador apropriado para o erlenmeyer, o estudante titula a soluo com NaOH 0,173 mol L-1, atingindo

56

Olimpada Brasileira de Qumica - 2015

Modalidade A

Questes Analtico-Expositivas

o ponto final aps a adio de 20,52 mL de soluo de base. Calcule a


concentrao molar da soluo de cido propanoico.
c) O estudante solicitado para redesenhar a experincia para determinar a concentrao de uma soluo de cido butanoico (CH3CH2CH2
COOH) em vez de uma soluo de cido propanoico. Para o cido butanoico o valor de pKa 4,83. O estudante reivindica que um indicador
diferente ser necessrio para determinar o ponto de equivalncia da
titulao com preciso. Com base na sua resposta ao item (b), voc concorda com a afirmao do estudante? Justifique sua resposta.
Questo 15 Para a reao genrica abaixo, a 298 K:
3X2Y + WZ3
produtos
Foram obtidos os seguintes dados cinticos:
Experimento
I
II
III
IV

Concentrao inicial/mol L-1


[X2Y] 0
[WZ3]0
1,72
2,44
3,44
2,44
1,72
0,10
2,91
1,33

Velocidade Inicial/mol L-1 s-1


0,68
5,44
2,8 x 10-2
?

a) Em relao a cada reagente, determine a ordem da reao. Determine


tambm a ordem global da reao.
b) A partir das informaes da tabela, determine a Lei da Velocidade para a
reao:
c) A partir dos dados, determine o valor da Constante de Velocidade para a
reao genrica acima.
d) Utilizando os dados fornecidos, calcule a velocidade de reao para o
Experimento IV.
e) A velocidade de reao aumenta por um fator de 100 na presena de
um catalisador, a 298K. A energia de ativao aumentar, diminuir ou
permanecer a mesma? Justifique.

Olimpada Brasileira de Qumica - 2015

57

Modalidade A

Questes Analtico-Expositivas

Questo 16 Medicina Nuclear a especialidade que utiliza pequenas


quantidades de substncias radioativas ou traadores para diagnosticar
ou tratar certas doenas. Traadores so substncias que so atradas para
rgos especficos (os ossos por exemplo). Quando introduzidos no corpo
eles marcam as molculas participantes nesses processos fisiolgicos com
istopos radioativos. Estes denunciam sua localizao por emitirem radiao
nuclear (onda eletromagntica de comprimento de 0,01 a 1 nm do espectro dos raios gama). A deteco localizada de muitos ftons gama com uma
cmara gama permite formar imagens ou filmes que informem acerca do
estado funcional dos rgos. Entre os radioistopos mais utilizados est o
Tecncio-99 meta estvel, usado em exames de cintilografia do miocrdio e
os istopos de Iodo 123 e 131, usados nos diagnsticos da tireoide. Sobre as
informaes do texto responda os itens a seguir:
a) Entre os istopos mencionados o Iodo-131 emite partcula beta, os demais emitem apenas radiao gama. Escreva as suas equaes de decaimento utilizando a simbologia qumica apropriada.
b) A atividade de uma amostra radioativa ou taxa de decaimento a velocidade com que uma amostra se desintegra por unidade de tempo. No
S.I. sua unidade o becquerel (Bq) e equivale a uma desintegrao por
segundo. Qual a atividade de uma amostra com 2,0 x 1020 tomos de 99Tc,
se sua constante de decaimento for 3,2 x 105 s1?
c) As meias vidas dos radioistopos do iodo apresentados so, respectivamente, 13 horas para o 123, e 8 dias para o 131. Identifique o mais instvel e explique atravs de suas velocidades de decaimento (atividades),
considerando que ambos apresentam amostras com o mesmo nmero
de tomos. Dado: ln2 = 0,693
d) Uma amostra a ser usada em um exame de cintilografia miocrdica
rotulada com 99Tc, radioistopo que tem uma constante de decaimento
igual a 0,1155 h1. Caso tenha sido injetado 0,5 mg desse radioistopo no
corpo de um indivduo, quanto ele ainda apresentar em seu organismo
de tecncio-99 aps dois dias e meio?
e) Calcule a energia gerada por 0,5 mol de ftons mais energticos, em
MeV, ou seja, em milhes de eltron-volts, que so detectados pela cmara gama
Dados: constante de Planck, = 6,6 x 10-34 Js
velocidade da onda eletromagntica, c = 3,0 x 108 ms-1
1 eV = 1,6 x 10-19

58

Olimpada Brasileira de Qumica - 2015

OBQ 2015

Apresentao

Olimpada Brasileira
de Qumica Modalidade B
(3 SRIE) - 29/08/2015

LUZ, CINCIA E VIDA


Luz, cincia e vida o tema da 12 Semana Nacional de Cincia e Tecnologia - SNCT 2015 e baseia-se na deciso da Assembleia Geral das Naes
Unidas, que proclamou 2015 como o Ano Internacional da Luz, com objetivo
de celebrar a luz como matria da cincia e do desenvolvimento tecnolgico.
Em termos tecnolgicos, as lmpadas de LED que consomem menos
energia, a grande quantidade de dados transmitidos em alta velocidade via
cabos de fibra ptica e a obteno de energia solar, so alguns exemplos
de como a luz est ligada de forma visceral vida na terra e ao caminho da
humanidade.
A comodidade gerada por todo desenvolvimento cientfico e tecnolgico concede ao cotidiano, de grande parte da sociedade, energia eltrica,
gua tratada, transportes, alimentos, alm de diversos produtos teis ao homem, como plsticos, vidros, papel e tintas, que so resultados de processos
e transformaes qumicas.
Dessa forma, o desafio do ensino de qumica est em harmonizar o desenvolvimento tecnolgico e a conservao do meio ambiente, fazendo com
que os futuros consumidores, empresrios e cientistas tenham uma nova forma de pensar sobre este modo de vida.
Alm disso, socializar o conhecimento da cincia Qumica, a fim de desmistific-la como vil nos principais impactos das atividades humanas, e promover o aperfeioamento das indstrias qumicas, de forma a assegurar a
sustentabilidade ambiental, econmica e social de seus processos e produtos, bem como contribuir para a permanente melhoria da qualidade de vida
da sociedade.
A Comisso

Olimpada Brasileira de Qumica - 2015

59

Modalidade B

Questes Mltipla Escolha

QUESTES
MLTIPLA ESCOLHA

Questo 1 As reaes qumicas so o corao da qumica. Compreender


a ocorrncia e os mecanismos das reaes qumicas permite ainda o entendimento de muitos processos que ocorrem em nossas vidas, como o metabolismo, a ao de medicamentos, o cozimento de alimentos, entre tantos
outros exemplos (Rosa, M. I. F. P. S.; Schnetzler, R. P. O Conceito de Transformao Qumica.
Qumica Nova na Escola, n. 8, 1998). Ao aplicar as reaes qumicas para quatro
metais distintos (A, B, C e D) foram obtidos os seguintes resultados.
I. Apenas B e C reagem com HCl 0,5 mol L-1 para produzir H2 no estado gasoso.
II. Quando o metal B adicionado a solues que contm os ons dos
outros metais, so formados A, C e D metlicos.
III. A reage com HNO3 6 mol L-1, mas D no reage.
Com base nas informaes acima, disponha os metais em ordem crescente
como agentes redutores.
a) D < A < C < B
b) D < C < A < B
c) B < A < D < C
d) A < D < B < C
e) B < A < C < D

Questo 2 As transformaes qumicas so representadas por equaes


qumicas em que as substncias que sofrem transformao os reagentes
so escritas no lado esquerdo e as substncias formadas os produtos
aparecem no lado direito. As equaes
qumicas devem
ser 3balanceadas 5
d) um grama de cloro ocupa
d) 1,96
um grama
x 10 3dem3cloro
, 1,114575
ocupa x1,96
105xPa10e 31700
m , 1,114575
K . Quaisxos
10 Pa e 1700
de acordo com as leis ponderais, principalmente na lei da conservao das
de das
Kp, Kpropores
valores
de(ou
K/p,2 definidas),
afixas
reao
Kx2(g)
e KC paranas
a, reao
supondo
x e KC para
2comportamento
2(g)
massasvalores
e na lei
quais/o
nmero
de , supondo co
tomosideal?
de cada tipo de elemento
ideal? tem de ser igual nos reagentes e nos produtos, bem como as quantidades de cargas. Quando
5 a equao
5

Paa1600
0,071.
os valoresPa 0,071. Qu
e) O grau de dissociao doe)cloro
O grau
a 1600
de dissociao
K e 1,01325x10
do cloro
K Quais
e 1,01325x10

- , supondo
de3(aq)
Kp, K
de
e KC para+a___H
reao
,+K___OH
comportamento
(g) 6]3- + ___NH
ideal?
, supondo
___NO
+x___Al(s)
O(K2)p(g)
(aq)a reao
___[A
(g) comportamento
x e KC para
2(OH)
2
3

f) Uma clula eletroqumicaf) dada


Uma por:
clula
Pb/Pb
eletroqumica
dada
por:2 Pb/Pb
2
2(m)/Hg
2

60

Olimpada Brasileira de Qumica - 2015

2(m)/Hg2

2
4

de 0,5357 V a 25 C e aumenta
de 0,5357
com aVtemperatura
a 25 C e aumenta
na razocom
de 1,45x10
a temperatura
V.K na
. razo de 1,45x10

Modalidade B

Questes Mltipla Escolha

equilibrada corretamente com os menores coeficientes de nmeros inteiros, qual a soma dos coeficientes dos reagentes e dos produtos, respectivamente.
a) 48 e 11

b) 48 e 10

c) 50 e 10

d) 50 e 11

e) 49 e 11

Questo 3 As titulaes esto entre os procedimentos analticos mais exatos. Em uma titulao, o analito reage com um reagente padronizado (o titulante) em uma reao de estequiometria conhecida, em que a quantidade
de titulante variada at que a equivalncia qumica seja atingida, sendo
esta equivalncia verificada pela mudana de cor de um indicador ou pela
mudana na resposta de um instrumento. A quantidade do reagente padronizado necessria para atingir a equivalncia qumica relacionada com a
quantidade de analito (Skoog, D. A. Fundamentos de Qumica Analtica, 8 e., Thomson,
2010, 1026 p). Para exemplificar, foram titulados 25,0 mL de uma soluo que
contm ons de Fe2+ e de Fe3+ com 23,0 mL de KMnO4 0,0200 mol L-1 (em
cido sulfrico diludo). Como resultado, todos os ons de Fe2+ foram oxidados para ons Fe3+. Em seguida, uma nova alquota de 25,0 mL da soluo foi
tratada com Zn metlico para converter todos os ons de Fe3+ em ons de Fe2+.
Finalmente, a soluo que contm apenas os ons de Fe2+ consumiu 40,0 mL
do mesmo titulante (soluo de KMnO4) para a oxidao para Fe3+. Calcule
as concentraes molares de Fe2+ e de Fe3+ na soluo original. A equao
inica simplificada
MnO4+5Fe2+ + 8h+

Mn2+ + 5Fe3+ + 4H2O

[Fe2+]/mol L-1 [Fe3+]/mol L-1


a) 0,0680
b) 0,0920
c) 0,0680
d) 0,0920
e) 0,0460

0,0920
0,0680
0,0460
0,0340
0,0340

Questo 4 O equilbrio entre uma substncia slida e seus ons hidratados em soluo fornece um exemplo de equilbrio heterogneo. A extenso
do equilbrio na qual a reao de dissoluo ocorre expressa pela ordem
de grandeza de sua constante de equilbrio, conhecida como constante do
I

Olimpada Brasileira de Qumica - 2015

61

Modalidade B

Questes Mltipla Escolha

3
5
ma de cloro
d)ocupa
umde
grama
1,96
xde10cloro
mK3ocupa
,ps1,114575
1,96 x 10 53 Pa
mo3e,conhecimento
1,114575
1700 K . xQuais
10de
Pa
os
e 1700 K . Quai
produto
solubilidade,
. Considerando-se
algumas

regras gerais de precipitao e os equilbrios de solubilidade, em contraste,


podemos fazer suposies quantitativas sobre quanto de certa substncia
se dissolver
ideal? ou formar precipitado. Assim, ao misturar 15,0 mL de 0,0040
-1
3 (II)
mol.L
de
nitrato
de chumbo
com
15,0
de5cloreto
de sdio
mol
5 mL
5 . 0,0040
grama
de-1 e)
cloro
ocupa
1,96
x 10
m3,cloro
1,114575
xK
10
Pa eQuais
1700
os
Pa
0,071.
osK
valores
PaQuais
0,071.
Quais os val
de
dissociao
O
do
grau
cloro
de
a
dissociao
1600
K
e
1,01325x10
do
a
1600
e 1,01325x10
L , resultar:

de Kp, Kx e K
valores
Kp, Kx /e2 KC2(g)
a reao
para a reao
, supondo
/2 2(g)comportamento
, supondo comportam
C para de

ores
de para
Kp, K
eKKPbC
/3,2 supondo
(g) -(aq)
, supondo
comportamento
xreao
C para
22C
-5
(s)
Pb2+3a(aq)
=, supondo
1,7x10
K
a de
e22(g)
KaCxreao
para
reao
comportamento
ideal?
comportamento ideal?
x e KC de
p, Kx1,96
2(g)
grama
cloro
ocupa
10
m
, +1,114575
x 105KPa
ps e 1700 K . Quais os
al?
lula
eletroqumica
dada
clulapor:
eletroqumica
Pb/Pb
dada por:
Pb/Pb
(m)/Hg
2(m)/Hg2 5 2
res de
Kp, f)KUma
,2 supondo
comportamento
x e KC para a reao3 /223 2(g) 2
32 2 3
grama
de cloro d)
umcloro
grama
1,96
x1600
de
10 cloro
me,ocupa
1,114575
1,96x5x10
105 Pa
me ,1700
1,114575
K 1.os
Quais
xvalores
10 os
Pa e 1700 K .
3ocupa
4,25
1,62
4
Pa
0,071.
Quais
rau
de
dissociao
do
a
K
1,01325x10
57
C ede
aumenta
0,5357 com
V a 25
a temperatura
C e aumenta
na com
razoa de
temperatura
1,45x10 na
V.K
razo
. de
1,45x10 4 V.K 1.
l? V a 25Dado:
ores
de KK
K
Kreao
valores
adereao
Kpprecipitar
, K/
para
a reao
comportamento
, supondo compo
x eslido
C para
x 2e K
2,(g)
Cesupondo
2 excesso
2(g)
+ ,5 supondo
a)pC,cloro
Um
PbC
ir
ons Na
e Pa/em
permaKgrama
para
aocupa
ideal?
p, Kx ede
1,96a22x(g)
10 3Kme3,1,01325x10
1,114575
x5 comportamento
10
e 1700
K
. iro
Quais
os
Pa

0,071.
Quais
os
valores
au
de
dissociao
do
cloro
1600
em
soluo.
eva a reao necer
def1)eletrodo
escreva
eaareao
reaode
total
eletrodo e a reao total
al?
ideal?
a clula
eletroqumica
dada por:
Pb/Pb
2(m)/Hg
2
2
ores
de K
K
reao
/2 e
supondo
comportamento
x eslido
C para
2,2(g)
b)Cp, para
Um
PbC a22(g)
ir
precipitar
ons Pb2+5comportamento
em, excesso
iro
permanecer
em
K
aK
reao
supondo
ideal?
p, Kx e K
5
4 os valores
1Pa 0,071. Quais o
Pa

0,071.
Quais
rau
de
dissociao
e)
do
O
grau
cloro
de
a
dissociao
1600
K
e
1,01325x10
do
cloro
a
1600
K
e
1,01325x10
soluo.
0,5357
V
a
25
C
e
aumenta
com
a
temperatura
na
razo
de
1,45x10
V.K
.
ule
as
propriedades
f2)
Calcule
termodinmicas
as
propriedades
termodinmicas
rev
rev
al?clula eletroqumica dada por: Pb/Pb
2
2(m)/Hg2
2
c)C para
Um slido
ons C 2- 5(g)
em
excesso iro
permanecer
em
Kp, Kx e K
a reao
de KPbC
eirKprecipitar
,esupondo
comportamento
, supondo
ideal? comportamento
idea
p, Kx22(g)
C para a reao
4
1
Pa de
0,071.
Quais
os valores
rau
de V
dissociao
do
cloro com
ae 1600
K etotal
1,01325x10
,5357
25soluo.
C de
e aumenta
a temperatura
na razo
1,45x10
V.K
.
escreva
aareao
eletrodo
a reao
a clula eletroqumica
f) Umadada
clula
por:eletroqumica
Pb/Pb 2 dada 2por:
Pb/Pb
(m)/Hg
2
2 2
2(m)/Hg2
2
d)C para
Um slido
PbC 22(g)
ir precipitar ,em
meio aquoso.
Kp, Kx e K
a reao
supondo
comportamento ideal?
4
1
0,5357
apropriedades
25Uma
Cdeesoluo
aumenta
de 0,5357
aasem
25
temperatura
Ctotal
e aumenta
na razo
com adetemperatura
1,45x10
V.K
na razo
. de 1,45x10 4 V.
screva V
aase)
reao
eletrodo
ecom
aVreao
Calcule
termodinmicas
rev
lmpida
precipitado.
a clula eletroqumica
dada
por: Pb/Pb
(m)/Hg
2
2
2
2

0,5357
Va reao
apropriedades
25 Cde
e f1)
aumenta
a temperatura
na razo
de rev
1,45x10
escreva
eletrodo
escreva
ecom
aa reao
reao
total
de eletrodo
e a reao
total 4 V.K 1.
alcule as
termodinmicas
Questo
5

O
dispositivo
experimental
usado
para
produzir eletricidade,
MA 6
PROBLEMA 6
a partir de uma reao espontnea, designado por clula galvnica ou
escreva
aclula
reao
def2)
eletrodo
e as
a reao
total termodinmicas
alcule as
propriedades
Calcule
termodinmicas
propriedades
rev
rev
voltaica, em homenagem aos cientistas italianos
Luigi Galvani e AlesNobel desandro
qumica
O prmio
E.
Nobel
J.
Corey
de
qumica
(E.
J.
Corey
E.
J.
;
Corey
Richard
(E.
D.
J.
Balanson.
Corey
;
Richard
J.
Am.
D.
Balanson.
J.
Volta, que construram os primeiros prottipos do dispositivo. O poCalcule
propriedades
termodinmicas
rev
daSoc.,
clula
para 96,
aum
clula
voltaica
esquematizada
abaixo
de 0,109V,
., 1974,astencial
96,
Chem.
65166517)
1974,
usou
65166517)
conjunto
de usou
reaes
um nas
conjunto
etapas
dedereaes
sntese
nas etapas de sn
BLEMA sob
6 condies padro, 1 mol L-1 de Ni2+(aq) e 1 mol L-1 Pb2+(aq). Que altede poranterina.
do alcalide
A sntese
poranterina.
dapoderia
poranterina
A sntese
ocorre
deporanterina
acordo
ocorre
as seguintes
de
com as segui
rao
nesta
clula
causar
um da
aumento
nacom
diferena
de acordo
potencial
entre
os
eletrodos?
mio
Nobel
de qumica E. J. Corey (E. J. Corey ; Richard D. Balanson. J. Am.
LEMA
6 etapas:

Soc., 1974, 96, 65166517) usou um conjunto de reaes nas etapas de sntese
mio
Nobel
E. J. Corey
LEMA
6 de qumica
PROBLEMA
6 (E. J. Corey ; Richard D. Balanson. J. Am.
alide poranterina. A sntese da poranterina ocorre de acordo com as seguintes
Soc., 1974, 96, 65166517) usou um conjunto de reaes nas etapas de sntese
mio
Nobel
O prmio
E. J.
Nobel
Corey
de(E.
qumica
J. Corey
E. J.; Richard
Corey (E.
D. J.Balanson.
Corey ; Richard
J. Am. D. Balanso
6 de qumica
:LEMA
alide poranterina. A sntese da poranterina ocorre de acordo com as seguintes
Soc., 1974, 96, Chem.
65166517)
Soc., 1974,
usou um
96, conjunto
65166517)
de reaes
usou umnas
conjunto
etapas de sntese
reaes nas etapas d
mio Nobel de qumica E. J. Corey (E. J. Corey ; Richard D. Balanson. J. Am.
alide poranterina.
do alcalide
A sntese poranterina.
da poranterina
A sntese
ocorre da
de poranterina
acordo com ocorre
as seguintes
de acordo com as
Soc., 1974, 96, 65166517) usou um conjunto de reaes nas etapas de sntese
etapas:
alide poranterina. A sntese da poranterina ocorre de acordo com as seguintes
62

Olimpada Brasileira de Qumica - 2015

Modalidade B

Questes Mltipla Escolha

a)
b)
c)
d)
e)

Adicionar mais soluo de 1 mol L-1 de Pb2+ a essa semiclula.


Usar um eletrodo de Ni com maior massa.
Adicionar 50 mL de uma soluo 1 mol L-1 de NaCl para precipitar PbCl2.
Diluir com H2O a soluo de 1 mol L-1 de Ni2+.
Usar um eltrodo de Pb com maior massa.

Questo 6 A qumica nuclear no pouco tempo de


histria da humanidade polmica e controversa,
mas inegvel a sua importncia no nosso cotidiano.
Como a meia-vida de qualquer nucldeo constante,
a meia-vida pode servir como um relgio nuclear para
determinar as idades de diferentes materiais. O 14C,
por exemplo, tem sido usado para determinar a idade
de materiais orgnicos (Figura 1). O procedimento
baseado na formao de 14C por captura de nutrons
na atmosfera superior:
N+ n

C+ p

Figura 1. O manto de Turin


declarado como sendo a
mortalha de Jesus Cristo foi
datado com 14C entre 1260
e 1390 d.C.

Essa reao fornece uma fonte de pequena, mas razoavelmente constante.


O 14C radioativo, sofrendo decaimento beta com meia-vida de 5.715 anos:
C

N +

(Brown, T. L. Qumica, a cincia central. 9 ed., Pearson Education do Brasil Ltda., 2014, 972p.).

Todas as seguintes sentenas abaixo so verdadeiras para o mtodo de datao por 14C, EXCETO:
a) A proporo de 14C/12C a mesma em organismos vivos terrestres como
na atmosfera.
14
b) C sofre -decaimento para produzir 14N.
c) O teor de 14C de um organismo mantido constante durante sua vida e
inicia decrscimo depois de sua morte.
d) A datao por carbono igualmente til para as amostras que tem milhes de anos de idade, como para as amostras que tem cerca de 10.000
anos de idade.
e) A proporo de 14C/12C pode ser usada para datar uma amostra de um
organismo morto.
I

Olimpada Brasileira de Qumica - 2015

63

Modalidade B

Questes Mltipla Escolha

Questo 7 - Janaguba uma espcie arbrea que cresce at 7 m de altura,


com folhagem densa nas extremidades dos ramos. Sua distribuio geogrfica vai desde o sudeste do Brasil at a Guiana Francesa, Suriname e Guiana. No Brasil ocorre nos estados de Minas Gerais, Bahia, Sergipe, Alagoas,
Pernambuco, Rio Grande do Norte, Cear, Paraba, Piau, Maranho, Par e
Roraima (http://pt.wikipedia.org/wiki/Janaguba). uma planta nativa da Serra do
Araripe que, de acordo com a medicina popular, um santo remdio no
combate ao cncer e no tratamento de tumores, furnculos, edemas, artrites e ainda como vermfugo e laxante (http://diariodonordeste.verdesmares.com.br/
cadernos/regional/leite-da-janaguba-tem-propriedade-medicinal). Uma das substncias
existentes na seiva a plumericina.

Fonte: https://www.google.com.br/
search?q=janaguba&newwindow, acessado em 26/03/2015.

Plumericina

Janaguba

Sobre a plumericina so dadas as seguintes proposies.


I. Na molcula da plumericina ocorrem apenas as funes ter e ster.
II. Na molcula da plumericina ocorrem 5 carbonos assimtricos ou centros
estereognicos.
III. A frmula molecular da plumericina C15H14O6.
IV. Na molcula de plumericina h 3 (trs) tomos de carbono com hibridao sp, 7 (sete) com hibridao sp2 e 5 (cinco) com hibridao sp3.
V. Na molcula da plumericina h vrias duplas ligaes na cadeia, conferindo-lhe alta reatividade.
VI. A plumericina muito solvel em gua devido a presenas de vrios tomos de oxignio, que muito eletronegativo.
Marque a alternativa que indica apenas as proposies verdadeiras:
a) III, IV, V e VI
b) I, II, III e IV
c) I, II, III e V
d) I, II, V e VI
e) I, II, III, IV, V e VI

64

Olimpada Brasileira de Qumica - 2015

Modalidade B

Questes Mltipla Escolha

Questo 8 Luciferina uma substncia produzida por vagalumes e outros


seres vivos, produzindo bioluminescncia. A luciferina sofre oxidao, reao
catalisada pela enzima luciferase, e emite energia na forma de luz visvel. O
nome luciferina vem de lcifer (diabo), atribuda a uma crena antiga. Considerando a frmula estrutural da luciferina,

As funes orgnicas presentes na molcula so:


a) Fenol, tioter ou sulfeto, cido carboxlico e imina (amina insaturada)
b) lcool, tilcool, amida, cido carboxlico
c) cido carboxlico, amida, tioter ou sulfeto, lcool
d) Cetona, imina (amina insaturada), sulfona, fenol
e) Enol, aldedo, tiolcool, nitrila
Questo 9 Luminol uma substncia sinttica usada para obteno de
quimioluminescncia, usada em pescas submarinas, explorao de cavernas
e camping. O luminol reage com H2O2 emitindo luz azul e brilhante. Em anlises qumicas, usado como indicador de quimioluminescncia. Na qumica
forense usado para detectar resduos de sangue. O luminol ocorre nas seguintes formas:

O equilbrio entre as duas formas chamado de:


a) Diastereoisomeria
b) Enatiomeria
c) Tautomeria
d) Metameria
e) Epimeria

Olimpada Brasileira de Qumica - 2015

65

Modalidade B

Questes Mltipla Escolha

Questo 10 As reaes de oxidao so muito comuns no cotidiano. Por


exemplo:
I.

Quando usamos o permanganato de potssio, um sal violeta que podemos adquirir na farmcia, e o dilumos em gua para lavar feridas da catapora ou frieiras nos ps, estamos fazendo reao de oxidao branda.
Na Qumica esta soluo chamada de Reativo de Baeyer.

II. Nos bafmetros usado dicromato de potssio (alaranjado) dissolvido


em gua e cido sulfrico. Esta soluo, chamada soluo sulfocrmica, oxida o etanol a cido etanoico, mudando a colorao variando de
alaranjado para verde ou azul, de acordo com a concentrao de etanol
contida no ar expirado.
III. Na purificao de ambientes, materiais hospitalares e at no tratamento
da gua usado o oznio. A reao chamada ozonlise, seguida de
hidratao, na presena de zinco.
O esquema a seguir mostra as reaes quando o ciclo-hexeno submetido
s respectivas oxidaes.

Os produtos A, B e C so, respectivamente:


a) hexanodial, cido hexanodioico, ciclo-hexano-1,2-diol
b) ciclo-hexano, ciclo-hexanol, cido hexanoico
c) ciclo-hexano-1,2-diol, cido hexanoico, hexanal
d) hexanodial, ciclo-hexanol, cido hexanoico
e) cido hexanodioico, hexanal, ciclo-hexanol

66

Olimpada Brasileira de Qumica - 2015

Modalidade B

Questes Analtico-Expositivas

QUESTES
ANALTICO-EXPOSITIVAS

Questo 11 - As lmpadas incandescentes ou de filamento transformam


energia eltrica em energia luminosa e trmica, mas, progressivamente, esto sendo substitudas por outras de menor consumo, pois perdem em calor a maior parte da energia que consomem, e transformam em iluminao
apenas 5 % desta. Essas lmpadas utilizam um filamento de tungstnio que,
quando percorrido por uma corrente eltrica, torna-se incandescente, produzindo luz. Uma lmpada de 60 W, submetida a uma diferena de potencial
de 220 V, ligada quatro horas diariamente durante um ms em um cmodo
onde h uma pequena planta. Essa planta consegue aproveitar cerca de 10%
da energia luminosa que a atinge para a realizao da fotossntese. A partir
do exposto, responda:
Dados:
o
o
o
o
o

Entalpia de formao do dixido de carbono:94 kcal mol1


Entalpia de formao da gua: 58 kcal mol1
Entalpia de formao da glicose: 242 kcal mol1
1 cal
4J
e = 1,6 x 1019 C

a) Qual a energia absorvida pela planta nesse perodo?


b) Qual o nmero de mols de gs oxignio gerado?
c) Quantos litros, aproximadamente, de oxignio so gerados, sendo que a
sala tem uma temperatura mdia de 104 F e presso de 1 atm?
d) Sabendo que um adulto consome em mdia 3 L de oxignio por minuto, quantas plantas, iguais a essa e recebendo energia nas mesmas propores, seriam necessrias para suprir esse consumo no perodo de 30
dias?
e) Quantos eltrons atravessaram a lmpada nesse perodo?

Olimpada Brasileira de Qumica - 2015

67

Modalidade B

Questes Analtico-Expositivas

Questo 12 - O cido lctico, CH3CH(OH)COOH, recebeu esse nome porque est presente no leite azedo de gosto desagradvel como um produto
de ao bacteriana. tambm responsvel pela irritabilidade nos msculos
depois de exerccio vigoroso.
Dados: 10-3,85

1,4 x 10-4; 10-10,15

7,1x10-11 e

2,65

a) A adio de hidrxido de sdio para reduzir a acidez causada pelo cido lctico formado pela ao de microrganismos no leite comercial para
consumo humano crime de adulterao de produtos alimentcios (art.
272 do Cdigo Penal). Considere uma concentrao de 1,8 g L1 de cido
lctico em um lote de 500 L de leite. Qual o volume necessrio para neutralizar completamente todo o cido contido nesse lote, sabendo que a
concentrao do hidrxido de sdio 0,5 mol L1.
b) O pKa do cido lctico 3,85. Compare esse valor com o valor para o
cido propinico (CH3CH2COOH, pKa = 4,89) e explique a diferena.
c) Calcule a concentrao de on lactato em uma soluo de 0,050 mol L-1
de cido lctico.
d) Quando o lactato de sdio, CH3CH(OH)COO- +Na, misturado com
uma soluo de cobre (II), possvel obter um sal slido de lactato de
cobre (II) como um hidrato de azul-esverdeado, [CH3CH(OH)COO]2Cu.
xH2O. A anlise elementar do slido nos diz que ele contm 22,9 % de Cu
e 26,0 % de C em massa. Qual o valor de para o hidrato.
e) A constante de dissociao cida para o on Cu2+(aq) 1,0 x 10-8. Com
base nesse valor, determine se uma soluo de lactato de cobre (II) ser
cida, bsica ou neutra. Justifique sua resposta.
Questo 13 - O cido propanoico, CH3CH2COOH, um cido carboxlico
que reage com a gua de acordo com a equao abaixo.
H3C CH2 COOH (aq) + H20 (l) H3C CH2 COO- (aq) + H30+ (aq)
A 25 C o pH de uma amostra de 50,0 mL de CH3CH2COOH 0,20 mol L-1
2,79.
a) Identifique o par cido-base conjugado de Bronsted-Lowry na reao.
Rotule claramente qual o cido e o qual a base.

68

Olimpada Brasileira de Qumica - 2015

Modalidade B

Questes Analtico-Expositivas

b) Determine o valor de Ka para o cido a 25 C.


c) Para cada uma das seguintes afirmaes, determinar se a afirmao
verdadeira ou falsa. Em cada caso, explicar o raciocnio que suporta a sua
resposta.
I. O pH de uma soluo preparada pela mistura de 50,0 mL da amostra de CH3CH2COOH 0,20 mol L-1 com uma amostra de 50,0 mL de
NaOH 0,20 mol L-1 7,00.
II. Se o pH de uma soluo de cido clordrico o mesmo que o pH
de uma soluo de cido propanoico, em seguida, a concentrao
molar da soluo de cido clordrico deve ser menor do que a concentrao molar da soluo do cido propanoico.
d) Um estudante recebe a tarefa de determinar a concentrao de uma soluo de cido propanoico. Uma soluo NaOH 0,173 mol L-1 est disponvel para usar como titulante. O estudante utiliza uma pipeta volumtrica
de 25,00 mL para transferir a soluo de cido propanoico a um Erlenmeyer limpo e seco. Aps a adio de um indicador apropriado para o
Erlenmeyer, o estudante titula a soluo com NaOH 0,173 mol L-1, atingindo o ponto final aps a adio de 20,52 mL de soluo de base. Calcule
a concentrao molar da soluo de cido propanoico.
e) O estudante solicitado para redesenhar a experincia para determinar a concentrao de uma soluo de cido butanoico (CH3CH2CH2
COOH) em vez de uma soluo de cido propanoico. Para o cido butanoico o valor de pKa 4,83. O estudante reivindica que um indicador
diferente ser necessrio para determinar o ponto de equivalncia da
titulao com preciso. Com base na sua resposta ao item (b), voc concorda com a afirmao do estudante? Justifique sua resposta.
Questo 14 Medicina Nuclear a especialidade que utiliza pequenas
quantidades de substncias radioativas ou traadores para diagnosticar
ou tratar certas doenas. Traadores so substncias que so atradas para
rgos especficos (os ossos por exemplo). Quando introduzidos no corpo
eles marcam as molculas participantes nesses processos fisiolgicos com
istopos radioativos. Estes, denunciam sua localizao por emitirem radiao nuclear (onda eletromagntica de comprimento de 0,01 a 1 nm do espectro dos raios gama). A deteco localizada de muitos ftons gama com
uma cmara gama permite formar imagens ou filmes que informem acerca
I

Olimpada Brasileira de Qumica - 2015

69

Modalidade B

Questes Analtico-Expositivas

do estado funcional dos rgos. Entre os radioistopos mais utilizado est o


Tecncio-99 meta estvel, usado em exames de cintilografia do miocrdio e
os istopos de Iodo 123 e 131, usados nos diagnsticos da tireoide. Sobre as
informaes do texto responda os itens a seguir:
a) Entre os istopos mencionados o Iodo-131 emite partcula beta, os demais emitem apenas radiao gama. Escreva as suas equaes de decaimento utilizando a simbologia qumica apropriada.
b) A atividade de uma amostra radioativa ou taxa de decaimento a velocidade com que uma amostra se desintegra por unidade de tempo. No
S.I. sua unidade o becquerel (Bq) e equivale a uma desintegrao por
segundo. Qual a atividade de uma amostra com 2,0 x 1020 tomos de 99Tc,
se sua constante de decaimento for 3,2 x 105 s1?
c) As meias vidas dos radioistopos do iodo apresentados so, respectivamente, 13 horas para o 123, e 8 dias para o 131. Identifique o mais instvel e explique atravs de suas velocidades de decaimento (atividades),
considerando que ambos apresentam amostras com o mesmo nmero
de tomos. Dado: ln2 = 0,693
d) Uma amostra a ser usada em um exame de cintilografia miocrdica
rotulada com 99Tc, radioistopo que tem uma constante de decaimento
igual a 0,1155 h1. Caso tenha sido injetado 0,5 mg desse radioistopo no
corpo de um indivduo, quanto ele ainda apresentar em seu organismo
de tecncio-99 aps dois dias e meio?
e) Calcule a energia gerada por 0,5 mol de ftons mais energticos, em
MeV, ou seja, em milhes de eltron-volts, que so detectados pela cmara gama
Dados: constante de Planck, h = 6,6 x 10-34 Js
velocidade da onda eletromagntica, cx = 3,0 x108 ms -1
1 eV = 1,6 x 10-19 J
Questo 15 Retinal a substncia responsvel pela viso humana. Existem
as configuraes cis e trans. Quando a luz incide sobre a rodopsina - protena conjugada existente nos bastonetes da retina dos mamferos, transforma
o cis-retinal em trans-retinal, marcando o incio do processo visual, agindo
como elo entre a incidncia da luz e a srie de reaes qumicas que geram
o impulso nervoso que d origem viso.

70

Olimpada Brasileira de Qumica - 2015

Modalidade B

Questes Analtico-Expositivas

a) Considerando as molculas relacionadas, associe corretamente as frmulas estruturais com os respectivos nomes da segunda coluna:

b) Cis-retinal e trans-retinal so estereoismeros denominados diastereoismeros. Qual deles mais estvel: a configurao cis ou a configurao
trans? Justifique.
c) O betacaroteno, carotenoide existente em alimentos como abbora,
cenoura, mamo, manga, damasco, espinafre, couve, tem a funo de
ajudar a diminuir o risco de cncer. Quando ingerimos gorduras e protenas, o betacaroteno se converte em vitamina A, protegendo as clulas do
envelhecimento. Dada sua frmula estrutural,

pede-se:
c.1) sua funo orgnica
c.2) sua frmula molecular
c.3) o nmero de ligaes (pi) existentes na molcula.
d) Outro carotenoide importante a lutena da gema do ovo. Sua frmula
estrutural :

Olimpada Brasileira de Qumica - 2015

71

Modalidade B

Questes Analtico-Expositivas

Identifique os carbonos assimtricos (centros estereognicos) existentes na


molcula e quantos pares de enantimeros podem ocorrer.
Questo 16 Flavorizantes so essncias artificiais de frutas, imitando seu
cheiro e gosto. Os flavorizantes naturais so misturas de dezenas at centenas de substncias, onde uma ou duas so as principais. Assim, o flavorizante de banana o acetato de isoamila, de uva o antranilato de metila, de
abacaxi o butanoato de etila, de banana o etanoato (acetato) de octila etc.
Na indstria, esses flavorizantes so produzidos em grande quantidade por
reaes de esterificao, tambm conhecidas como esterificao de Fischer.
Quando se compra um preparado para bolo de laranja ou um suco de laranja
em p, na verdade, nestes so utilizadas essncias artificiais para conferir o
gosto e o cheiro, e corantes para conferir a cor.
a) Equacione a reao de obteno do acetato de octila.
b) As reaes de esterificao necessitam de sistema de refluxo e uso de
catalisador (geralmente o H2SO4). Sugira um mecanismo da reao, demonstrando a ao do catalisador.
c) As reaes de esterificao de Fischer so reversveis. Para deslocar o
equilbrio para a direita pode-se acrescentar um sal higroscpico no meio
reacional ou colocar um dos reagentes em excesso. Considerando-se o
uso de 24 g de cido etanoico (actico) e 26 g de octan-1-ol, demonstre
qual desses reagentes est em excesso.
d) de grande interesse da indstria que o rendimento da reao seja em
torno de 80 %. Considerando as massas citadas no item c, qual a massa
do produto necessria para ter um rendimento de 80 %?
e) Na reao de produo da essncia de laranja, qual oxignio permaneceu no ster como heterotomo: o do cido carboxlico ou o do lcool?
Justifique.

72

Olimpada Brasileira de Qumica - 2015

Solues escolhidas

OBQ Modalidade A

MELHORES
RESPOSTAS

Questo 11
Resoluo apresentada por Arthur Gabriel Moura Vieira, Colgio Militar, Juiz
de Fora - MG
Alternativa (a): AgNO3 + Cl- AgCl + NO3Limite: 250mg/L
Soluo de AgNO3: V1 = 11,5mL N = 0,1 mol/L; n1= 1,15 x 10-3 mol
Titulao: ncl= nAgNO3= 1,15 x 10-3 mol
Massa molar Cl- = 35,5 g/mol; Massa molar = 35,5 x 1,15 x 10-3 = 40, 285 mg.
Alternativa (b): Volume da amostra analisada: 100 mL.
N de mols Cl = ncl/V = 1,15 x 10-3/ 100 x 10-3 = 0,0115 mol/L
Ccl- = mcl- = 40,825 x 10-3 g / 100 x 10-3 L = 0,40825 g/L ou 408,25 mg.
Alternativa (c): Volume final: 400 m3 + 500 m3 = 900 m3
Massa de Cl- final = Massa inicial de Cl- + Massa adicionada de Cl= 408, 25 mg/L x 500 000 L + 105 mg/L x 400 000 L = 2041,25 x 105 mg + 420
x 105 mg = 2461,105
Concentrao final = Massa final / Volume final = 273,472 mg/L.
No, pois 273,472 mg maior que 250 mg/L.
Alternativa (d): Limite: 250 mg/ L
Volume de gua = V
250 mg/L = 2461,25 x 105 mg / V.;

V = 9,845 x 102m3 = 984,5 m3

O volume final de gua a ser acrescentado = volume final volume existente


VA = 984,5 900 = 84,5 m3
I

Olimpada Brasileira de Qumica - 2015

73

OBQ Modalidade A

Solues escolhidas

Alternativa (e): aps o recebimento dos 400m3 de gua contendo 105 mg/L
de cloreto, havia 2461,25 x 105 mg de cloretos no reservatrio. Como a massa de Cl- 35,5g/mol, o nmero de mols de Cl- na soluo, n, ://n = 2461,25.
105 x 103 g / 35,5 g/mol = aproximadamente 69,33 x 108 mol;
Como a formao do sal de cozinha, NaCl, por: Na+ + Cl- NaCl.
O nmero de mols de Cl- corresponde ao nmero de mols de NaCl possveis
de se formar, 69,33 x 108 mol. Como a massa molar do NaCl 58,5 g/mol, a
massa possvel de se formar de NaCl 69,33 x 108 x 58,5 = 4055,805 x 108 g.

Questo 12
Resoluo apresentada por Italo Rennan Lima Silva, Colgio 7 de Setembro,
Fortaleza - CE
a) Tempo em que a lmpada permanece acesa: t: 4h * 30 = 120 h =
120 * 3600 s. Energia consumida pela lmpada: E = pot * t= 60 w
* 120 * 3600 J. como apenas 5 % dessa energia usada para iluminao e a planta absorve apenas 10 %, temos:
E absorvida= 0,05*0,10*60*120*3600J =129600J= 129,6J,
Logo a planta absorve 129,6KJ.
b) Equao da fotossntese 6CO2 (g) + 6H2O (l) C6H12O6(s) + 6O2
Entalpia da reao: H = HF (C6H12O6) + * HF (O2) 6 * HF(CO2) 6
HF(H2O).
Calculando o nmero de mols de O2 gerado
N = 6 * E absorvida/H = 6 * 129,6 KJ / 2680 KJ/Mol N= 0,290 mol O2
c) Tc/5 = TF-32/9 Tc/5 = 104-32/9 Tc= 40 C= TK = 313 K
Pela equao de Clapeyron:
Pv=nRT -> 1 atm* V= 0,2909 mol* 0,0082 atm L mol -1 k-1 * 313 K
v = 7,45 L
Portanto so gerados, aproximadamente 7,45 L de O2.
d) Calculando a quantidade de litros de O2 consumidos:
VO2: 3L/min* 30 * 24 *60 min = 12900l.
Portanto, seriam necessrios 12900L/7,45L/planta = 17396 plantas.

74

Olimpada Brasileira de Qumica - 2015

OBQ Modalidade A

Solues escolhidas

e) Calculando a corrente que atravessa a lmpada:


Pot: U*i 60 w = 220 V * i i = 3/11 A
Sabendo que i = 2/t e Q= n*e
Ne/t = i i*t/e = 3/11*A*120*3600s/1,6.10-19 n = 7,36 . 10 23
Logo, aproximadamente 7,36. 1023 atravessam a lmpada nesse
perodo.

Questo 13
Ver resoluo apresentada por Joo Martins Cortez na questo 12 da OBQ
Modalidade B.

Questo 14
Resoluo apresentada por Celso Renan Barbosa Soares Lima, Colgio Master, Fortaleza - CE
a) H3C- CH2-COOH + H2O H3C- CH2-COO- + H3O+
cido

base

base

cido

conjugada

conjugada

b) pH = -log [H ]
+

- 2,79 = log [H+]


[H+] = 10-2,79
Ka = [H3C- CH2-COO-] . [H+] / [ H3C- CH2-COOH]
Ka = 10-2,79 . 10-2,79 / 0,20 - 10-2,79
Ka = 1,6. 10-3 . 1,6. 10-3 / 0,20 - 1,6. 10-3
Ka = 1,28 . 10-5
c) I - FALSO, pois quando o nmero de mols de cido e base, o que
determina o pH a hidrlise do sal, no caso H3C- CH2-COONa, o
qual possui hidrolise bsica.
I

Olimpada Brasileira de Qumica - 2015

75

VERDADEIRO, pois
pois oo cido
cido clordrico
clordrico um
um acido
acido forte
forte ee apresenta
apresenta
IIII -- VERDADEIRO,
++
+
ou2seja
seja liberar
liberar
H com
com mais
mais facilidade
facilidade ee portanto
portanto necessitando
necessitando de
de uma
uma c
CH
-COO
+ Na oo H
3C- ou
menor.
menor.
3C- CH2-COOH + OH

O- + H2

OBQ Modalidade A

er maior que 7

Solues escolhidas

-3-3

+
d)
NaOH==
0,173
.20,52
20,52
.10
NaOH
C-CH
CH22-C-CH22-COO
-CO
NaOH
0,173
.10
NaOH
++HH33C33C-CH
H3C-d)
CHnn2-COONa
H3C- .CH
-COO
+ Na
2
EIRO, pois o cido clordrico
um acido forte e apresenta menor
- pKa,
H C- CH2-COO- + H2O -3H
-33C- CH2-COOH + OH
NaOH==e3,55
3,55
10
NaOH==nnHH33CC-CH
CH22-COOH
-COOH
nnNaOH
..10
nnNaOH
o H+ com 3mais
facilidade
portanto
necessitando
de uma concentrao
Ou seja, o pH ser maior que 7

M==3,55
3,55..10
10-3-3//25
25..10
10-3-3
M

II - VERDADEIRO,
pois o cido clordrico um acido forte e apresenta menor
-3
0,173 . 20,52
.10ou
NaOH
+ H3C- CH
3C-CH2-COONa + H20
facilidade e portanto necessitando de
pKa,
seja liberar
o2-H+ com mais
uma
concentrao
menor.
5 . 10-3
n NaOH = nH C- CH -COOH

e) pKa
pKa==--log
logKa
Ka
e)

-1-1

2
4,83
-logKa
Ka
4,83
==-log

M = 3,55 . 10-3 / 25 . 10-3


-4,83
d)
n
NaOH
=
0,173 =
.=20,52
.10-3 NaOH + H3C- CH2-COOH H3C-CH2Kacido
cidobutanico
butanico
10-4,83
Ka
10
-1
-COONa + H20

Ka

ico = 10

No,npois
pois
os dois
cidos,
cido butanico
butanico
cido
propanico,
apresentam KK
No,
os
ee cido
propanico,
apresentam
n NaOH
= nH
C- CH2-COOH
NaOH
=dois
3,55cidos,
. 10-3 cido
3
4,83 = -log
Ka
-4,83
-4,83
-5-5
1,28 .M
. 10
10
ou. 10
seja
para
uma
titulao
distantes, respectivamente
respectivamente 10
10 ee 1,28
,, ou
seja
uma
titulao
distantes,
-3 para
-3
/ 25
. 10
= 3,55
-4,83
faixade
deviragem
viragemdo
doindicador
indicadorpoder
poderser
seraamesma
mesma
semperda
perdade
de
faixa
sem
M 0,140 mol.L-1

ois cidos, cido butanico e cido propanico, apresentam Ka no muito


- log
-log Kasemelhante a
-5 Ka
1,28 =. 10
, ou seja para4,83
uma =titulao
ctivamente 10-4,83e)e pKa
-4,83
m do indicador poder
a mesma
sem perda
de
Kaser
cido
butanico
= 10

preciso
preciso
No, pois os dois cidos, cido butanico e cido propanico,
apresentam Ka no muito distantes, respectivamente 10-4,83 e 1,28
. 10-5, ou seja para uma titulao semelhante a faixa de viragem do
indicador poder ser a mesma sem perda de preciso

QUESTO15
15
QUESTO
Questo 15

Resoluo apresentada por Vitor Gomes Pires, Colgio Etapa, So Paulo - SP


Resoluoapresentada
apresentadapor
porVitor
VitorGomes
GomesPires,
Pires,Colgio
ColgioEtapa,
Etapa,So
SoPaulo
Paulo--SP
SP
Resoluo
sentada por Vitor Gomes Pires, Colgio Etapa, So Paulo - SP
a) V0=[X2 Y] [WZ3]

a)
a)

A respeito dos experimentos I e II:

experimentos I e II:

a o reagente

respeito dos
dos experimentos
experimentos II ee II:
II:
AA respeito
Ordem
3para
para
reagente [X2 Y]...
Ordem
reagente
..Ordem
33para
oooreagente

mentos I e III:

o reagente

Para os experimentos I e III:

. os
Para
os experimentos
experimentos II ee III:
III:
Para
I
Olimpada Brasileira de Qumica - 2015
76 Ordem
Ordem
paraooreagente
reagente
11para

..

Resoluo apresentada por Vitor Gomes Pires, Colgio Etapa, So Paulo - SP


a)

OBQ Modalidade A

Solues
escolhidas
A respeito dos
experimentos
I e II:

Na+

. Ordem 3 para o reagente

aH+ + OH-

Para -os experimentos I e III:


a++
OH
Ordem 1 para o reagente
.
Ordem 1 para o reagente [WZ3].

+ OH-

um acido forte e apresenta menor pKa,


Ordem global=3+1=4
portanto necessitando de uma concentrao
um acido forte e apresenta menor pKa,
ortanto necessitando de uma concentrao 1
um acido forte e apresenta
menor
K assumindo
valor mostrado
no prximo
b)V=[X
Y]3 pKa,
[WZ3 ] , com, com
K assumindo
o valoro mostrado
no prximo
b)
2
ortanto
deouma
concentrao
, comnecessitando
K assumindo
valor
mostrado no prximo item.
item.

C- CH2-

3C-CH2-COONa

+ H2 0

-nCH
23C-CH
2-COONa + H20
NaOH
= nH3C- CH
2-COOH

c)c)

-3 CH
-3
- NaOH
CH
C-CH
+ H2 0
2- 3,55
= n.H10
3C2-COOH
M=
/ 325
. 102-COONa

-3 CH2-COOH
NaOH
= n. H
M
= 3,55
103C/ 25 . 10-1-3

-1-3
M = 3,55 . 10-3 / 25 . 10
-1

d) VoIV=[X2 Y] 3 [WZ3 ]=0,0557 2,913 1,331,82 mol/L s

cido propanico, apresentam Ka no muito


-5
, ou seja para uma titulao
semelhante a
d) energia
ido propanico, apresentam
no muito
e) A Ka
de ativao diminuir, como esperado de um cataliesma sem perda de
ao
diminuir,
como
esperado
deconstante
uma catalisador.
A constante deobedece, em geral, a equao
, ou seja
para uma
titulao
semelhante
sador.
A
de velocidades
ido propanico, apresentam Ka no muito
sma sem perda de
e
.a. Se
admitirmos
que,
K=K
e,, ou
emseja
geral,
equao
Se admitirmos
que,
parapara
a a reao catalisada, o fator
o
paraa uma
titulao
semelhante
pr-exponencial

o
mesmo
(K
),
ento
a diferena deve se dever
o
fator
pr-exponencial
o mesmo (Ko), ento a diferena deve
se
sma
sem
perda de
e) variao
A energia
de ativao
diminuir,
como
esperado
deeum
catalisador
da
energia
de
ativao,
j
que
R uma
constante
T=298
energia de ativao, j que R uma constante e
para este

K para este experimento.

. Se admitir
velocidades obedece, em geral, a equao
reao catalisada, o fator pr-exponencial o mesmo (Ko), ento a d
Questo
16 variao da energia de ativao, j que R uma constante e
dever
experimento.
, Colgio Etapa, So Paulo - SP

Resoluo
apresentada
porFortaleza
Dayanne
Rolim Carvalho, Farias Brito, Fortaleza - CE
Rolim Carvalho,
Farias Brito,
- CE
ada por Dayanne

Colgio
Etapa, So Paulo - SP
0

Tc + 00

a) - SP43Tc
Colgio Etapa, So Paulo
99

99
43

3I

+ 00

123
123 16 0
QUESTO
I
I+ 0
53
53

4I

+ 0 -1

I
I + -1
53
54
Resoluo
apresentada
por Dayanne Rolim Carvalho, Farias Brito, Fortalez

* 3,2 .10

131

131

b) A = K * N

99
0
Tc10 20 *993,2
a) A=
432.
.10 -5+ 0
. 43Tc

-5

A= 6,4. 10

15

Bq

123

53I

123

53I

+ 00

A= 6,4. 10 15 Bq
I

Olimpada Brasileira de Qumica - 2015

77

gio Etapa, So Paulo - SP

OBQ Modalidade A

Solues escolhidas

gio Etapa, So Paulo - SP

c)

t = 13h

t = 8 dias

123
131

Como transmutaes seguem a cintica de primeira ordem:


T = ln2/k assim,
k e inversamente proporcional ao tempo de meia.
vida de um radioistopo, menor sua velocidade de decaimento. En.
to, 131I mais estvel que o 123I, pois o primeiro tem maior tempo de
meia-vida. Numericamente:
V 131 I = ln2/ 8*24h
V 123 I = ln2/ 13 h
Tal que V 123I > V 131I
Como 123I decai mais rapidamente, ele o radioistopo mais instvel.
d) M = Mo * e kt
M= 0,5 * e 0,1155* 60
M= 0,5 * e 0,693 * 10 * (-1)
M= 0,5 * 2 -10
M = 2 -11 mg
e) E = hc/
E= (6,6 . 10 -34 j.s )* (3. 10 8 m/s) * (1/ 1. 10 -11m) . (1 ev / 1,6. 10 -19) *
(1 Mev / 10 6 Mev)
E= (0,12 Mev / 1 Fton) * (3. 10 23 ftons) = 3,6 .10 22

78

Olimpada Brasileira de Qumica - 2015

OBQ Modalidade B

Solues escolhidas

MELHORES
RESPOSTAS

Questo 11
QUESTO 12
Ver resoluo apresentada por Italo Rennan Lima Silva Questo 12 da OBQ
Resoluo apresentada
por Joo Martins Cortez, Instituto Dom Barreto, Teresina - PI
Modalidade
A.
a)
Em 500 12
mL de leite, h 900 g de cido lctico (1,8 g/L). A massa molar do cido
Questo
M= 90 g/mol, logo h 10 mols do cido. Como o cido lctico monocarboxlico, tem
Resoluo apresentada por Joo Martins Cortez, Instituto Dom Barreto, Teapenas- 1H
resina
PI ionizvel, vai precisar de 10 mols de NaOH, uma monobase. Assim, o
volume da soluo de NaOH 10/v = 0,5, v = 20 L.
a) Em 500 mL de leite, h 900 g de cido lctico (1,8 g/L). A massa molar do cido
M=
90 g/mol, logo h 10 mols do cido. Como o ciSero necessrios
20 L de
soluo.
do lctico monocarboxlico, tem apenas 1H ionizvel, vai precisar
de 10 mols de NaOH, uma monobase. Assim, o volume da soluo
de NaOH 10/v = 0,5, v = 20 L.
b) O pKa do cido lctico menor que o pKa do cido propinico, o que significa que o
Sero
necessrios
20 L=de
1,4soluo.
. 10 -4) maior que o Ka do cido propinico
pKa do cido
lctico
Ka = 10-3,85
-4,89
((Ka =b)10O pKa
), oudoseja,
o cido
lctico
mais
forte.
A explicao
esta nas estruturas
cido
lctico
menor
que
o pKa
do cido propinico,
o
qumicas. que significa que o pKa do cido lctico Ka = 10-3,85 = 1,4 . 10 -4)
maior que o Ka do cido propinico ((Ka = 10-4,89), ou seja, o cido
lctico mais forte. A explicao esta nas estruturas qumicas.

OH
H3C

C C
H

OH

O
H3C
OH

cido
cidolctico
lctico

C C

H3C

Base
Baseconjugada
conjugada

CH2 C

O
H3C

CH2 C

OH

cido
cido propinico
propinico

Baseconjugada
conjugada
Base

O
OHdo
do22carbono
carbono
cido
lctico
estabiliza
base conjugada
O grupo
grupo OH
no no
cido
lctico
estabiliza
a baseaconjugada
por meiopor
de um
meio de um efeito indutivo I, que dispersa mais a carga negativa na base
efeito indutivo I, que dispersa mais a carga negativa na base estabilizando-a. Tal grupo
estabilizando-a. Tal grupo no existe na base conjugada do cido propinino existe na base conjugada do cido propinico, tornando-a menos estvel que a base
co, tornando-a menos estvel que a base do cido lctico. De acordo com a
do cido
De acordo com
a teoria de dois
Bronsted-Lowry,
comparando
cidos,
teoria
delctico.
Bronsted-Lowry,
comparando
cidos, o mais
forte geradois
a base
o mais fortemais
gera afraca.
base conjugada
fraca.
Porisso,
o forte
cido que
lctico
mais forte que
conjugada
Por isso, omais
cido
lctico
mais
o propinico.
o propinico.
c) adotemos a notao: HLa = cido lctico
temos: HLa H+ + Lac) adotemos a notao: HLa = cido lctico
+

Olimpada Brasileira de Qumica - 2015

79

OBQ Modalidade B
incio:

reao:

M.

M. M.

fim:

M(1- ) M. M.

Solues escolhidas

Ka = 1,4 . 10 -4
Ka = M. 2 / (1- )

M = 0,05 = 5 . 10 -2

1,4 . 10 = 5 . 10 . / (1- )
-4

-2

5 2 = 1,4 . 10 -2 (1- )
5 2 + 1,4 . 10 -2 - 1,4 . 10 -2 = 0 Resolvendo a equao, temos:
= - 1,4 . 10-2 + 28. 10-2 /10 = 5,3. 10-1 - 1,4 . 10-2/ 10 = 5,16. 10-2
Portanto, a concentrao do lactato [La-] = M. = 2,58. 10-3 mol/ L.
A concentrao do on lactato 2,58.10-3 mol/L
d) hidrato = [CH3CH(OH)COO]2Cu x H2O = C6H10O6Cu x H2O
massa molar = 6.12 +1.10+ 6.16 +1.63,5 + 18 . x = (18 x + 241,5) g
considerando 1,0 mol temos: 100% --------------- (18 x + 241,5) g
26% ---------------- M (C) = 72 g
M total: x =2
Portanto, o valor de x x=2
e) A soluo de lactato de cobre(II) ser cida, porque o lactato de
cobre sofrer hidrlise, ambos os ons. O lactato, cujo cido conjugado o lctico, com pKa = 3,85, vai hidrolisar, e o Cu2+ vai hidrolisar, mas com um pKb maior que o pKa do cido lctico, pois sua
constante de dissociao cida 1,0 10-8. Portanto, o pKa (lactato)
> pKb (Cu2+), logo o pH da soluo aps hidrolise ser cido. Para
um sal derivado de cido e base fracos, o pH do meio ser cido se
pKa > pKb, e bsico se pKb < pKa.

80

Olimpada Brasileira de Qumica - 2015

OBQ Modalidade B

Solues escolhidas

Questo 13
Ver resoluo apresentada por Celso Renan Barbosa Soares Lima em OBQ
Modalidade A, Questo 14

Questo 14
Ver resoluo apresentada por Dayanne Rolim Carvalho Questo 16 da
OBQ Modalidade A,

Questo 15
Resoluo apresentada por Valentino Amadeus Sichinel, Colgio Politcnico
da UFSM, Santa Maria - RS
a) Retinol - B

cido Retinico - C

Cis-Retinal - D

Trans-Retinal - A

b) A configurao trans a mais estvel, pois seu arranjo espacial


mais linear, o que lhe confere maior estabilidade devido a melhor distribuio dos orbitais, estando elementos iguais em planos
opostos.
c) 1- Hidrocarboneto
2- C40H56
3- 11 ligaes d) 3 carbonos assimtricos, 23=8, 4 pares de enantimeros.

Questo 16
Resoluo apresentada por Leandro Alves Cordeiro, Liceu Jardim, Ribeiro
Pires, SP
a)
H3CCOOH + HOCH2CH2CH2CH2CH2CH2CH2CH3
H3CCOOCH2CH2CH2CH2CH2CH2CH2CH3
b) NO RESPONDEU (Resoluo sugerida pela banca)

Olimpada Brasileira de Qumica - 2015

81

OBQ Modalidade B

Solues escolhidas

Mecanismo - 1 - Dissociao do cido sulfrico:

2 - protonizao do cido actico

3 - ataque do lcool

4 - Regenerao do cido sulfrico e formao do acetal

5 - Desidratao do acetal e formao do ster

82

Olimpada Brasileira de Qumica - 2015

OBQ Modalidade B

Solues escolhidas

c)
C2H4O2 = 60 g/mol
C8H18O = 130 g/mol
60 g------130 g

130X = 60 x 26 X = 12 g de C2H4O2

X g-------26 g
O reagente em excesso o C2H4O2 (cido etanoico).
d)
26 g------X g

X = 34,4 g

130 g-------172 g
34,4 x 0,8 27,52 g
e) O oxignio que permaneceu no ster como heterotomo foi o do
lcool, visto que a hidroxila do cido utilizado para formar a molcula de gua. Devido a ressonncia da carboxila o cido carboxlico
forma um on mais estvel que o do lcool.

Quando voc perceber que, para produzir precisa obter a


autorizao de quem no produz nada; quando comprovar que
o dinheiro flui para quem negocia, no com bens, mas com
favores; quando perceber que muitos ficam ricos pelo suborno
e por influncia, mais que pelo trabalho, e que as leis no nos
protegem deles, mas, pelo contrrio, so eles que esto protegidos
de voc; quando perceber que a corrupo recompensada, e a
honestidade se converte em auto sacrifcio; ento poder afirmar,
sem temor de errar, que sua sociedade est condenada.
Ayn Rand
Filsofa russo-americana (judia, fugitiva da revoluo russa, que
chegou aos Estados Unidos na metade da dcada de 1920)
mostrando viso com conhecimento de causa.
I

Olimpada Brasileira de Qumica - 2015

83

OBQ Modalidade A

Resultados

Olimpada Brasileira
de Qumica Modalidade A
RESULTADO
Nome

Escola
OURO
Etapa
So Bento
Ari de S
Bernoulli
Objetivo
Ari de S
Olimpo
Farias Brito

So Paulo
Rio de Janeiro
Fortaleza
Belo Horizonte
Indaiatuba
Fortaleza
Goinia
Fortaleza

SP
RJ
CE
MG
SP
CE
GO
CE

80
72,5
72,1
71,2
70,6
70,2
68,0
67,7

PRATA
Etapa
7 de Setembro
Master Bezerra
Magnum
Farias Brito
Dom Barreto
Militar de Fortaleza
Farias Brito
So Bento
Bandeirantes
Farias Brito
Simbios
Master
Etapa
Madre Maria Villac
Adventista
Leonardo da Vinci

So Paulo
Fortaleza
Fortaleza
Belo Horizonte
Fortaleza
Teresina
Fortaleza
Fortaleza
Rio de Janeiro
So Paulo
Fortaleza
Goinia
Fortaleza
Valinhos
Teresina
Porto Alegre
Taguatinga

SP
CE
CE
MG
CE
PI
CE
CE
RJ
SP
CE
GO
CE
SP
PI
RS
DF

66,5
66,2
65,9
65,5
65,4
65,4
65,1
64,0
64,0
63,8
62,4
61,6
61,4
60,3
59,4
58,5
58,2

83,13
82,75
82,38
81,88
81,75
81,75
81,38
80,00
80,00
79,75
78,00
77,00
76,75
75,38
74,25
73,13
72,75

BRONZE
Magno Agostiniano
Santo Incio
Anglo Sorocaba
Colgio Militar
Ari de S
Albert Sabin
Contato
Etapa
Etapa
Militar

Belo Horizonte
Rio de Janeiro
Sorocaba
Porto Alegre
Fortaleza
Cotia
Macei
So Paulo
So Paulo
Juiz de Fora

MG
RJ
SP
RS
CE
SP
AL
SP
SP
MG

57,1
56,2
55,2
55,2
54,8
54,6
53,9
53,7
52,7
52,6

71,38
70,25
69,00
69,00
68,50
68,25
67,38
67,13
65,88
65,75

Vitor Gomes Pires


Lucas Paulo de Lima Camillo
Gerardo Albino Nogueira Filho
Eduardo Custdio Leal
Felipe Mourad Pereira
Gabriel Ferreira Gomes Amgarten

Leonardo Augusto Garcia Cunha

Francisco Luiz Isael Junyor

Pedro Seber e Silva


Italo Rennan Lima Silva
Tiago de Sousa Viana
Gustavo Dehaini
Carlos Alfredo C. de Vasconcelos Filho
Thiago de Carvalho Gonalves Nunes
Davi Oliveira Arago
Gustavo Manfio Leme de Campos
Joo Paulo Carvalho da Cruz
Mateus Nakamune Tubone
Dayanne Rolim Carvalho
Lucca Borges Prado
Celso Renan Barbosa Soares Lima
Eduardo de Oliveira Prates Fantini Parma
Rubens Henrique F. dos Reis
Lucas Kendrick Dal Castel
Leonardo Almeida Lessa
Leandro Guatimosim Gripp
Bernardo Sobral Werneck
Diogo Correia Netto
Guilherme Goulart Kowalczuk
Amanda Camelo Paulino
Rafael Peixoto Pagliaro
Lus Felipe Chagas Caldeira Cato
Guilherme Obeid
Andr Oliveira Soares
Arthur Gabriel Moura Vieira

84

Olimpada Brasileira de Qumica - 2015

Cidade

UF Nota Escore
100,0
90,63
90,13
89,00
88,25
87,75
85,00
84,63

Resultados

Ana Beatriz Timb de Oliveira


Gabriel Moura Brauna
Rodrigo Massato Nobetani
Danilo Soares Barbosa
Andr Victor Fernandes Bacci
Marcos Montandon Magalhes
Leonardo Mouta Pereira Pinheiro
Victor Moreira Santos
Luciana Filoni Ferreira
Vinicius Abreu de Morais
Andr Saugo Mazzari
Vincius Castagna Lepca
Beatriz Albuquerque Bomfim
Isabela Borges Barreto
Joo Raphael Ferreira Guimares
Gabriel Henrique Cabezas Assis
Vincius Roberto Gomes Queiroz
Jonny Farias Vicente Ferreira
Adrienny Rbia de Oliveira Soares
Renan Muniz dos Santos
Arianny Vitoria Rocha Alves
Rodolfo Nogueira Lima
Matheus Cardoso Arago
Henrique Hackbart Porn
Victor Cambraia Nogueira de Oliveira
Luana Jalantonio Cattan

OBQ Modalidade A
Master
Ari de S
Etapa
3 Milnio
Singular So Bernardo
Etapa
Olimpo
Master
Objetivo
Ruy Barbosa
Notre Dame
Positivo Ang. Sampaio
Militar
Integral
Santa Maria
7 de Setembro
Diocesano
So Bento
Militar
Pensi
Master
Ari de S
Ari de S
Marista Rosrio
Farias Brito
Objetivo Integrado

Fortaleza
Fortaleza
So Paulo
Limoeiro
So Bernardo
So Paulo
Braslia
Aracaju
Guarulhos
Trs Rios
Campinas
Curitiba
Manaus
Salvador
Recife
Fortaleza
Caruar
Rio de Janeiro
Juiz de Fora
Rio de Janeiro
Fortaleza
Fortaleza
Fortaleza
Porto Alegre
Fortaleza
So Paulo

CE
CE
SP
PE
SP
SP
DF
SE
SP
RJ
SP
PR
AM
BA
PE
CE
PE
RJ
MG
RJ
CE
CE
CE
RS
CE
SP

52,6
52,1
52,0
51,9
51,6
51,6
51,4
51,2
50,7
50,7
50,6
50,6
50,3
49,3
49,3
49,2
48,8
48,7
47,6
47,6
47,5
47,2
46,6
46,3
46,2
46,1

65,75
65,13
65,00
64,88
64,50
64,50
64,25
64,00
63,38
63,38
63,25
63,25
62,88
61,63
61,63
61,50
61,00
60,88
59,50
59,50
59,38
59,00
58,25
57,88
57,75
57,63

PRATA
Pedro Seber e Silva
Etapa
Italo Rennan Lima Silva
7 de Setembro
Tiago de Sousa Viana
Master Bezerra
Gustavo Dehaini
Magnum
Carlos Alfredo Cordeiro de Vasconcelos Filho
Farias Brito
Thiago de Carvalho Gonalves Nunes
Dom Barreto
Davi Oliveira Arago
Militar de Fortaleza
Gustavo Manfio Leme de Campos
Farias Brito
Joo Paulo Carvalho da Cruz
So Bento
Mateus Nakamune Tubone
Bandeirantes
Dayanne Rolim Carvalho
Farias Brito
Lucca Borges Prado
Simbios
Celso Renan Barbosa Soares Lima
Master
Eduardo de Oliveira Prates Fantini Parma
Etapa
Rubens Henrique F. dos Reis
Madre Maria Villac
Lucas Kendrick Dal Castel
Adventista
Leonardo Almeida Lessa
Leonardo da Vinci

So Paulo
Fortaleza
Fortaleza
Belo Horizonte
Fortaleza
Teresina
Fortaleza
Fortaleza
Rio de Janeiro
So Paulo
Fortaleza
Goinia
Fortaleza
Valinhos
Teresina
Porto Alegre
Taguatinga

SP
CE
CE
MG
CE
PI
CE
CE
RJ
SP
CE
GO
CE
SP
PI
RS
DF

66,5
66,2
65,9
65,5
65,4
65,4
65,1
64,0
64,0
63,8
62,4
61,6
61,4
60,3
59,4
58,5
58,2

83,13
82,75
82,38
81,88
81,75
81,75
81,38
80,00
80,00
79,75
78,00
77,00
76,75
75,38
74,25
73,13
72,75

Olimpada Brasileira de Qumica - 2015

85

OBQ Modalidade B

Resultados

Olimpada Brasileira
de Qumica Modalidade B
RESULTADO
Nome

Escola
OURO

Cidade

UF

Nota Escore

Teresina
Fortaleza
Ribeiro Pires
Teresina
Aracaju
Teresina
Aracaju

PI
CE
SP
PI
SE
PI
SE

84,40
82,00
81,50
81,20
77,60
76,75
76,50

100,0
97,17
96,58
96,22
91,96
90,95
90,65

Rio de Janeiro
So Paulo
Teresina
Fortaleza
Rio de Janeiro

RJ
SP
PI
CE
RJ

75,50
75,40
74,60
73,75
73,00

89,47
89,35
88,40
87,39
86,51

Fortaleza
Belo Horizonte
Recife
Fortaleza
So Paulo
Braslia
Santa Maria
Braslia
Fortaleza
Fortaleza
So Lus
So Jos do Rio
Preto
Porto Alegre
Teresina

CE
MG
PE
CE
SP
DF
RS
DF
CE
CE
MA
SP

71,70
71,60
71,20
71,00
70,80
69,45
68,75
68,70
68,40
68,30
68,25
67,95

84,96
84,85
84,37
84,14
83,90
82,30
81,47
81,41
81,05
80,94
80,88
80,52

Felipe Espreafico Guelerman Ramos


Colgio FAAP
Ribeiro Preto
Joo Pedro Ramos Milhome
Dom Barreto
Teresina
Arthur Klemenchuk Sueiro
Liceu Jardim
Santo Andr
Yu Hao Wang Xia
CMB
Braslia
Valentino Amadeus Sichinel
Politcnico da UFSM Santa Maria

SP
PI
SP
DF
RS

Felipe Vieira Coimbra


Pedro Teotonio de Sousa
Leandro Alves Cordeiro
Joo Martins Cortez Filho
Delson Barros Oliveira Filho
Pedro Guilherme Soares Vieira
Joao Guilherme Porto Santos

Dom Barreto
Ari de S
Liceu Jardim
Dom Barreto
Coesi
Dom Barreto
Amadeus
PRATA

Gabriel Pineschi Braun


Elcio Koodiro Yoshida
Victria Moreira Reis Cogo
Giovanni Elson Rafael de Souza
Wallace Ferreira Tefilo
Joo Pedro de Oliveira Morais da Costa
Marina Mendes Fonseca
Gustavo de Souza Medeiros
Renner Leite Lucena
Leonardo Henrique Martins Florentino
Roberto Ferreira Franco Moura
Lucas Samuel Formolo
Mikael Akihitto Hirata Iwamoto
Lucas Bastos Oliveira
Juan Freire Dantas Galvo
Juliana Mendes Sousa
Leonardo Dias Sanabria

PH
Etapa
Dom Barreto
Farias Brito
Sistema Elite de
Ensino
Antares Papicu
Santo Antnio
Motivo
Farias Brito
Objetivo Integrado
Olimpo
Colegio Riachuelo
Olimpo
Master Sul
Farias Brito
Educallis
Colgio London

Gabriel Kripka
caro Arajo de Sousa

Israelita Brasileiro
Instituto Federal
BRONZE

86

Olimpada Brasileira de Qumica - 2015

RS 67,15 79,57
PI 66,60 78,92

65,75
65,25
65,10
65,10
65,00

77,91
77,32
77,14
77,14
77,03

OBQ Modalidade B

Resultados

Rafael Ferreira Martins


Felipe Batista Meirelles
William Chaves Lima
Lucas Maia Morais
Arthur Coutinho Valadares

Joo Lus Sousa Guedes Alcoforado


Rafael Verissimo Martins
Matheus Faria Mello
Paula Coelho Gyori
Girgio Franciscatto Pereira
Bernardo Schmitberger Moraes
Bruno Diniz Rocha Pechina
Caio Koiti Ogata Ariga
Matheus Ucha Constante
Lorenna Conti L. L. Frana da Silva
Thales de Oliveira Sabino
Marcus Vincius Faustino
Victor Raniery Silva de Holanda
Matheus Ribeiro Sampaio

CEFET-MG
Bom Jesus
Farias Brito
Cincias Aplicadas
Elite Vale de Ao

Marista So Lus
Olimpo
Colgio Militar
Albert Sabin
Politcnico da UFSM
CEFET-MG
Liberato Salzano
Objetivo Vergueiro
Olimpo
Instituto Federal
Instituto Federal
Colgio Militar
Instituto Federal
Master

Belo Horizonte
Curitiba
Fortaleza
Natal
Ipatinga

Recife
Braslia
Juiz de Fora
So Paulo
Santa Maria
Belo Horizonte
Novo Hamburgo
So Paulo
Goinia
Rio de Janeiro
Rio de Janeiro
Belo Horizonte
Parnamirim
Aracaju

MG
PR
CE
RN
MG

PE
DF
MG
SP
RS
MG
RS
SP
GO
RJ
RJ
MG
RN
SE

64,50
63,90
62,60
62,30
62,00

61,90
61,40
60,90
60,35
60,20
59,90
59,35
59,10
58,70
58,60
58,50
58,40
58,40
58,20

76,43
75,72
74,18
73,83
73,47
73,35
72,76
72,17
71,51
71,34
70,98
70,33
70,03
69,56
69,44
69,32
69,20
69,20
68,97

Veja lista de agraciados com Meno Honrosa e Demais Classificados em www.obquimica.org

Voc nunca sabe quais resultados viro da sua ao.


Mas se voc no fizer nada, no existiro resultados.
Mahatma Gandhi
I

Olimpada Brasileira de Qumica - 2015

87

Processo Seletivo

Progama
Nacional
Olimpadas
de Qumica

Fase IV

OLIMPADA BRASILEIRA
DE QUMICA 2014 - FASE IV
PROVA EXPERIMENTAL EM VDEO
PROCESSO SELETIVO PARA AS
OLIMPADAS INTERNACIONAIS DE QUMICA

Nome:
email:

Cdigo

Caro estudante,
Este exame de cunho experimental tem por finalidade selecionar os 15
(quinze) estudantes que participaro do Curso de Aprofundamento e Excelncia (Fase V), para a futura escolha dos representantes do Brasil nas olimpadas internacionais de Qumica.
Voc dispe de 4 (quatro) horas para ver o vdeo e responder s questes
deste exame.

INSTRUES
1. A prova contm 10 (sete) questes, que abrangem os 10 (dez) experimentos contidos no
vdeo.
2. Veja atentamente, na projeo, as imagens do filme que contm os fundamentos deste
exame.
3. Seu coordenador, inicialmente, exibir a gravao completa do exame e, a seguir, apresentar cada experimento separadamente. Caso seja necessrio, ele repassar as imagens,
at esclarecer as suas dvidas.
4. Leia as perguntas relativas a cada experimento, constantes nesta folha, e escreva as respostas nas folhas oficiais de respostas, nos espaos destinados a cada questo.
5. Os resultados desse exame sero encaminhados para o seu coordenador (e tambm diretamente para voc, caso tenha e-mail). Veja o resultado, tambm, na internet em www.
obquimica.org (clique em novidades).

88

Olimpada Brasileira de Qumica - 2015

Processo Seletivo

Fase IV

QUESTO 1 (EXPERIMENTO 1) - Nas condies apresentadas, determine a densidade do gs produzido e a pureza do carbonato de sdio
utilizado.
Dados: P = 1,0 atm; T = 27oC; R = 0,082 atm.mol-1.K-1
Massas atmicas: C = 12 u; Cl = 35,5 u; H = 1 u; O = 16 u; Na = 23 u.
QUESTO 2 (EXPERIMENTO 2) - Explique o porqu da colorao inicial
do suco e a mudana de colorao observada no tubo de ensaio aps a
adio da gua de bromo, identificando as fases 1 e 2.
QUESTO 3 (EXPERIMENTO 3) - Identifique as amostras de leite modificadas, propondo possveis substncias adulterantes. Justifique.
QUESTO 4 (EXPERIMENTO 4) - Equacione a transformao qumica
apresentada e explique o efeito observado quando o feixe de luz atravessa os tubos de ensaio.
QUESTO 5 (EXPERIMENTO 5) - Determine a concentrao, em mg/L,
de cloretos na gua da torneira.
Dados: Massas atmicas: Ag=108 u; Cl=35,5 u; Cr=52 u; K = 39 u; N=14 u;
O = 16 u.
QUESTO 6 (EXPERIMENTO 6) - Explique, equacionando as reaes, o
aparecimento das coloraes.
QUESTO 7 (EXPERIMENTO 7) - Considerando a estrutura da cistena,
presente no ovo, e os potenciais de reduo fornecidos, explique as transformaes observadas no objeto de prata.
Dados: Ag+(aq) + e - Ag (s) E0 = + 0,80 V
Al3+(aq) + 3e -

Al (s) E0 = - 1,66 V
Cistena

QUESTO 8 (EXPERIMENTO 8) - Segundo a anlise apresentada, qual


dos nions abaixo est presente na soluo eletroltica?
a) Cloreto.

b) Fosfato.

c) Nitrato.

d) Sulfito.

Explique.
QUESTO 9 (EXPERIMENTO 9) - Explique a formao das chamas, equacionando as reaes ocorridas na palha de ao.
QUESTO 10 (EXPERIMENTO 10) - Explique as transformaes qumicas observadas.
I

Olimpada Brasileira de Qumica - 2015

89

Processo Seletivo

Progama
Nacional
Olimpadas
de Qumica

Fase IV

OLIMPADA BRASILEIRA
DE QUMICA 2014 - FASE IV
Resultado da prova experimental - 2015

Aps a finalizao desta etapa tivemos como classificados para a fase seguinte, Curso de Aprofundamento e Excelncia em Qumica, os seguintes estudantes:
Nome

Escola

UF

Felipe Vieira Coimbra


Instituto Dom Barreto PI
Joo Martins Cortez Filho
Instituto Dom Barreto PI
Giovanni Elson Rafael de Souza
Farias Brito
CE
Gabriel Moura Brana
Ari de S Cavalcante CE
Italo Lesione De Paiva Rocha
Master Bezerra
CE
Dayanne Rolim Carvalho
Modulo /Objetivo
CE
Pedro Teotonio de Sousa
Ari de S Cavalcante CE
Gabriel Ferreira Gomes Amgarten
Ari de S Cavalcante CE
Seon Augusto de Souza Ferreira
Colgio Militar
BA
Renata Braga de Sousa Cidrack
Master Bezerra
CE
Elcio Koodiro Yoshida
Etapa
SP
Gabriel Pineschi Braun
Colgio PH
RJ
Rafael Wendel Carvalho Cruz
COLTEC
MG
Lorenzo Pellizzaro Lima
Coleguium
MG
Vitor Alexandre Santos da Silva
IFRJ - Maracan
RJ
Leonardo Henrique M. Florentino
Objetivo Vergueiro
SP
Vitor Gomes Pires
Campos Salles
SP
Matheus Henrique de Al. Camacho
Objetivo Paulista
SP

Escore Escore ResultaFASE III FASE III do Final


100,0 82,48
93,04
93,40 91,76
92,75
91,92 93,82
92,68
89,59 93,82
91,28
85,78 98,98
91,06
89,91 91,76
90,65
89,59 81,45
86,33
85,99 86,60
86,24
90,02 80,42
86,18
83,82 89,70
86,17
75,61 100,0
85,37
84,46 84,54
84,49
83,13 84,54
83,70
88,00 73,20
82,08
77,62 88,67
82,04
73,55 93,82
81,66
90,02 69,08
81,64
74,02 92,79
81,53

Lista completa desse resultado pode ser vista em www.obquimica.org/resultados/download/141


Os estudantes acima relacionados foram convocados para participar do Curso de Aprofundamento e
Excelncia em Qumica ministrado na Universidade Federal do Rio Grande do Norte no perodo de
01.03 a 14.03.2015.
O curso foi fundamentado na lista de exerccios existente em:
www.obquimica.org/exames/download/155

90

Olimpada Brasileira de Qumica - 2015

Processo Seletivo

Fase V

Progama
Nacional
Olimpadas
de Qumica

OLIMPADA BRASILEIRA
DE QUMICA 2014 - FASE V
Curso de Aprofundamento
e Excelncia em Qumica

De 01 a 14 de abril de 2015 realizou-se no Instituto de Qumica da Universidade Federal do Rio Grande do Norte o Curso de Aprofundamento e Excelncia em Qumica ministrado por professores do Curso de Ps-Graduao
em Qumica da UFRN e docente convidado. Trata-se de um curso itinerante
que ministrado por dois anis consecutivos em uma das universidades parceiras que possuam Programa de Ps-graduao em qumica.
A UFRN ainda ministrar pela segunda vez esse curso, em 2016, para os estudantes classificados na Fase IV da OBQ-2015.

Alunos do Curso de Aprofundamento e Excelncia em Qumica com o coordenador estadual, Prof. Dr. Fabiano Gomes, o
Diretor do Instituto de Qumica, Prof. Dr. tom Anselmo de Oliveira e a Reitora da UFRN, Profa. Dra.Angela Ma Paiva Cruz.

Olimpada Brasileira de Qumica - 2015

91

Processo Seletivo

Fase VI

Constantes fsicas, Unidades, Frmulas e Equaes


ConstantesBRASILEIRA
fsicas, Unidades, Frmulas e Equae
OLIMPADA
Progama

DE QUMICA 2014 - FASE VI

Exame
aplicado em
18.05.2015
ConstanteNacional
universal dos gases
..........................R
= 8,3145
J K-1 mol-1
Constante universal dos gases ..........................R = 8,3145 J K-1 mol-1
Olimpadas

PROCESSO SELETIVO PARA AS

Pressode
padro
..................................................p = 1 bar = 105 Pa
Qumica
OLIMPADAS
INTERNACIONAIS DE QUMICA
Presso
padro ..................................................p
= 1 bar = 105 Pa

Presso atmosfrica ..........................................1 atm = 1,01325 105 Pa = 760 mmHg 5


Constantes
Unidades,
Frmulas e Equaesatm = 1,01325 10 Pa
Pressofsicas,
atmosfrica
..........................................1
Zero na escala Celsius ......................................273,15 K
Zerodos
na escala
Constante universal
gases .Celsius
. . . ......................................273,15
. . . . . . . .R = 8,3145 JK-1Kmol-1

-31
Massa
do eltron
...............................................m
Presso
padro
. . . . . . . . . . . . . e .= .9,1094
. . . . 10
p = 1kgbar = 105 Pa -31
kg
Massa
do eltron ...............................................m
e = 9,1094 10

Presso atmosfrica . . . . . . . 1 atm = 1,01325 105 Pa = 760 mmHg

mol-1
Eltron-volt .......................................................1 eV = 1,6022 10-19
Zero na escala Celsius
. . .......................................................1
. . . . . . . . . . . . . . . . . eV
. 273,15
K 10-19
Eltron-volt
= 1,6022
MassaLei
do da
eltron
. . . . . ........................
. . . . . . . . . . . me = 9,1094 10 kg
Primeira
Termodinmica
.......................
Eltron-volt . . Primeira
. . . . Lei
. . da. Termodinmica
. . .1 eV = 1,6022
10-19 J = 96485 Jmol-1
-31

Trabalho
de Lei
umda
gsTermodinmica
..........................................
= trabalho po
Primeira
. . . . . . . . . . . . . .(compresso
+ dW
Trabalho de um gs
..........................................
(compr
expanso
=
trabalho
negativo)
Trabalho de um gs . . . . . . dW=-pdV (compresso = trabalho positivo;
expanso = trabalho nega
expanso = trabalho negativo)

;
Relao entre capacidades calorficas ..............
Relao entre
capacidades
Relao entre capacidades
calorfi
cas . . calorficas
. . . . . ..............
. . . .

(para gases
gasesdiatmicos)
diatmicos)
(para
(para gases diat
Problema 1

Ciclos termodinmicos

150 pontos

Desde as suas proposies em 1824, o ciclo e o teorema de Carnot so algumas das maiores contribuies da termodinmica clssica para a cincia das
mquinas trmicas. A mquina de Carnot aquela que operando entre duas
temperaturas, uma fonte de calor e um sorvedouro de calor, possui a mxima
eficincia possvel, sendo a eficincia calculada como a razo entre o trabalho produzido pelo ciclo e o calor absorvido da fonte temperatura quente.
O ciclo de Carnot compreende as etapas:
1) Expanso isotrmica reversvel do gs a uma temperatura quente T1 (Figura 1 = sequncia de 1 a 2);

92

Olimpada Brasileira de Qumica - 2015

Processo Seletivo

Fase VI

2) Expanso adiabtica (ou isoentrpica) reversvel (trabalho isoentrpico


realizado pelo gs Figura 1 = sequncia de 2 a 3);
3) Compresso isotrmica reversvel do gs a uma temperatura fria T2 (Figura 1 = sequncia de 3 a 4);
4) Compresso adiabtica (ou isoentrpica) reversvel (trabalho isoentrpico
realizado sobre o gs Figura 1 = sequncia de 3 a 4).

Figura 1. Diagrama Presso x Volume (P x V) ilustrando o ciclo de


Carnot para a restrio de que x = n = Cp/Cv = .

Observa-se na Figura 1 que o ciclo de Carnot somente ser corretamente


ilustrado se admitirmos a restrio de que x = n = = Cp/Cv, onde Cp e Cv
so as capacidades calorficas molares do gs a presso e volume constante,
respectivamente. Em outras palavras, devemos escolher os estados, tal que:
S2 S1 = S4 S3, onde Si a entropia do estado i (a isoentropia do processo
garante que S2 = S3 e S1 = S4). Os parmetros n e x so denominados de ndices politrpicos, iguais ao expoente de Poisson do gs, , no ciclo de Carnot.
Diferentes ciclos termodinmicos podem ser estudados modificando os valores dos ndices politrpicos e usando ou no a restrio de n = x.
1. Para o ciclo de Carnot, a deduo da equao = 1 (T2/T1), para o rendimento do processo, pode ser feita sem o uso explcito da segunda lei da
termodinmica. Mostre tal deduo para o modelo de gs ideal.
2. Na Figura 2 so apresentados dois ciclos termodinmicos conhecidos
como ciclo de Stirling (esquerda) e ciclo de Ericsson (direita). Nestes ci3e4
1) no so utilizados, sendo
clos os processos adiabticos (2
substitudos, respectivamente, por processos isovolumtricos e isobricos. Demonstre que ambos os ciclos apresentam rendimento igual ao da
mquina de Carnot, quando operam reversivelmente com um gs ideal.
I

Olimpada Brasileira de Qumica - 2015

93

Processo Seletivo

Fase VI

Figura 2. Ciclos termodinmicos de Stirling (esquerda) e de Ericsson (direita).

3. Qual o valor de n (e x, pois considere n = x) nos ciclos de Ericsson e Striling?


4. Uma mquina trmica opera com um mol de um gs ideal diatmico realizando um ciclo que consiste de trs etapas: (1) Uma expanso adiab4. Uma mquina trmica opera com um mol de um gs ideal diatmico realizando um ciclo que consiste de trs
tica de uma presso inicial de 2,64 atm e um volume inicial de 10 L para
etapas: (1) Uma expanso adiabtica de uma presso inicial de 2,64 atm e um volume inicial de 10 L para uma
uma presso
final
de
atm
e(2)
volume
final
dea20
L;presso
(2) Uma
compresso
4. Uma
mquina
comfinal
um 1
mol
gs
diatmico
realizando
um
ciclo
que
consiste
de trs
presso final detrmica
1 atm eopera
volume
de
20de
L;um
Umaideal
compresso
uma
constante
para
o seu volume
a
uma
presso
constante
para
o
seu
volume
original
de
10
L,
e
(3)
um
etapas:
(1)
Uma
expanso
adiabtica
de
uma
presso
inicial
de
2,64
atm
e
um
volume
inicial
de
10
L
para
umaa
original de 10 L, e (3) um aquecimento a volume constante at a sua presso original de 2,64 atm. Calcule
presso
final
de
1
atm
e
volume
final
de
20
L;
(2)
Uma
compresso
a
uma
presso
constante
para
o
seu
volume
aquecimento
a
volume
constante
at
a
sua
presso
original
de
2,64
atm.
eficincia do ciclo.
original deCalcule
10 L, e (3)
um aquecimento
a volume constante at a sua presso original de 2,64 atm. Calcule a
a eficincia
do ciclo.
eficincia do ciclo.

ProblemaProblema
2. Lei de 2velocidade (150 pontos)
Lei de velocidade

150 pontos

Problema
2. Lei
, apontos)
uma dada
temperatura,
reao
NOdada
e (150
Htemperatura,
A reaoAentre
NO de
eentre
Hvelocidade
descrita
pela equao: descrita pela equao:
2, a uma
2
k

obs
A reao entre NO e H2, a uma dada temperatura,
descrita
pela
equao:
2H (g) 2NO(g)
N (g)
2H O(g)

kobs

Como ocorre
reduo
da presso
no 2H
decorrer
da
reao,
a variao
2)/dt pode ser medida experimentalmente
2NO(g)
N 2 (g) dP(N
2H
Como
ocorre
reduo
da2 (g)
presso
no decorrer
da2 O(g)
reao,
a variao dP(N2)/
pela diminuio da presso total, tal como mostra a tabela abaixo. A expresso genrica que descreve
dt
pode
ser
medida
experimentalmente
pela
diminuio
damedida
presso
total,a lei de
Como
ocorre
reduo
da presso
no decorrer da reao, a variao dP(N2)/dt pode ser
experimentalmente
velocidade
para
essa reao
:
tal como
a tabela
abaixo.
genrica
quegenrica
descreve
a lei dea lei de
pela diminuio
da mostra
presso total,
tal como
mostraAaexpresso
tabela abaixo.
A expresso
que descreve
dP(N
)/dt=
[P(H
)]
[P(NO)]
2
obs
2
velocidade
para
essa
reao
:
velocidade para essa reao :
dP(N2)/dt=
Po(H2)
(torr)

obs [P(H2)]

Po(NO)
(torr)

[P(NO)]

P(H2)
(torr.s-1)
(velocidades
iniciais)

/ t

1 289encontre a 400
1,60 experimental, especificando os valores de ,
1. Com base nos dados experimentais
lei de velocidade
2 147
400
0,77
e obs.
3 400encontre a lei
300 de velocidade
1,03experimental, especificando os valores de ,
1. Com base nos dados experimentais
4 400
152
0,25
2. Abaixo apresentada
uma
proposta
mecanstica
para
essa
reao:
e obs.
k

1
I
Olimpada
de Qumica
I essa reao:
2. Abaixo
umaBrasileira
proposta
mecanstica
94 apresentada
H 2 - 2015
NO para
NO N 2 O H 2 O

k1

pela diminuio da presso total, tal como mostra a tabela abaixo. A expresso genrica que descreve a lei de
dP(N2)/dt= obs [P(H2)] [P(NO)]
velocidade para essa reao :
dP(N2)/dt=

obs [P(H2)]

[P(NO)]

Processo Seletivo

Fase VI

base
nos dados
experimentais
encontre
a lei deespecificando
velocidadeosexperi1. Com1.
baseCom
nos dados
experimentais
encontre
a lei de velocidade
experimental,
valores de ,
mental,
.
e obs. especificando os valores de
1. Com base nos dados experimentais encontre a lei de velocidade experimental, especificando os valores de ,
2. Abaixo
apresentada
uma proposta mecanstica
para essa mecanstica
reao:
2. Abaixo
uma proposta
para essa reao:
e obs. apresentada
k

2. Abaixo apresentada uma proposta mecanstica


essa1 reao:
H 2 NOpara
NO
N 2O H 2O

H2

k1

NO NOk2 N 2 O H 2 O
H 2 N 2O
N 2 H 2O
k2

Faa as Faa
suposies
necessrias para
que o mecanismo
leimecanismo
de2 O
velocidade emprica.
Expresse
H 2 para
N 2 O gere
Na2o
H
obs em termos
as suposies
necessrias
que
gere a lei
de velocidas constantes
de
velocidade
das
etapas
individuais
do
mecanismo.
em termos
de velocidade
emprica.
Expresse
Faa as dade
suposies
necessrias
para que o mecanismo
gere a das
lei deconstantes
velocidade emprica.
Expresse obsdas
em termos
etapas
individuais
do
mecanismo.
3. Outra
proposta
mecanstica
essaindividuais
reao apresentada
abaixo.
das
constantes
de velocidade
daspara
etapas
do mecanismo.
1
Outra
proposta
mecanstica
para
essaabaixo.
reao apresentada abaixo.
3. Outra3.proposta
mecanstica
para
essa
NOreao
NO apresentada
N O (rpido,
equilbrio)

k1
k1

NO NO N 2 Ok22 (rpido, equilbrio)


k
H 2 N1 2 O 2 N 2 O H 2 O

H 2 N 2O2
H 2 N 2O
H

NO

k2
k3
k3

N 2O H 2O
N 2 H 2O
N

HO

2
2
Faa as suposies necessrias para que o mecanismo
gere a2 lei de2 velocidade emprica. Expresse obs em termos
das constantes de velocidade das etapas individuais do mecanismo.
as suposies
necessrias
paragere
quea lei
o mecanismo
gere a Expresse
lei de velociFaa as Faa
suposies
necessrias para
que o mecanismo
de velocidade emprica.
obs em termos
4. Qual
dos mecanismos
propostos
o mais
plausvel?
Justifique.
dade
emprica.
Expresse
kobs
emdotermos
das constantes de velocidade das
das
constantes
de
velocidade
das etapas
individuais
mecanismo.

etapas individuais do mecanismo.

4. Qual dos mecanismos propostos o mais plausvel? Justifique.

4. Qual dos mecanismos propostos o mais plausvel? Justifique.

Problema 3

A qumica das cores e os compostos


de coordenao

150 pontos

A cor foi a principal caracterstica que chamou a ateno dos primeiros qumicos inorgnicos para os compostos de coordenao. Hoje sabemos que
as cores dos complexos resultam de certos fatores eletrnicos que envolvem
a simetria da molcula e as caractersticas do metal e ligantes envolvidos.
Assim responda os itens abaixo:
1. Desenhe o diagrama de desdobramento de campo para os complexos
K[FeCl4], Ca2[CoCl6] e Na2[PdCl4]. Informe a energia de estabilizao do
campo cristalino para os compostos K[FeCl4] e Ca2[CoCl6].
2. Considerando os compostos K[FeCl4] e Ca2[CoCl6], informe se os mesI

Olimpada Brasileira de Qumica - 2015

95

Processo Seletivo

Fase VI

mos possuem transies permitidas, parcialmente permitidas ou proibidas pelas regras de seleo de transies d-d.
3. Considerando que os seguintes reagentes esto disponveis: trifenilfosfina (PPh3), amnia e [PtCl4]2-, mostre as reaes necessrias para obteno
dos compostos -[PtCl2(NH3)(PPh3)] e
-[PtCl2(NH3)(PPh3)]. Desenhe
as estruturas dos complexos formadas em cada reao.
4. Desenhe a estrutura dos possveis ismeros com geometrias octadricas
obtidos a partir da reao do composto K[FeCl4] com o ligante cyclam
(estrutura abaixo). Informe o nome do mecanismo envolvido na reao.
NH NH
Estrutura do Cyclam =
NH NH

Problema 4

Corroso

150 pontos

O diagrama de Pourbaix um mtodo grfico muito fcil til para prever as


condies de potencial e pH sob as quais um metal pode estar em corroso,
imunidade e passividade. Este diagrama mostra de forma sumria o comportamento previsto para um metal puro imerso em gua pura. Esses diagramas de equilbrio eletroqumicos potencial e pH so extremamente teis
no estudo da corroso e da proteo contra a corroso em meio aquoso. O
diagrama de Pourbaix abaixo representa o comportamento do chumbo em
soluo aquosa.

96

Olimpada Brasileira de Qumica - 2015

Processo Seletivo

Fase VI

Baseado neste diagrama, responda:


1.1.Represente as reaes de equilbrio para as retas 1, 2, 3 e 4, observando

Baseado neste diagrama,


responda:
a condio
de proteo para o chumbo.

1.1. Represente
as reaes que
de equilbrio
paraesteja
as retas
1, 2, 3 e 4,
observando
a condio
para
1.2.Supondo
o chumbo
na seguinte
condio
E-pH:
E = 0,2 de
V eproteo
pH
chumbo. = 1. Mantendo o pH inalterado, cite a espcie ou espcies envolvida(s)

para
conseguir
da condio
condio
de corroso
men1.2. Supondo que
o chumbo
esteja mudar
na seguinte
E-pH:
E = 0,2 V epara
pH =passivao
1. Mantendo oe pH
inalterado, ci
cione
se
essa
espcies
podem
sofrer
desproporcionamento.
a espcie ou espcies envolvida(s) para conseguir mudar da condio de corroso para passivao e mencion
se essaA
espcies
sofrer desproporcionamento.
partir podem
do diagrama
de Pourbaix, diagramas de Latimer podem ser cons-

considere
o diagrama
Latimer
para o chumbo
o estanho
A partir do trudos.
diagramaAssim,
de Pourbaix,
diagramas
de Latimerde
podem
ser construdos.
Assim,econsidere
o diagrama d
em
meio
cido:
Latimer para o chumbo e o estanho em meio cido:
1,698 V

PbO2
SnO2

- 0,088 V

2+

- 0,125 V

2+

- 0,104 V

Pb

Sn

Pb
Sn

Calculedeos
deestado
nE para
cada estado
de oxidao
do chumbo
doDiagrama d
2.1. Calcule2.1.
os valores
nEvalores
para cada
de oxidao
do chumbo
e do estanho,
e construa eum
estanho,
construa
um Diagrama
FrostTenha
comparativo
estes dois
Frost comparativo
parae estes
dois elementos
em meiode
cido.
certeza quepara
o diagrama
construdo este
em meio cido. Tenha certeza que o diagrama construdo escom escala elementos
correta.
teja com escala correta.

2.2. Qual das espcies que foram plotadas o agente oxidante mais forte? Justifique sua resposta.

2.2.Qual das espcies que foram plotadas o agente oxidante mais forte?

2.3. Qual das espcies


plotadas o agente redutor mais forte? Justifique sua resposta.
Justifique
queforam
sua resposta.

estvel
soluo
cida? que
Justifique
suaplotadas
resposta. o agente redutor mais forte?
2.4. O Pb2+2.3.
Qualem
das
espcies
foram

Justifique sua resposta.

2.5. Que produto(s) pode(m) ser formado(s) se PbO2 for misturado com Sn2+ em soluo cida? Escreva a equa
2.4.O Pb2+ estvel em soluo cida? Justifique sua resposta.
balanceada.

com dado
Sn o potenci
2.5.Que
produto(s)
pode(m)
formado(s)
PbO2 for misturado
2.6. Demonstre
que espcie
contendo
chumboser
existiria
se PbOse
com Mg, sendo
2 fosse reagido
2+
em
soluo
cida?
Escreva
a
equao
balanceada.
, E = 2,36 V.
padro de reduo do Mg: Mg + 2e
2+

reagi-em solu
Demonstre
contendo
chumboqumico
existiria
PbO2 efosse
2.7. Use o 2.6.
Diagrama
de Frostque
paraespcie
comparar
o comportamento
dose
chumbo
do estanho
2+
do
com
Mg,
sendo
dado
o
potencial
padro
de
reduo
do
Mg:
Mg
+
aquosa.
_
2e-

Mg, E = 2,36 V.

2.7.Use o Diagrama de Frost para comparar o comportamento qumico do


chumbo e do estanho em soluo aquosa.
Problema 5. Uma lenda sobre Buda (150 pontos)

Uma antiga lenda da ndia conta que Buda, para no se deixar dominar pelo sono, cortou seus prprios clios par
que seus olhos jamais fechassem. Os clios caram no cho e neste local nasceu uma planta chamada Papave
somniferum. A palavra grega papaver s
somniferum
mais tarde se descobriu que esta planta possui uma srie de princpios ativos da famlia dos pios, sendo se
principal componente um opiceo, que foi chamado de morfina em homenagem ao deus grego do sono Morfeu.
I
Olimpada Brasileira de Qumica - 2015
I
97
primeira sntese desta molcula foi desenvolvida em 1952 pelo professor Marshall Gates e tem como etap

Processo Seletivo
Problema 5

Uma lenda sobre Buda

Fase VI

150 pontos

Uma antiga lenda da ndia conta que Buda, para no se deixar dominar pelo
sono, cortou seus prprios clios para que seus olhos jamais fechassem. Os
clios caram no cho e neste local nasceu uma planta chamada
niferum. A palavra grega
significa papoula e, do Latim,
significa eu trago sono. Anos mais tarde se descobriu que esta planta possui uma srie de princpios ativos da famlia dos pios, sendo seu principal
componente um opiceo, que foi chamado de morfina em homenagem ao
deus grego do sono Morfeu. A primeira sntese desta molcula foi desenvolvida em 1952 pelo professor Marshall Gates e tem como etapa principal uma
reao muito famosa hoje em dia, mas que havia sido descoberta h apenas
10 anos.
1. As primeiras etapas da rota sinttica de Gates esto representadas no esquema a seguir. O composto C obtido por uma oxidao e o espectro
de RMN 13C apresenta entre outros sinais, trs sinais em torno de 195,
185 e 165 ppm. Os espectros de RMN 1H dos compostos D e F apresentam dois sinais em torno de 3,7 ppm (3H, simpleto). O composto H
possui um enol em sua estrutura.

Escreva as estruturas moleculares dos compostos A-H.

Escreva as estruturas moleculares dos compostos A-H.

2. Devido ao efeito analgsico eficaz da morfina, indispensvel no tratamento de dores associada com o cncer e
tambm devido ao interessante esqueleto pentacclico, incluindo um centro estereognico de carbono
quaternrio, esta substncia estimulou extensos esforos sintticos, gerando inmeros estudos e snteses totais
reportadas na literatura. Kenji Uchida e colaboradores publicaram mais recentemente uma rota sinttica
I
Olimpada
de Qumica
- 2015 representada
I
moderna
paraBrasileira
produo
da Morfina,
no esquema a seguir.
98 e eficiente
i.MOMCl, piridina

Processo Seletivo

Fase VI

2. Devido ao efeito analgsico eficaz da morfina, indispensvel no tratamento de dores associada com o cncer e tambm devido ao interesEscreva as estruturas
compostos A-H.
sante moleculares
esqueletodos
pentacclico,
incluindo um centro estereognico de carbono
quaternrio,
esta
substncia
estimulou
extensosdeesforos
sintticos,
2. Devido ao efeito analgsico eficaz da morfina, indispensvel
no tratamento
dores associada
com o cncer e
gerando
inmeros
estudos
e
snteses
totais
reportadas
na
literatura.
Kenji
tambm devido ao interessante esqueleto pentacclico, incluindo um centro estereognico
de carbono
e colaboradores
publicaram
recentemente
umaestudos
rota sintquaternrio,Uchida
esta substncia
estimulou extensos
esforos mais
sintticos,
gerando inmeros
e snteses totais
modernaKenji
e eficiente
produopublicaram
da Morfina,
no esreportadas tica
na literatura.
Uchida epara
colaboradores
maisrepresentada
recentemente uma
rota sinttica
quema para
a seguir.
moderna e eficiente
produo da Morfina, representada no esquema a seguir.
i.MOMCl, piridina
ii. PTSA, MeOH (excesso)
iii. n-BuLi, I2

MeO
CHO

HO

AcOH
THF-H2O

MeOCH2PPh3Cl

NaBH4
MeOH/
CH2Cl2

X
i. TCDI, DMAP
ClCH2CH2Cl

N
O

L
M
C
H
IO
C24H39IO6Si Pb2(dba3), P(2-furil)3 11 15 4

MeCN

O
i. Dess-Martin
periodinano
CH2Cl2

P
ii. Et3B, n-Bu3SnH C19H21NO5 ii. LiAlH4, THF

HCl
MeOH

C(CH 3)3
Si
O
MeO

Aq H2O2
NaOH
MeCN

NaHMDS-THF

THF (2 etapas)

HO
BBr

Q
C18H21NO3 CHCl2 HO

NMe

Morfina

TCDI
S
N

O composto L apresenta um sinal com amplitude negativa no espectro de RMN 13C obtido pela tcnica DEPT 135.
O composto L apresenta um sinal com amplitude negativa no espectro de
A reao de formao do composto N a partir de X gera dois novos centros estereognicos, ambos com
RMN 13C obtido pela tcnica DEPT 135. A reao de formao do composto
configurao S, e na etapa seguinte ocorre a formao de um novo centro com configurao S, gerando o composto
N a partir de X gera dois novos centros estereognicos, ambos com confiO.

gurao S, e na etapa seguinte ocorre a formao de um novo centro com


configurao S, gerando o composto O.

Escreva as estruturas moleculares dos compostos I-Q, com a estereoqumica


definida para os compostos N-Q.

Olimpada Brasileira de Qumica - 2015

99

Processo Seletivo

Problema 6

Fase VI

Alm da controvrsia da legalizao


da maconha: sntese total do THC

150 pontos

Os tetraidrocanabinis (THC) compreendem uma classe de substncias conhecidas por suas atividades biolgicas, especialmente quelas relacionadas a efeitos sobre o sistema nervoso central. Dentre as fontes naturais de
derivados
do THC destaca-se
a planta
herbcea sntese total do ,THC
da famlia
roblema 6.
Alm da controvrsia
da legalizao
da maconha:
(150 pontos)
Cannabiaceae, popularmente conhecida como maconha. O uso da maconha
s tetraidrocanabinis
(THC)
compreendem
uma
classe desetores
substncias
conhecidas por
atividades
de grande
controvrsia
em
diversos
da sociedade,
emsuas
especial
no biolgicas,
pecialmente
quelas
relacionadas
efeitos
sobre
sistema nervoso central.
Dentre efeito
as fontes
naturais de derivados
que
se trata
ao conflaito
entre
seuo popularmente
conhecido
psicoativo
e seu emergente
emprego
no sativa,
combate
a patologias.
Em ambos
os casos,
THC destaca-se
a planta herbcea
Cannabis
da famlia
Cannabiaceae,
popularmente
conhecida como
provm
dos derivados
presentes
na planta.
Para no
osque se trata
aconha. O os
usoefeitos
da maconha
de grande
controvrsiacanabinlicos
em diversos setores
da sociedade,
em especial
cientistas,
em especial
aos adeptos
da qumica
discutirno
sobre
THC
conflito entre
seu popularmente
conhecido
efeito psicoativo
e seu orgnica,
emergente emprego
combate
a patologias.
secasos,
torna os
uma
boaprovm
oportunidade
para discutir
tambm
sobre
orgnica.
m ambos os
efeitos
dos derivados
canabinlicos
presentes
nasntese
planta. Para
os cientistas, em

creva as estruturas moleculares dos compostos I-Q, com a estereoqumica definida para os compostos N-Q.

pecial aos adeptos da qumica orgnica, discutir sobre THC se torna uma boa oportunidade para discutir tambm
Abaixo so apresentadas trs rotas sintticas para obteno de derivados
bre snteseTHC,
orgnica.
aqui intitulados THC-1, THC-2 e THC-3.

baixo so apresentadas trs rotas sintticas para obteno de derivados THC, aqui intitulados THC-1, THC-2 e
1. Na formao do THC-1 o composto E um substrato apto a sofrer adio
HC-3.

1,4-conjugada (Michael), e pode ser obtido a partir do composto biccli-

co THC-1
D. D oascomposto
estruturas
dosacompostos
de forma(Michael),
a com- e pode ser
Na formao do
E moleculares
um substrato apto
sofrer adio A-E,
1,4-conjugada
pletar
corretamente
a
rota
sinttica.
obtido a partir do composto bicclico D. D as estruturas moleculares dos compostos A-E, de forma a completar
corretamente a rota sinttica.
COOH

D
C10H16
i) KMnO4, H2SO4
ii) Ph2Se2, SeO2
iii) H2O2

MeO

COOH

i) SOCl2, C5H5N, CH2Cl2


ii) BuLi, THF

H2NNH2, HO-

BBr3, H2O

AlCl3,
CH2Cl2/MeNO2

OMe

H
O
O

H
HO

C5H11

THC-1

THC-2 obtido conforme descrito no esquema reacional a seguir. O composto F um composto , -insaturado.
O composto J enantiomericamente puro e possui dois centros estereognicos, ambos com configurao
absoluta S. O composto I apresenta uma banda de absoro intensa em 1680 cm-1 no espectro de Infravermelho
(IV). D as estruturas
moleculares dos reagentes
e produtos dos compostos F-N, de forma a completar
I
Olimpada Brasileira de Qumica - 2015
I
100
corretamente as rotas sintticas, considerando os fatores estereoqumicos.

Processo Seletivo

Fase VI

2. THC-2 obtido conforme descrito no esquema reacional a seguir. O


composto F um composto ,-insaturado. O composto J enantiomericamente puro e possui dois centros estereognicos, ambos com configurao absoluta S. O composto I apresenta uma banda de absoro
intensa em 1680 cm-1 no espectro de Infravermelho (IV). D as estruturas
moleculares dos reagentes e produtos dos compostos F-N, de forma a
completar corretamente as rotas sintticas, considerando os fatores estereoqumicos.

I2, K2CO3, DMAP,


THF/H2O

Pd(PPh3)2, H, LiCl,
Na2CO3, H2O, DME

OMe

C8H10O3

H2 (1 atm), Pd/C, EtOH


O

I2, K2CO3, DMAP,


THF/H2O

Pd(PPh3)2, H, LiCl,
Na2CO3, H2O, DME

reduo

OMe

NaH, DMF

C5H11

OMe

H2 (1 atm), Pd/C, EtOH

OH

C8H10O3

OF

(COCl2)2, DMSO,O F
Et3N, CH2Cl2

C5H11

J
reduo

NaH,
DMF
Et2NCH2NaH,
CH2SH,
L
DMF

M
C21H32O3

OH

F OH

OMe

C5H11

(COCl2)2, DMSO,
Et3N, CH2Cl2

F
NaH,
ZnCl2, HCl, AcOH OH
Et2NCH2CH2SH,
N
DMF

CH3CH2C(CH3)2OK, C6H6

ZnCl2, HCl, AcOH

CH3CH2C(CH3)2OK, C6H6

C5H11

OH

OH

C5H11

THC-2

H32O3
n
3. Para a formao
THC-3,
no esto THC-3,
sendo considerados
estereoqumicos.
Determine a
O
C5H11
3. ParadoaC21derivado
formao
do derivado
no esto aspectos
sendo
considerados
asTHC-2
estrutura qumica
do
composto
O
que
reage
com
P
atravs
de
um
processo
pericclico.
O
composto
pectos estereoqumicos. Determine a estrutura qumica do composto OP apresenta
um plano de simetria.
Escreva
a estrutura
molecular
dos compostos
O e P. (MOMCl
= CH3OCH
2Cl)
que reage
com
P atravs
de um processo
pericclico.
O composto
P apre-

3. Para a formao do derivado THC-3, no esto sendo considerados aspectos estereoqumicos. Determine a
sentado
um
plano de
simetria.
Escreva
a estrutura
molecular
dosOcompostos
estrutura qumica
composto
que
reage com
P atravs
de um processo
pericclico.
composto P apresenta
i) MOMCl,
K2CO3,O
DMF
Cl)
Osimetria.
eii)P.BuLi,
(MOMCl
=estrutura
CH3OCH
TMDA, DMF
um plano de
Escreva
a
molecular
dos
compostos
O
e
P.
(MOMCl
=
CH
3OCH2Cl)
2
OH
iii) CH3COCH3, NaOH(aq)

OH

i) MOMCl, K2CO3, DMF


ii) BuLi, TMDA, DMF
iii) CH3COCH3, NaOH(aq)

O
O

P, H2O, 150 atm

P, H2O, 150 atm

COCH3
OCH2OCH3

O
THC-3

COCH3
OCH2OCH3

O
THC-3

Problema 7. No limite entre a vida e a morte (100 pontos)

Numa
aluso 7.personagem
principal
do clssico
O mdico
e o monstro (Strange Case of Dr Jekyll and Mr Hyde
Problema
No limite entre
a vida
e a morte
(100 pontos)
I
Olimpada
Brasileira opostas
de Qumica entre
- 2015 o Ibem 101
de R. L. Stevenson), que apresentava uma personalidade ambgua
em direes
e o mal, alguns
Numa alusodependendo
personagemdeprincipal
do clssico
O mdico
o monstro
(Strange
Caseproduzir
of Dr Jekyll
and teraputicos
Mr Hyde,
medicamentos,
uma discreta
diferena
nas esuas
dosagens,
podem
efeitos

3. Para a formao do derivado THC-3, no esto sendo considerados aspectos estereoqumicos. Determine a
estrutura qumica do composto O que reage com P atravs de um processo pericclico. O composto P apresenta
um plano de simetria. Escreva a estrutura molecular dos compostos O e P. (MOMCl = CH3OCH2Cl)

OH

Processo Seletivo

i) MOMCl, K2CO3, DMF


ii) BuLi, TMDA, DMF
iii) CH3COCH3, NaOH(aq)

Problema 7

Fase VI

P, H2O, 150 atm


COCH3
OCH2OCH3

No limite entre a vida e a morte

100 pontos
THC-3

Numa aluso personagem principal do clssico


, de R. L. Stevenson), que apresentava uma
personalidade ambgua em direes opostas entre o bem e o mal, alguns
Problema
7. No limite entre
a vida e a morte
(100 pontos)
medicamentos,
dependendo
de uma
discreta diferena nas suas dosagens,
podem
produzir
efeitos
teraputicos
desejados
ou(Strange
provocar
po-Mr Hyde,
Numa aluso personagem principal do clssico O mdico e o monstro
Casetoxicidade,
of Dr Jekyll and
dendo
levar

morte.
Os
barbitricos,
usados
como
hipnticos,
sedativos,
de R. L. Stevenson), que apresentava uma personalidade ambgua em direes opostas entre o bem e o mal, alguns
anticonvulsivantes
e anestsicos,
constituem
uma dessas
frmamedicamentos,
dependendo de uma
discreta diferena
nas suas dosagens,
podemclasses
produzirde
efeitos
teraputicos
cos
poderosos
e
perigosos
ao
mesmo
tempo.
Tais
frmacos
so
derivados
desejados ou provocar toxicidade, podendo levar morte. Os barbitricos, usados como hipnticos, sedativos,
do cidoe barbitrico,
descoberto
em 1864
qumico
alemoe Adolf
von
anticonvulsivantes
anestsicos, constituem
uma dessas
classespelo
de frmacos
poderosos
perigosos
ao mesmo
Baeyer
e resultante
da condensao
do cido
malnico
composto
A1.
tempo. Tais
frmacos
so derivados
do cido barbitrico,
descoberto
em 1864com
pelooqumico
alemo
Adolf von
foi odaqumico
francs
Edouard
em 1879,
quem
pela Edouard
Baeyer eMas
resultante
condensao
do cido
malnicoGrimaux,
com o composto
A1. Mas
foi o realizou,
qumico francs
Grimaux,primeira
em 1879, quem
pela primeira vez, a sua sntese.
vez, arealizou,
sua sntese.

1.1. Escreva
estrutura adoestrutura
composto A1.
1.1.aEscreva
do composto A1.
1.2. Que1.2.
produto
combinaoda
de combinao
A1 com o cido de
malnico
promove
a anlise
do cloreto de
fosforila? a
Quedaproduto
A1 com
o cido
malnico
promove

anlise do cloreto de fosforila?

1.3. Desenhe as formas tautomricas e de ressonncia do cido barbitrico.

1.3.Desenhe as formas tautomricas e de ressonncia do cido barbitrico.


Aps 1902, vrios barbitricos foram sintetizados e, dentre eles, o fenobarbital, lanado pela Bayer em 1912, passou a ser, at hoje, o anticonvulsivante
mais usado em todo o mundo. Cerca de 75% do fenobarbital administrado
Aps 1902, vrios barbitricos foram sintetizados e, dentre eles, o fenobarbital, lanado pela Bayer em 1912,
no corpo sofre biotransformao no fgado pela enzima citocromo P450 e
passou a ser, at hoje, o anticonvulsivante mais usado em todo o mundo. Cerca de 75% do fenobarbital
excretado pelo rim, na forma conjugada ao cido glicurnico, enquanto a
administrado no corpo sofre biotransformao no fgado pela enzima citocromo P450 e excretado pelo rim, na
parte
excretada
sofrer
alterao
qumica.
a reao
deObserve a
formamenor
conjugada
ao cido
glicurnico, sem
enquanto
a menor
parte excretada
semObserve
sofrer alterao
qumica.
modifi
cao
qumica
do
fenobarbital
ilustrada
na
fi
gura
abaixo:
reao de modificao qumica do fenobarbital ilustrada na figura abaixo:

2.1. Explique por


que
essa reao de hidroxilao no
corresponde forma de ao da citocromo P450.
I
Olimpada Brasileira de Qumica - 2015
I

102

2.2. Refaa o esquema para que o mesmo reflita a ao correta da citocromo P450 sobre o fenobarbital.

Processo Seletivo

Fase VI

2.1.Explique por que essa reao de hidroxilao no corresponde forma


de ao da citocromo P450.
2.2.Refaa o esquema para que o mesmo reflita a ao correta da citocromo
P450 sobre o fenobarbital.
2.3.Justifique a importncia da reao catalisada pela citocromo P450 para
os passos seguintes de metabolizao do fenobarbital.
Alm de sofrer biotransformao pelo citocromo P450, o fenobarbital tambm apresenta uma potente ao indutora da produo dessa enzima pelo
fgado, interferindo, assim, na metabolizao de outros medicamentos. Um
exemplo disso a associao do fenobarbital com a varfarina, um anticoagulante oral usado na preveno e tratamento de indivduos com risco de trombose. Alguns casos fatais j foram relatados em decorrncia da interrupo
do uso do fenobarbital, sem o devido ajuste da dosagem de varfarina, aps
um longo perodo de tratamento simultneo.
3. Com base na capacidade de induzir o aumento dos nveis da citocromo
P450 pelo fenobarbital, deduza como a interao entre os dois medicamentos levou s mortes quando o tratamento pelo fenobarbital foi descontinuado.

"O dinheiro faz homens ricos;


o conhecimento faz homens sbios
e a humildade faz homens grandes."
I

Olimpada Brasileira de Qumica - 2015

103

47th IChO

Exame Terico

47a Olimpada
Internacional
de Qumica
25 de julho de 2015
Baku - Azerbaijo

Constantes
fsicas,
frmulas
and
equaes
Constantes
fsicas,
and
Constantes
fsicas,unidades,
unidades,
frmulas
andequaes
equaes
Constantes
fsicas,
unidades,
frmulas
and
equaes
Constantes
fsicas, unidades,
unidades,
frmulas frmulas
and
equaes
Constantes
fsicas, unidades,
and equaes
Constantes
fsicas, unidades,
frmulas frmulas
and equaes

Constantes
fsicas, unidades,
frmulas
1
1
1
1
1
1and
1 equaes
1
1
1
1
1
Universal
dos
Constante
ConstanteUniversal
Universaldos
dosR
Constante
Universal
dos
==8,
8,8,
ConstanteConstante
Universal
dos
R=
= 8,
8, RRR=
1
1
1
1
gases
gases
gasesdos gases
gases
Constante
Universal
dosR = 8, R = 8,
gases
Constante
Universal
dos
Constante
Universal
5
gases
gases
5 Pa = 750555mmHg
pp =
10
Presso
padro
Presso
Pressopadro
padro
10
Pa
p=
bar====750
10 mmHg
Pa===750
750mmHg
mmHg
Presso
padro
Presso
padro
==1151bar
bar
10
Pa
750
mmHg
Presso
padro
= 11 bar
barpp=
=
10
Pa
5
5 bar
5 ==750
Presso
padro
Presso atmosfrica
padro
p
=
1
10
Pa
=
750
mmHg
5
p
=
1
bar
=
10
Pa
mmHg
5
5
Presso
1
atm
=
1,013
10
Pa
=
760
mmHg
5
Presso
Pressoatmosfrica
atmosfrica 1 atm = 1,013
111atm
===51,013
10
Pa
atm10
1,013
10 mmHg
Pa===760
760mmHg
mmHg
Presso
atmosfrica
Presso
atmosfrica
Presso
atmosfrica
atm
1,013
10
Pa
760
mmHg
Pa = 760
51,013 105 Pa = 760 mmHg
Presso
atmosfrica
Presso
atmosfrica
1
atm
=
1
atm
=
1,013
10
Pa
=
760
mmHg
Zero
da escala
escala
Celsius
273,15 K
K 273,15
Zero
da
Zero
da
Celsius
Zero
daescala
escalaCelsius
Celsius 273,15
273,15K
Zero
da
escala
Celsius
273,15
KK
Zero
da escala
Celsius

Zero Celsius
da escala Celsius 273,15 K 273,15 K
Zero da escala
1 R
R //C
CVV
1
111RR/R/CC/VCV
Processo
adiabtico
reversvel
para
um
gs
ideal
V ==const
Processo
adiabtico
reversvel
para
um
Processo
adiabtico
reversvel
para
umgs
gsideal
ideal
pV
const
Processo
adiabtico
reversvel
para
um
gs
ideal
pV
const
ProcessoProcesso
adiabtico
reversvel
para
um um
gs
ideal
pV
= const
pV 1 R /CV =
=pV
const
adiabtico
reversvel
para
gs
ideal
1 R /CV
1 R /CV
Processo
adiabtico
reversvel
para
um
gs
ideal
Processo
adiabtico
reversvel
para
um
gs
ideal
pV
=pV
const = const
Trabalho
realizado
em
um
ideal
em
um
Trabalho
realizado
em
um
gs
ideal
Trabalho
realizado
em
um
gsem
ideal
emum
um W = nCV (T2 T1)
realizado
emgs
um
gs
ideal
umem
Trabalho
realizado
em
um
gs
ideal
em
um
TrabalhoTrabalho
realizado
em
um
gs
ideal
em
um
V
22=
11)VVV(T
W
W
(T222TTT11)1))
W
==nC
nC
(T
W
=
nC
TnC
V (T
processo
adiabtico
processo
adiabtico
processo
adiabtico
processo
adiabtico
adiabtico
Trabalho
realizado
um em
gsum
ideal em um
processo
adiabtico
Trabalhoprocesso
realizado
em
um gsem
ideal
W2 = nC
W = nCV (T
T1)V (T2 T1)
processo
adiabtico
processo
adiabtico
Dependncia
da
interna
com
aacom acom
Dependncia
da
energia
interna
Dependncia
da energia
interna
Dependncia
da
energia
interna
comaaa
Dependncia
da
energia
interna
com
Dependncia
da energia
energia
interna
com
2) = U(T
1)2)+
VU(T
U(T
))VVV(T
2
1
V
111C
U(T
U(T
=U(T
+C
(T222TTT11)1))
2
U(T
)
U(T
C
(T
U(T2) = U(T1)2+)==C
C
(T22211)1))++T
T
V (T
temperatura
temperatura
temperatura
temperatura
temperatura
Dependncia
energia
interna
com a
temperatura
Dependncia
da energiada
interna
com
a
U(T
)
=
U(T
)
+
C
(T
2
1
V
2 T1)
U(T2) = U(T
)
+
C
(T

T
)
1
V
2
1
temperatura
temperatura
Relao
entre
capacidades
calorficas
molares
entre
capacidades
calorficas
molaresmolares
Relao
entre
capacidades
calorficas
Relao
entre
capacidades
calorficas
molares
Relao
entre
capacidades
calorficas
molares
Relao Relao
entre
capacidades
calorficas
molares
C
=
C
+
R
CC
==C
CCVVV+
++RRR
Cppp = CVVV +C
Rppp=
isobrica
eeisobrica
isocrica
um
gs
ideal
eeepara
isocrica
para
um
gs
isobrica
isocrica
para
um
gsideal
ideal
e isocrica
para
um
gs
ideal
isobrica
isocrica
para
um
gs
ideal
Relao
entre
capacidades
calorficas
molares
isobrica
isocrica
para
um
gs
ideal
Relao isobrica
entre
capacidades
calorficas
molares
Cp = CV +CRp = CV + R
isobrica
e para
isocrica
para
um gs ideal
isobricaEnergia
isocrica
um gs
ideal
Energia
deeEnergia
Gibbs
G=
=H
H TS
TS
de de
Gibbs
Gibbs
G
Energia
de
Gibbs
G
Energia
de
Gibbs
TS
Energia
de
Gibbs
GG===H
HHTS
TS
deaGibbs
G = H TS
Energia Relao
deEnergia
Gibbs
G = H TS
entre
constante de equilbrio e a
G
Relao
entre
aa constante
de
equilbrio
eeequilbrio
aa
GG
Relao
entre
de
eeeaaa K = exp K =G
G exp
Relao
entreaaaconstante
constante
deequilbrio
Relao
entre
constante
de
equilbrio
Relao energia
entre
constante
de
equilbrio
exp
depadro
Gibbs
padro
K = exp KK ==exp
energia
de
Gibbs
energia
de
Gibbs
padro
G
energia
de
Gibbs
padro
RT
G
RT
energia
de
Gibbs
padro
RT
Relao
entre
a
constante
de
equilbrio
e
a
energia
de
Gibbs
padro
Relao entre a constante de equilbrio e a
RT
K = exp K =RTexp
de Gibbs padro
energia deenergia
Gibbs padro
RT
RT
a
aaaprod
prod
aprod
Dependncia da energia de Gibbs de uma reao
prod
G
=
G
,, ln
=== ln
RT
GGRT
GG prodRT
RT
lnln prod ,, ,
G= G G
RT
lnG
a
Dependncia
da
energia
de
Gibbs
de
uma
reao
com
a
concentrao
ou
com
a
presso
a
reag
a
prod
Dependncia
Gibbs
de
Dependncia
da
energia
deuma
Gibbs
deuma
umareao
reao
reag
Dependncia
da
energia
de
Gibbs
de
uma
reao
reag ,
Dependncia
da energiada
deenergia
Gibbs de
reao
reag
reag
= lnGaprod
RT
G = G GRT
, ln areag
com
aa concentrao
ou
aa ou
presso
com
aaaconcentrao
aaapresso
com
concentrao
oucom
com
presso
a
a
com
concentrao
ou
com
presso
Dependncia
da
de uma
Gibbs
de uma reao
com
concentrao
ou com
com
presso
Dependncia
da
energia
deenergia
Gibbs
de
reao
reag
reag
aa =
c
/
(1
mol/L)
para
as
substncias
aaa===ccc///(1
para
(1mol/L)
mol/L)
paraas
assubstncias
substncias
(1
mol/L)
para
as
substncias
= c / (1 mol/L)
para
as substncias
com a concentrao
ou com a presso
com a concentrao
ou com a presso
gases
em
=
// mol/L)
(1
em
abar)
bar)
para
=pppara
p///(1
(1gases
bar)
paragases
gases
em
soluo,
aa==substncias
(1
bar)
para
gases
aem
=aasoluo,
csoluo,
as
substncias
em
soluo,
=/ pp(1
(1 as
bar)
para
a = soluo,
c / (1 mol/L)
para
emGaGsoluo,
a = ppara
/ (1 gases
bar) para gases
emGsoluo,
= p / (1 bar)
Variao
da
energia
de
Gibbs
por
unidade
de
Variao
da
de
por
Variao
da
energia
deGibbs
Gibbs
por
unidadede
de
Syst
Variao
da
energia
de
Gibbs
por
unidade
de
GSyst
VariaoVariao
da
energia
deenergia
Gibbs
por
unidade
deunidade
GSyst
Syst
da energia
de Gibbs
por
unidade
Syst
Syst
G
G
rrG
r1 G
G22r2r22r2
1
1
2
2
G
G
G
r
G
1r
1
2G
211rr
1
1
tempo
e
volume
para
o
sistema
com
duas
reaes
tempo
e
volume
para
o
sistema
com
duas
reaes
1
tempo
e
volume
para
o
sistema
com
duas
reaes
1
1
2
2
tempo
ee volume
parapor
ocom
sistema
com
duas reaes
Variao
da
Gibbs
por
unidade
de GSyst
tempo
e de
volume
para
oenergia
sistema
duas
reaes
GtSyst
Variao
da
energia
de
Gibbs
unidade
de
tt
t
tempo
volume
para
ode
sistema
duas
qumicas
11qumicas
ee 22 com
rr111sistema
ee duas
rr222 com
G2 r2
G1r1t
G2G
r21r1
11velocidades
com
velocidades
rr1r11eeerduas
qumicas
1eeoe22sistema
velocidades
qumicas
velocidades
tempo
e volume
para
ocom
com
qumicas
com
velocidades
tempo e reaes
volume
para
reaes
qumicas
12com
ecom
2 com
velocidades
r1 er2r2r22 reaest
t
e 2 com velocidades
r1 e r2
qumicas 1qumicas
e 2 com 1velocidades
r1 e r2

104

Olimpada Brasileira de Qumica - 2015

47th IChO

Exame Terico

Os novos e os bem-esquecidos
sistemas de refrigerao velhos

Problema 1

Questo

1.1

1.2

1.3

2.1

2.2

2.3

Pontuao

3
3

08 pontos
Total

4
4.1

4.2

4.3

4.4

10

33

O problema para escolher o lquido refrigerante para sistemas de refrigerao e ar condicionado atraiu os cientistas e engenheiros durante o ltimo
sculo. Durante este perodo foram desenvolvidos quatro tipos diferentes de
lquidos refrigerantes. Amnia, considerado como o sistema de refrigerao
de primeira gerao, e que foi usado nos sistemas de refrigerao mais antigos. Foi mais tarde substitudos por clorofluorcarbonos (CFCs) derivados
do metano e etano onde os tomos de hidrognio foram substitudos por
tomos de flor e cloro.
Em Baku, na fbrica Bakkonditsioner foram
produzidos os primeiros aparelhos de ar
condicionado domstico BK-1500 da Unio
Sovitica. O clorodifluorometano (CHF2Cl)
foi usado como uma segunda gerao de
lquidos refrigerantes. Neste problema so
comparados vrios lquidos refrigerante em
termos termodinmicos.

Primeiro ar condicionado produzido


na fbrica de Baku e instalado numa
loja na cidade velha (Icheri Sheher)

Propriedades termodinmicas de vrios lquidos refrigerante


Lquido
refrigerante

Gerao

Hvap / kJmol1
(a 280 K)

Cv (gas) /
JK1mol1

NH3

21,3

26,7

CHF2Cl

20,0

48,8

CF3CH2F

22,1

79

CF3CF=CH2

19,1

120

Olimpada Brasileira de Qumica - 2015

105

47th IChO

Exame Terico

Considere um modelo de ciclo de refrigerao consistindo de 4 etapas esquematicamente mostrado abaixo onde as coordenadas (p) representam
presso e (U) energia interna.

p
3

p2

2 (T2)

liquid

p1

liquid + gas
gas

0 (T1)

1 (T1)

U
Diagrama 1 (Nota As linhas a tracejado indicam as linhas de transio de fase)
Diagrama 1 (Nota As linhas a tracejado indicam as linhas de transio de fase)

Durante a primeira etapa do ciclo (linha 0-1 do diagrama 1), o lquido refriDurante
primeira etapa
do ciclo
(linha
0-1 do diagrama
1), o plquido
refrigerante levado

gerante
alevado
ebulio
presso
constante
e temperatura
T1 (tem1
T1 (temperatura Nesta
de ebulio)
at aevaporar
ebuliode ebulio)
presso constante
p1 e temperatura
peratura
at evaporar
completamente.
etapa
unidade
completamente.
Nesta etapa
a unidade
dos objetos
a rodeiam.etapa,
Na
refrigerante
absorve
o calor
dosrefrigerante
objetosabsorve
que ao calor
rodeiam.
Naque
segunda
segundarefrigerante
etapa, o lquido sofre
refrigerante
uma compresso
adiabtica reversvel
e aquece
at
o lquido
umasofre
compresso
adiabtica
reversvel
e aquece
Em seguida,
o lquido
refrigerante comprimido
em
atingir a temperatura
T2 (linha 1-2).
at atingir
a temperatura
T2 (linha
1-2).
Em seguida,
o lquido resfriado
refrigerante
um condensador
a presso constantes
p2 (linha
2-3) e regressaaaopresso
estado inicial
(linha 3-0). p (linha
comprimido
resfriado
em um
condensador
constantes
2
que oao
cicloestado
envolve 1inicial
mol de (linha
lquido refrigerante,
que est inicialmente no estado lquido
2-3) Considere
e regressa
3-0).
T2

(ponto 0), T1

lquido refrigerante se

Considere
envolve
1 mol determodinmicas
lquido refrigerante,
que est esto
inicialcomporta que
como o
umciclo
gs ideal.
As caractersticas
dos lquidos refrigerantes
indicadas
na tabela da
pgina anterior.
mente
no estado
lquido
(ponto 0), T1 = 280 , e que a T2 = 380 e que o
vapor de qualquer lquido refrigerante se comporta como um gs ideal. As
caractersticas termodinmicas dos lquidos refrigerantes esto indicadas na
1.1.da
Parapgina
cada lquido
refrigerante, amnia e clorodifluoroetano, calcule a quantidade de calor Q
tabela
anterior.
absorvida pela unidade refrigerante durante a troca de calor (linha 0-1) e o trabalho W necessrio

1.1.para
Para
cada lquido
amnia
e clorodifluoroetano, calcule a
a compresso
adiabticarefrigerante,
do seu vapor (linha
1-2).
quantidade
Clculos de calor Q absorvida pela unidade refrigerante durante a troca
de calor
(linha 0-1) e o trabalho
W necessrio para
Amnia
Q=
W =a compresso adiabtica
do seu vapor (linha 1-2).
Clorodifluorometano

Q=

Clculos

W=

Amnia
Q = durante a etapa da compresso
W =adiabtica?
1.2. Que quantidade(s) se mantm constante
Assinale a sua resposta com um crculo.

Clorodifluorometano
U

106

Q=

W=

Olimpada Brasileira de Qumica - 2015

47th IChO

Exame Terico

1.2. Que quantidade(s) se mantm constante durante a etapa da compresso adiabtica?


Assinale a sua resposta com um crculo.
U

Para comparar a eficincia energtica de ciclos de refrigerao com diferentes parmetros e lquidos refrigerantes usa-se o coeficiente de eficincia/performance (COP), que definido como a razo entre o calor removido de um
sistema de refrigerao e o trabalho do compressor: COP = Q/W.
1.3. Calcule os valores COP do ciclo apresentado para a amnia e clorodifluorometano.
Clculos
Amnia

COP =

Clorodifluorometano

COP =

2.1. Porque a amnia foi substituda pelos CFCs nas unidades de refrigerao domsticas? (Escolha apenas uma opo)
a) para aumentar a eficincia de energia do ciclo refrigerante
b) por que nas mesmas condies a densidade do amnia inferior
do ar.
c) por razes de segurana
Resposta

A procura de substitutos dos CFCs como lquido refrigerante comeou quando se demonstrou que a utilizao destes provocava danos irreparveis na
camada de oznio. Surgiu assim, a terceira gerao de lquidos refrigerante
amigos do oznio. Os mais representativos so os fluoroalcanos.
I

Olimpada Brasileira de Qumica - 2015

107

47th IChO

Exame Terico

2.2. Qual a causa da destruio da camada de oznio pelos CFCs? (Escolha


apenas uma opo)
a) a molcula de oznio adiciona-se facilmente a uma ligao CF
b) a ligao CF facilmente quebrada por radiao formando radicais livres
c) a molcula de oznio adiciona-se facilmente a uma CCl
d) a ligao CCl facilmente quebrada por radiao formando radicais livres
Resposta

Contudo, em 1997 com o protocolo de Kyoto, os fluoroalcanos foram tambm banidos em consequncia da sua acumulao na atmosfera e capacidade de absorverem a radiao infravermelha, o que causa aumento da
temperatura atmosfrica (efeito estufa). Os lquidos refrigerantes da quarta
gerao, como por exemplo o 2,3,3,3-tetrafluoropropeno (CF3CF=CH2) foram sugeridos e comearam a ser utilizados.
2.3. Por que que este composto contribui menos para o efeito estufa do
que os fluoroalcanos. (Escolha apenas uma opo)
a) mais reativo e de fcil decomposio
b) reage facilmente com o oznio
c) mais solvel em gua
Resposta

3. Calcule os valores de COP no ciclo apresentado acima para os dois lquidos refrigerante de terceira e quarta geraes CF3CH2F e CF3CF=CH2. E
indique se a eficincia energtica aumentou em comparao com o CHF2Cl?
Escolha Sim ou No.

108

Olimpada Brasileira de Qumica - 2015

c) mais solvel em gua


Resposta

47th IChO

Exame Terico

3. Calcule os valores de COP no ciclo apresentado acima para os dois lquidos refrigerante de
terceira e quarta geraes CF3CH2F e CF3CF=CH2. E indique se a eficincia energtica aumentou
em comparao com o CHF2Cl? Escolha Sim ou No.

Clculos
Clculos

CF3CH2F
CF3CH2F

CF3CF=CH2
CF3CF=CH2

COP =
COP =

Sim
Sim

COP =

COP =

No
No

Sim

Sim

No

No

Ao contrrio das aplicaes domsticas, nos sistemas de refrigerao industrial


usado
a amnia.
Esta
contribui
para oindustrial
efeito ainda
estufa
e nem
Ao ainda
contrriodas
aplicaes
domsticas,
nosno
sistemas
de refrigerao
usado
a
destri
camada
de oznio.
Estas
industriais
muito
amnia. aEsta
no contribui
para o efeito
estufaunidades
e nem destri
a camada deso
oznio.
Estas grandes
unidades
e industriais
caras. Antes
de grandes
sereme construdas,
devem
ser planejadas
tendo em
so muito
caras. Antes de serem
construdas,
devem ser planejadas
tendoconem
taconta
diferentes
parmetros.
Em
sistemas
reais,
parte
do
lquido
refrigerante
diferentes parmetros. Em sistemas reais, parte do lquido refrigerante liberado, no incio do
liberado,
nosob
incio
dodeaquecimento,
sob(ponto
a forma
de vapor
o (ponto
ambiente
aquecimento,
a forma
vapor para o ambiente
0 no diagrama
2) e,para
no final
1),
(ponto
0
no
diagrama
2)
e,
no
final
(ponto
1),

sempre
aquecido
acima da
sempre aquecido acima da sua temperatura de ebulio.
sua temperatura de ebulio.

p
3 (T3)

p2
liquid

p1

2 (T2)
gas

liquid +
gas

1 (T1)

0 (T0)

U
Diagrama
2(Nota:2As
linhas
tracejadas
linhas
de transio
Diagrama
(Nota:
As linhas
tracejadasindicam
indicam asas
linhas
de transio
de fase) de fase)

Considere que o ciclo envolve 1 mol de amnia. As suas propriedades termoConsidere que
o ciclo envolvede
1 mol
de amnia. AsH
suas propriedades
termodinmicas
so:entalpia
1
dinmicas
so:entalpia
vaporizao
vap = 23,35 kJmol a Tvap = 239,8
1
=
23,35
kJmol
a
T
ra
de
ebulio

presso
de da
1
H
vap
vap
(temperatura de
ebulio presso
de 1 bar). A capacidade calorfica
1
1
bar).
A
capacidade
calorfica
da
fase
lquida

C
=
77
JK
mol
,
e
da
fase
gasosa

C
=
v(gas)
fase lquida Cv(liq) = 77 JK1mol1, e dav(liq)
fase gasosa Cv(gas) = 26,7 JK1mol1.
6
Consiedere que as capacidades calorficas
so independentes da temperatura e o vapor comporta-se como um gs ideal.A relao entre a temperatura
e a presso de vapor da amnia pode ser descrita pela equao emprica:
log (p/bar) = 4,87 1114 / (T/K 10,4).
Durante a primeira etapa do ciclo (linha 0-1 no diagrama 2), a mistura em
equilbrio, lquido refrigerante e seu vapor, recebe calor do ambiente a presI

Olimpada Brasileira de Qumica - 2015

109

47th IChO

Exame Terico

so constante p1 = 3,0 bar. O lquido refrigerante evapora completamente e


superaquece at temperatura T1 = 275 K. No inicio do processo (ponto 0), a
frao molar da fase gasosa da amnia x = 0,13.
4.1. Calcule a temperatura inicial de refrigerao T0, a variao de volume V
e a quantidade de calor Q absorvida pelo sistema refrigerante durante este
etapa. Considere que a dependncia de Hvap com a temperatura no pode
ser desprezada.
Clculos
V =

T0 =

Q=

Em seguida num processo reversvel o lquido refrigerante comprimido em


condies adiabticas e aquece at temperatura T2 = 393 (linha 1-2).
4.2. Determine o trabalho W necessrio para esta compresso e o COP do sistema. Se no conseguiu calcular o valor de Q no item 4.1, utilize Q = 20,15 kJ.
Clculos:

W=

COP =

A etapa seguinte correspondente linha 2-3 no diagrama 2, o lquido refrigerante que est comprimido resfriado num condensador presso constante. Em seguida volta ao estado inicial atravs de uma expanso adiabtica
com trabalho nulo (linha 3-0).
4.3. Determine a temperatura T3 no ponto 3 qual o lquido refrigerante
resfriado no condensador.
Clculos:
T3 =
Na produo de unidades de refrigerao necessrio ter em conta fatores
climticos. Se o condensador resfriado pelo ar atmosfrico, a temperatura
T3 aumenta se a temperatura do ar tambm aumenta.

110

Olimpada Brasileira de Qumica - 2015

47th IChO

Exame Terico

4.4. Como varia o COP se T3 aumenta e T0, T1, T2 se mantiverem inalterados?


(Escolha apenas uma opo)
a) aumenta
b) mantem-se inalterado
c) diminui
Resposta

Problema 2

Reaes qumicas acopladas


Questo
Pontuao

1.1

1.2

1.3

2.1

2.2

07 pontos
3

Total

25

Quando num sistema, uma reao faz com que uma outra reao acontea diz-se que estas reaes esto acopladas. Ilya Prigogine, que recebeu o
prmio Nobel da Qumica (1977) utiliza frequentemente nos seus livros o
conceito de reaes acopladas. Reaes acopladas so uma caracterstica
essencial nos sistemas vivos incluindo o corpo humano.

I.Prigogine (esquerda)

N. Shilov

W. Ostwald

Como uma reao faz com que outra ocorra? Neste problema vamos discutir
vrios possveis mecanismos de acoplamento.
I

Olimpada Brasileira de Qumica - 2015

111

AAsubstncia
substncia no
noreage
reagecom
coma aAc.
Ac.Contudo,
Contudo,nanapresena
presenadedeum
umterceiro
terceiroreagent
reage
indutor),
indutor),In,In,a substncia
a substncia reage
reagecom
comAc:
Ac:

47th IChO

AA AA

In the
absence
of of
In In
In the
absence

In the
presence
of of
In In
In the
presence
Acoplamento
Qumico

(I)A A

Exame Terico

nonoreaction!
reaction!

(1)(1)
(2)(2)

1 1

On Chemical
ttulo
da com
dissertao
defendida
1905 de
pelo
A coupling
substnciafoi ono
reage
a Ac. Contudo,
na em
presena
um terceiro r
qumico
russo
N.Shilov.
Ele
foi
aluno
de
doutoramento
do
Prof.
alemo
W.
m terceiro reagente (designado
indutor), In,
a substncia
reage com Ac:
Ostwald
ecatalisador!
descreveuno
seguintes
InInno
um
catalisador!
AasAsua
diminui
durante
a reao.
no
Aum
suaconcentrao
concentrao
diminui
durante
a de
reao.
substncia
reage
comreaes.
a Ac. Contudo,
na
presena
um terceiro reagente
A substncia
A
no
reage
com
a
Ac.
Contudo,
na
presena
de
um
terceiro
In reage
the absence
of In Ac:
indutor), In, a substncia
com
A A
no reaction!
(1)
reagente (designado indutor), In, a substncia A reage com Ac:
In the presence
of
In Shilov,
DeDeacordo
proposto
por
no
reage
com
acordocom
como oesquema
esquema
proposto
por
Shilov,
no
reage
com
mascom
co
AIn the absence of In
(2)A,A,mas
1
A A
no reaction!
(1)

intermedirio
dadareao
intermedirioR Rresultante
resultante
reaodede com
comIn.In.OOintermedirio
intermedirioR Rpode
podesofrer
sofr
In the presence of In
A
(2)
1
secundria
P2P. 2. um catalisador! A sua concentrao diminui durante a reao.
secundriae formar
e formar
In no

o.

In no um catalisador! A sua concentrao diminui durante a reao.


In no um catalisador! A sua concentrao diminui durante a reao.
De acordo
no reage com A, ma
k (3
a )a )o esquema proposto por Shilov,
(3
kcom
com A, mas
A Aproduto
InIn
RR
( a(com
)a ) um
intermedirio
R proposto
resultante por
da reao
com
In. O
intermedirio
R pode
De sofrer
acordo
com
o esquema
Shilov,de
Ac no
reage
com
A, mas
rio R pode
uma
reao
De
acordo
com
o
esquema
proposto
por
Shilov,
no
reage
com
A,
mas
com u
(3
)
k
b
(3
)
k
b
com
produto
intermedirio
R2.resultante
da reao de A com In. O interP
Pe2Pformar
(b()bum
(3)(3)
R secundria
) R
2
intermedirio
R
resultante
da
reao
de
com
In.
O
intermedirio
R
pode
sofrer
u
medirio R pode sofrer uma reao secundria e formar P2.
(3
)
k
c
(3
)
k
c
( c()secundria
c ) R R e formar P2. 1 1
k (3a )
A In
R
(a )

k (3
k (3
a )b) P
(3)
A(b) InR
R2
e e so
coeficientes
estequiomtricos.
Outros
coeficientes
estequiomtricos
so coeficientes estequiomtricos.
estequiomtricose ae ordem
a ord
k (3c )Outros coeficientes
k (3b)
1 (3)
(b) R ( c ) R P2
(a )

em
emrelao
relaoa atodos
todosososreagentes,
reagentes,em
emtodas
todasasastrs
trsreaes,
reaes,so
sonmeros
nmerosinteiros.
inteiros.

k (3c )
e so coeficientes
estequiomtricos.
Outros coeficientes estequiomtri(c) R
1
omtricos cos
e a ordem
de
reao
e a ordem ede reao
em
relao
a
todos
os reagentes,
em todasestequiomtricos
as trs
so coeficientes estequiomtricos.
Outros coeficientes
nena
reaes,
so
nmeros
inteiros.
meros
inteiros.
Nos experimentos de Shilov a razo das quantidades de Ac e In consumidas, I

I inteir
Nos experimentos
Shilov
a razo
das quantidades
In consumidas,
emde
relao
a todos
os reagentes,
em todasdeasAc
trsereaes,
so nmeros
n Inn
e so coeficientes
estequiomtricos.
Outros coeficientes
Nos experimentos
de Shilov
a razo das quantidades
de Ac eestequiomtricos
In consumidas, e a ordem

n relao a todos os reagentes, em todas as trs reaes, so nmeros inteiros.


em
aumenta
at
um
valor constante,
com o aumento
da
concentrao
I constante,
aumenta
at
nsumidas,
constante
um
com
o oaumento
dadaconcentrao
inicial
umvalor
valor
constante,
com
aumento
concentrao
inicialde
de[Ac]
[Ac]
a [In]
constan
0 a [In]
0=
0=
I
Nos experimentos de Shilov a razo das quantidades de Ac 0e In consumidas,
n In
inicial de [Ac]0 a [In]0 = constante.
a [In]0 = constante.
n
au
experimentos
devalor
Shilov
acom
razo
das Iquantidades
Inconstante?
consumidas,
valor
constante,
o aumento
da0 concentrao
inicial
de [Ac]0 aI[In]0 = co
[In]00e=
1.1.
Qual
oum
limite
do
da
constante
se
[Ac]
,de[In]
constante?
1.1.
o olimite
do
da
I se
[Ac]
,, Ac
[In]
constante?
1.1.Qual
QualNos
limite
dovalor
valor
daconstante
constante
I se
[Ac]
00
0==

n In

stante?
Breve
(pode
escrever
em
Portugus)
Breve
(pode
escrever
emem
Portugus)
Brevejustificativa
justificativa
(pode
escrever
Portugus)
umjustificativa
valor constante,
com
o aumento
da concentrao inicial de [Ac]0 a [In]0 = constante.

1.1. Qual o limite do valor da constante I se [Ac]0

I I= =

, [In]0 = constante?

Breve justificativa (pode escrever em Portugus)


, [In]0 = constante?
1.1. Qual o limite do valor da constante I se [Ac]0
I =
Breve justificativa (pode escrever em Portugus)
I =

o do1.2.
estado
estacionrio.
a expresso
1.2.Deduza
Deduza
a expressopara
paraI usando,
I usando,sesenecessrio,
necessrio,a aproximao
a aproximaododoestado
estadoestacion
estaci

uma
que
In foiocompletamente
1.2.
Deduza
a expresso
para
I usando,
seconstante.
necessrio,
a aproximao
doIn
estado
e
foi
Desenhe
dada
variao
dedeI vs
a [Ac]
constante.Assuma
Assumaque
queIn
foicom
co
Desenhe
ogrfico
grfico
variao
I vs[In]
[In]
0 a [Ac]
0 ==

112

Olimpada Brasileira de Qumica - 2015

Desenhe o grfico da variao de I vs [In]0 a [Ac]0 = constante. Assuma que In f


consumido
e eDeduza
estava
ememexcesso.
consumido
estava
excesso.
1.2.
a expresso
para I usando, se necessrio, a aproximao do estado estacionr

no reage com a Ac. Contudo, na presena de um terceiro reage

Exame
indutor),
In, Terico
a substncia

47th IChO

reage com Ac:

In the absence of In

A Deduza
A
reaction!
(1)
1.2.
a expresso parano
I usando,
se necessrio, a aproximao
do esIn
the
presence
of
In
tado
estacionrio.
A
(2)
1

Desenhe o grfico da variao de I vs [In]0 a [Ac]0 = constante. Assuma que In


foi completamente consumido e Ac estava em excesso.

In no um catalisador! A sua concentrao diminui durante a reao.


Clculos

De acordo com o esquema proposto por Shilov,


Grfico

(b)

(c)

In
k (3b)

k (3a )

P2

[Ac]0 = const

[Ac]0 = const

nAc/ nIn

(a )

no reage com A, mas co

com In. O intermedirio R pode sof

nAc/ nIn

intermedirio R resultante da reao de


secundria e formar P2.

onst

A substncia

(3)

k (3c )
1

[In]0

dor convencional da reaco (2)?


[In]0
E se
mecanismo
de Shilov no for
vlidocoeficientes
e In for um catalisador
convencional
r
uem. Oemecanismo
da oreao
neste
so coeficientes
estequiomtricos.
Outros
estequiomtricos
e a da
ord
In reage
com
e as as
suas
concentraes
diminuem.
O mecanismo
E se oSimultaneamente
mecanismo
de Shilov
no
fortodas
vlido
e In
for reaes,
um catalisador
convencioem relao
a todos os reagentes,
em
trs
so
nmeros
inteiros.d
nal dacaso
reaco
(2)?
Simultaneamente
In
reage
com
A
e
as
suas
concentraes

diminuem.
O mecanismo
da reao
neste

o mecanismo
de Shilov
no for vlido
e In for
um caso
catalisador
convencional da reaco (2)?
n
I
Nos experimentos
aInrazo
das
de Ac O
e In
consumidas,
(de
a ) Shilov
P2 quantidades
ltaneamente
In reage com
eAas suas
concentraes
diminuem.
mecanismo
da reao
n
n
(4)
In
,
catalysis

( b) A
1
um valor constante, com o aumento da concentrao
inicial de [Ac]0 a [In]0 = constan
1.3. Qual o limite do valor constante de I para o esquema de reao (4) se

(a )(4) seA[Ac]0In , [In]0P=2 constante?


eao
, [In]0 =deconstante?
1.1. Qual o1.3.
limite
do
valor da constante I se [Ac]
(4)
0
o limite do valor constante de I para
o esquema
reao (4) se [Ac]0
, catalysis
InQual
(b)Breve
A justificativa
1
Breve justificativa
(pode
escrever
Portugus)
(pode
escrever
em em
Portugus)
constante?

I = Breve justificativa (pode escrever em Portugus)

=
Qual o limite do valorI constante
de I para o esquema de reao (4) se [Ac]0

, [In]0 =

ante?1.2. Deduza a expresso para I usando, se necessrio, a aproximao do estado estaci


(II) Acoplamento cintico
I
Olimpada Brasileira de Qumica - 2015
I
113
e justificativa
escrever
em Portugus)
In foi c
Desenhe (pode
o grfico
da variao
de I vs [In]0 a [Ac]0 = constante. Assuma que

constante?
Breve justificativa (pode escrever em Portugus)
Breve justificativa (pode escrever em Portugus)

, [In]0 =
Qual o limite do valor constante
de I para o esquema de reao (4) se [Ac]0
Ith =
I
=
Exame
Terico
47 IChO
ante?

e justificativa (pode
em Portugus)
(II)escrever
Acoplamento
cintico
(II) Acoplamento cintico
(II) Acoplamento cintico
I = A energia
A energia
de padro
Gibbs padro
da fase-gasosa
da reao
seguinte
de Gibbs
da fase-gasosa
da reao
seguinte
A energia de Gibbs padro da fase-gasosa da reao seguinte
k5
(5)
Br H 2
HBrk5 H
(5)
Br
HBr H
Acoplamento cintico
k 5 H2
k5
positiva, G (5) = 66 kJ mol1 a
positiva,da G
(5) =seguinte
66 kJ mol1 a
ergia de Gibbs padro da fase-gasosa
reao
k5
(5)direta e inversa, r5 , a esta temperatura.
Br H 2
H a razo das velocidade da reao
r
2.1.HBr
Calcule
2.1.kCalcule
a
razo
das
velocidade
da
reao
direta
e inversa,
, a esta 5 , a esta tem
2.1.
Calcule
a
razo
das
velocidade
da reao
direta er inversa,
5
5
temperatura. Considere
que est em condies de presso padro para H2 er 5
1
a
itiva, GHBr
(5) =
66
kJ
mol
que
est
em
condies
de
presso padro para H2 e HBr e presses iguais para H e Br.
e presses iguais
para
e Br.
que est
emHcondies
de presso padro para H2 e HBr e presses iguais pa

Clculos
r
r5
Calcule a razo dasClculos
velocidade da reao direta e inversa, 5 , a esta temperatura.
r5 Considere
=
Se no tiver respondido
a esta questo, utilize rnos
clculos
Clculos
= a razo r5/r5 o
r 5seguintes para
5
r5
7no tiver respondido a esta questo, utilize nos clculos seguintes
Se
para a razo r5
3,14x10 de. presso padro para H2 e HBr e presses iguais para H e Br.
est em condies
7
3,14x10 .

Se
nosistema,
tiver respondido
esta questo,
utilize
seguintes
para a simultne
Neste
a reao (5)aevolui
no sentido
diretonos
emclculos
consequncia
da ocorrncia
r5
7
ulos
=
para a razo
r5/r5sistema,
valor
razo Neste
o ovalor
3,14
x
10
.
a reao (5) evolui
em consequncia da ocorrncia simu
r 5 no sentido direto10
reao (6):
10
Nestereao
sistema,
da
(6): a reao (5) evolui no sentido direto em consequncia
ocorrncia simultnea
da
reao
(6):
k5
Br H 2
H
(5)
kHBr
5
kH 52
ocorrncia simultnea
Br da
HBr H
(5)
10
k6 k 5
H Br2
HBr Br
(6)
k6
H Br2
HBr Br
(6)

ao

soconstantes
constantes
velocidades
das reao
e inversa
da (5)
rea-e da reao d
k5, k5, k6 so
dede
velocidades
das reao
diretadireta
e inversa
da reao
o
(5)
e
da
reao
direta
(6),
respectivamente.
k
,
k
,
k
so
constantes
de
velocidades
das
reao
direta
e
inversa
da
reao
(5) e da rea
5
5
6
(6), respectivamente.
respectivamente.
Isto (6),
o acoplamento
cintico de duas reaes.

Isto o acoplamento
cinticode
demolculas
duas reaes.
Considere
que as presses
neutras como sendo o valor padro
Isto
o2)acoplamento
de duas
reaes. dos radicais p(H), p(Br) atinp(Br
= p(HBr) = 1cintico
bar, e que
as presses
p(H2) =
gem valores de estado estacionrio. A constante de velocidade k6 10 vezes
Considere
quedireta
as presses de molculas neutras como sendo o valor padro p(H2) = p(
(5) superior
e da reao
constante
de velocidade k5.
Considere
presses dos
de radicais
molculas
neutras
sendo
o valor
padro
p(H2)
p(HBr)
= 1 bar, eque
que as
as presses
p(H),
p(Br)como
atingem
valores
de estado
estacion

p(HBr)
= 1 bar, e que
dos radicais
p(H), p(Br)
atingem kvalores
de estado est
constante
de velocidade
k6 as
10presses
vezes superior
constante
de velocidade
5.
r5
constante de velocidade
k6 10 vezes superior constante de velocidade k5.
2.2. Calcule G(5) e para estas condies.
r 5 e r5 para estas condies.
2.2. Calcule G(5)
r5
r5
I
114
ClculosI Olimpada Brasileira de Qumica - 2015G(5)
=
=
r5
r
5
G(5) =
=
r padro p(HClculos
2) = p(Br2) =

k , k , k so constantes de velocidades das reao direta e inversa da reao (5) e da reao direta

5 6
5 odireta
acoplamento
cintico
reaes.
idades dasIsto
reao
e inversa da
reaode
(5)duas
e da reao
direta

a fase-gasosa
darespectivamente.
reao seguinte
(II) Acoplamento
cintico
(6),

Considere que as presses de(5)


molculas neutras como sendo o valor padro p(H2) = p(Br2) =
Isto de
o Gibbs
acoplamento
cintico
de duas reaes.
A energia
padro
da
fase-gasosa
da reao
duas reaes.
Exame
Terico
47thvalores
IChOde estado estacionrio. A
p(HBr) = 1 bar, e que as presses dos radicais
p(H), seguinte
p(Br) atingem
1
k5
ol a constante de velocidade

10
vezes
superior

constante
de
velocidade
k5.
k
6
Br H 2 que as presses
HBr H
Considere
de molculas neutras como sendo o(5)
valor padro p(H2) = p(Br2) =
molculas neutras
como sendo
k 5 o r5valor padro p(H2) = p(Br2) =
para
estas
condies.
2.2.p(H),
Calcule
G(5)
p(HBr)
= 1atingem
bar, ee que
as presses
dos
radicais p(H),
s dos radicais
p(Br)
estado
estacionrio.
A p(Br) atingem valores de estado estacionrio. A
r5de
rvalores
1para
5 e
2.2. Calcule
G(5)
estas
condies.
, a esta
temperatura.
Considere
cidade da
reao
direta
e=inversa,
positiva,
G
(5)
66
kJ
mol
ezes superior constante
constantede
develocidade
velocidade kr65. 10 vezes superior constante de velocidade k5.
r5
5
r
Clculos
G(5) =
=
s condies. 2.2. Calcule G(5) e 5 para estas condies.
iguais
esso padro para
H2 e HBr e presses
Clculos
G(5) para H e Br.
r5 r 5
r5
, a esta temperatura. Considere
2.1. Calcule a razo das velocidade da reao direta e inversa,
r5
r 5 r5
G(5) =
=
Clculos
G(5) =
=
r5 r 5
r5
=de presso padro para H2 e HBr e presses iguais
para H e Br.
que est em condies
r 5lei da termodinmica dificulta o acoplamento
(III) A segunda

Br H

r
(III) Acom
segunda
lei dalei
termodinmica
dificulta
De acordo
a segunda
da termodinmica,
a ocorrncia
simultnea de duas reaes qumicas
ro 5acoplamento
De acordo com a segunda lei da termodinmica,
GSyst a ocorrncia simultnea
devem
diminuir
asimultnea
energia
de
do Sistema
Gsyst, a energia
0 . de Gibbs do Sistema
de
duas reaes
qumicas
devem
diminuir
termodinmica,
a ocorrncia
deGibbs
duas reaes
qumicas
t
De acordo
com a segunda lei da termodinmica, a ocorrncia
simultnea de duas reaes qumicas
G10
Syst
bs do Sistema
Gdestas
0 . pode ter energia de Gibbs positiva eGmesmo
syst,
Syst
Umadevem
reaes
diminuir
a energia de Gibbs do Sistema Gsyst,
0 .assim ocorrer no sentido direto
t
t
devido
ao acoplamento
a segunda
reao.
rgia de Gibbs
positiva
e mesmo assimcom
ocorrer
no sentido
diretoA segunda reao deve ter energia de Gibbs negativa
Uma
destas
reaes
ter energia
e mesmo
ocor10Gibbse positiva
Uma destas
reaes
podepode
ter energia
de Gibbsde
positiva
mesmo assim
ocorrerassim
no sentido
direto
e osrer
da
segunda
lei
da
termodinmica
devem ser
satisfeitos!
Considere
o exemplo.
gunda reao.
Arequisitos
segunda
reao
deve
ter
energia
de
Gibbs
negativa
no
sentido
direto
devido
ao
acoplamento
com
a
segunda
reao.
A se-negativa
devido ao acoplamento com a segunda reao. A segunda reao deve ter energia de Gibbs
termodinmicagunda
devem ser
satisfeitos!
o exemplo.
reao
deveConsidere
ter energia
de Gibbs negativa e os requisitos da segunda
e os requisitos da segunda lei da termodinmica devem ser satisfeitos! Considere o exemplo.
lei
da
termodinmica
devem
ser
satisfeitos! Considere o exemplo.
A sntese da Ureia em determinadas condies
das condies
(NH2)2CO + H2condies
O
(7)
2NH
3 + CO
2 determinadas
A sntese da
Ureia
em

(III)oA
segunda lei da termodinmica dificulta
5
nmica
dificulta
acoplamento
Clculos
= o acoplamento

condies
H2)2CO + H2OA sntese da Ureia1em determinadas
(7)

G(7) = 46,0 kJ mol

2NH3 + CO2 (NH2)2CO + H2O

G(7) = 46,0 kJ mol

(7)

est acoplada com oxidao completa da glicose (nas mesmas condies)


est acoplada com oxidao completa da glicose (nas mesmas condies)

pleta da glicose (nas mesmas condies)


CO2 + H2O

est acoplada com oxidao completa da glicose (nas mesmas condies)


+ O2 CO2 + H2O
(8)
1/6 C6H12O6 (8)
16 + O2
1/6kJ
C6mol
H12O
G(8) = 481,2
,
11
8

r(8) = 6,0x10 M min .

CO2 + H2O

11

(8)

11

Ambas as reaes esto apresentadas esquematicamente e no so consiAmbas asoutras


reaesreaes.
esto apresentadas esquematicamente e no so consideradas outras reaes.
deradas

3. Qual a velocidade mxima da reao (7) permitida pela segunda lei se esta reao est acopl
3.
Qual a velocidade mxima da reao (7) permitida pela segunda lei se
com a reao (8)?
esta reao est acoplada com a reao (8)?
Clculos
r7(max) =
Clculos
r7(max) =

Problema 3. Dois centros de ligao competio


ou cooperao?
I
Olimpada Brasileira de Qumica - 2015

115

(7 pont

3. Qual a velocidade mxima da reao (7) permitida pela segunda lei se esta reao est acoplad
3. Qual a velocidade mxima da reao (7) permitida pela segunda lei se esta reao est acopla
com a reao (8)?
com a reao (8)?
Clculos
r7(max) =
Clculos
r7(max) =

47th IChO

Exame Terico

Problema
Doiscentros
centrosdedeligao
ligao competio
competio
cooperao?
Problema
3 3.Dois
ouou
cooperao?
Problema
3. Dois
centros de ligao
competio
ou cooperao? 07 pontos

Questo
Pontuao

1
2
2
1 Total
2
Questo
Questo
1.1 1.2 2.1 2.2 2.3 2.4
2.1 2.2
2.3 2.4
1.1 1.2 2.1 2.2 2.3 2.4
228 8
3
6
6
8 Pontuao
3
6
63
Pontuao
3
2
8
3
6
6

1
1.1

1.2

(7 pontos
(7 ponto

Total
Tota
28
28

Muitas
reaes
qumicas
organismosvivos
vivosincluem
incluem aaformao
formao de
de comcomplexos do tip
Muitas
reaes
qumicas
nosnos
organismos
Muitas reaes qumicas nos organismos vivos incluem a formao de complexos do t
plexos
do
tipo
hospedeiro-hspede
(host-guest)
onde
a
molcula
doreversivelmen
hospedeiro-hspede (host-guest) onde a molcula do hospedeiro (host) liga-se
hospedeiro-hspede
(host-guest)
onde aa molcula
do
hospedeiro
(host)hspeliga-se reversivelme
hospedeiro
(host)
liga-se
reversivelmente
uma
ou
mais
molculas
a uma ou mais molculas hspedes (guest). Considere a molcula hostH com dois centros d
a uma ou
mais molculas
hspedes
(guest).
Considere
a molcula
hostH com
des (guest).
Considere
a molcula
hostH
com
dois centros
de ligao
de- dois centros
ligao denominados, a e b que tm diferentes afinidades para as molcula guest G:
nominados,
e b que tmadiferentes
para as para
molcula
guest
G: G:
ligao a
denominados,
e b que tmafinidades
diferentes afinidades
as molcula
guest

[ HGa ]
K a = [ HG
]
K a =[ H ][Ga]
[ H ][G ]
[ HGb ]
K b = [ HG
H+G
HGb
Kb Ka.
]
K b =[ H ][Gb]
H+G
HGb
Kb Ka.
[ H ][G ]
onde HGa e HGb representam complexos onde G est ligado aos centros a e b respectivamenteb. K
onde onde
HGa e
HG
representam
complexos
onde
ligado
aos centros
aa
complexos onde
GG
estest
ligado
aos centros
a e b respectivamenteb.
HG
a ebHG
b representam
e Kb so as constantes de ligao aos centros a e b, respectivamente e os colchetes significam
e b respectivamenteb.
K
e
K
so
as
constantes
de
ligao
aos
centros
a
e
b,
a
b
e Kb so as constantes de ligao aos centros a e b, respectivamente e os colchetes
signific
concentraes molares.
respectivamente
e
os
colchetes
significam
concentraes
molares.
concentraes molares.
A ligao de uma molcula G a uma de H pode alterar a capacidade de ligao ao segund
A ligao deAuma
molcula
a umaGdea uma
H pode
a capacidade
dedeligaligao
de uma G
molcula
de Halterar
pode alterar
a capacidade
ligao ao segun
centro. Esta alterao descrita pelo fator de interferncia , que reflete a influncia de um centr
o aocentro.
segundo
centro.
Esta
alterao

descrita
pelo
fator
de
interferncia
Esta alterao descrita pelo fator de interferncia , que reflete a influncia de um cen
dereflete
ligao noinfluncia
outro e definida
, que
de umpor:
centro
de ligao no outro e definida por:
de ligaoa no
outro e definida
por:
[ HG2 ]
HGa + G
HG2
[ HG2 ] = K b
[ HGa ][G
] = Kb
HGa + G
HG2
[ HGa ][G ]
onde HG2 o complexo completamente ligado.
onde HG2 o complexo completamente ligado.

H+G
H+G

HGa
HGa

onde HG2 o complexo completamente ligado.

Determine
faixade
de valores
valores (ou
um valor,
se necessrio)
de quede
corresponde
s tr
1.1. 1.1.
Determine
a afaixa
(ouapenas
apenas
um valor,
se necessrio)

1.1. Determine a faixa de valores (ou apenas um valor, se necessrio) de que corresponde s t
possibilidades de
entre os centros
de ligao: a)
cooperao
(a ligao
que corresponde
sinterferncia
trs possibilidades
de interferncia
entre
os centros
de a um centr
possibilidades de interferncia entre os centros de ligao: a) cooperao (a ligao a um cen
ligao:
a) cooperao
(a ligao
um
centro facilita
ligao
ao outro
centro);
facilita
ligao ao outro
centro);ab)
competio
(a primeira
ligao
dificulta
a ligao ao outr
facilita ligao
ao outroligao
centro);dificulta
b) competio
(a primeira
ligao
dificulta
a ligao ao ou
b) competio
(a
primeira
a
ligao
ao
outro
centro);
c) indecentro);
centro);
pendente
(no h interferncia).

Cooperao:

116

Competio:

Olimpada Brasileira de Qumica - 2015

12
12Independncia:

c) independente (no h interferncia).


c) independente (no h interferncia).
Cooperao:
Cooperao:

Competio:
Competio:

Exame Terico

Independncia:
Independncia:

47 IChO
th

1.2. Calcule a constante de equilbrio do processo: HGb + G

1.2. Calcule
a constante
de equilbrio
processo: HGb + G
1.2. Calcule
a constante
de equilbrio
dodo
processo:
ligao e fator de
de interferncia.
mos de constante(s)
ligao
e
fator
de
interferncia.
ligao e fator de interferncia.

K=

Clculos
Clculos:

HG2 em termos de const


HG2 em
emtermos
ter- de cons

K=
K=

Clculos:

2.1. A soluo
foi preparada
comcom
concentraes
[H]00 ==1M
1Me e[G]
[G]
2.1. A soluo
foi preparada
concentraes iniciais
iniciais [H]
= =2M. Aps
0 0
2.1.
Areaes
soluo foi
preparada
com concentraes
iniciais [H]
1M
e [G]0 10
= 2M. Aps
0=
2M. Aps
as
estarem
completas,
a
concentrao
de
H
diminui
estarem completas, a concentrao de H diminui 10 vezes e a de G de 4 vezes. P
completas,
concentrao
de H diminui
10 Kvezes
a de G de
vezes e aestarem
de G de
4 vezes.a Para
estas molculas
H e G,
as 4 vezes.
b = 2Ke
a. Calcule
molculas H e G, Kb = 2Ka. Calcule as concentraes de todas as espcies em soluo, a
concentraes
de
todas
as
espcies
em
soluo,
a
constante
de
ligao
K
e soluo, a
a
molculas H e G, Kb = 2Ka. Calcule as concentraes de todas as espcies em
deinterferncia
ligao Ka e o fator
de interferncia .
o fator de
.
de ligao Ka e o fator de interferncia .
Clculos:
[HGa] = [HGb] =
Clculos:
[HGa] =
Clculos:

[HGa] =

[HG
b] =
[HG
2] =
[HGb] =

Ka =[HG2] = = Ka =
[HG2] =

Ka =

Se no tiver respondido a esta questo, utilize nos clculos seguintes para a constante K
Se no tiver respondido a esta questo, utilize nos clculos seguintes para a constante
3,14 respondido
e para o valora de
2,72.
Se no tiver
esta
questo, utilize nos clculos seguintes para a
3,14 e para o valor
de 2,72.

constante Ka o valor 3,14 e para o valor de 2,72.

2.2. Identifique a ordem correta da energia de Gibbs padro molar correspondente form

2.2. Identifique
a ordem
correta
da energiade
de Gibbs padro
molar
correspondente
form
2.2. Identifique
ordem
correta
da energia
molar
corres- abaixo
e de todosa os
complexos
formados
entre H e G.Gibbs
Utilizepadro
o esquema
representado
e
todos os complexos
formados
H e G. Utilize
o esquema
representado
pondentee de
formao
de H e de
todos entre
os complexos
formados
entre
H e G. abaixo
frmula qumica correspondente a cada linha.
Utilize o esquema
representado
abaixo
e escreva
frmula qumica
correspondente
a cada
linha. a frmula qumica correspondente a cada linha.

2.3. Uma certa quantidade de G foi adicionada a 1 mol de H e a mistura foi


dissolvida em gua obtendo-se 1 litro de soluo. O nmero de molculas
Uma certa
quantidade
de Gtotal
foi adicionada
a 1 mol
de H e uma
a mistura
foi dissolvida
HG2 em2.3.
soluo
igual
ao nmero
molculas
a efetuar
s liga2.3.
Uma certa
quantidade
de G foi de
adicionada
a 1 mol
de H e a mistura
foi dissolvid
em
soluo

igual
obtendo-se
1
litro
de
soluo.
O
numero
de
molculas
HG
2
o, HG. Calcule a quantidade inicial de G (em mol). As constantes Ka e Kb e o ao nmer
obtendo-se 1 litro de soluo. O numero de molculas HG2 em soluo igual ao nme
fator so
iguais s
da questo
molculas
a efetuar
uma s 2.1.
ligao, HG. Calcule a quantidade inicial de G (em mol). As

molculas a efetuar uma s ligao, HG. Calcule a quantidade inicial de G (em mol). As
Ka e Kb e o fator so iguais s da questo 2.1.
Clculos:
n0(G)
= 2.1.
Ka e Kb e o fator so iguais s da
questo
Clculos:
Clculos:

n0(G) =
n0(G) =
I

Olimpada Brasileira de Qumica - 2015

13

117

47th IChO

Exame Terico

2.4. Qual ser a composio da soluo no equilbrio se: a) = 0; b) muito


muitogrande
grande (
(
As constantes Ka e Kb assim como as concentraes iniciais de
). As
H equesto
G so2.1.
iguais s da questo 2.1.
iguais s da
=0
Clculos:
[H] =

[G] =

[HGa] =

[HGb] =

muito
(
). As
[HGgrande
b] =
is s da questo
2.1. [Breve justificativa (pode escrever em Portugus)
Clculos

HGb[HG
] = 2] =

mples

[H] =

Problema 4

[G] =

[HGa] =

[HGb] =

[HG2]

De um p amarelo a outro:
um enigma inorgnico simples

06 pontos

Questo

Total

Pontuao

24

1
2
3
4
Total
binrio
amarelo X1 foi completamente dissolvido por aqueciHG
8 2] = 8 O 3composto
5
24
mento em cido ntrico concentrado, o gs liberado 1,586 vezes mais denso que o ar. Pela adio de excesso de cloreto de brio soluo formada,
aquecimento
cidobranco
ntrico X2 precipita. Este foi filtrado. O filtrado reage com um exumem
slido
sdio de excesso
cesso de
soluo
de cloreto
de de sulfato de prata formando um precipitado de dois solidos
X
2 e X3, tambm separados da soluo por filtrao. Ao novo filtrado foi
ado. O filtrado reage com um
2
3 adicionado,
4
Totalgota a gota, uma soluo de hidrxido de sdio, at a soluo
dois solidos X2 e X3, tambm
ficar aproximadamente neutra (em torno de pH=7). Neste momento, um p
8 a 3gota, uma
5
24
gota
de % em massa de Ag) cristalizou da soluo. A massa de X4
amarelosoluo
X4 (77,31
(em tornoaproximadamente
de pH=7). Neste 2,4 vezes maior do que a massa da primeira poro de X2.
cimento
em
ntrico
da soluo.
A massa
de X

1. cido
Determine
as4 frmulas
qumicas X1 X4.
ode
deXexcesso
de cloreto de
2.
Clculos:
X2
X3
X4
O filtrado reage
com um X1
olidos X2 e X3, tambm
a a gota, uma soluo de
torno de pH=7).X4Neste
=
oluo. A massa deI XOlimpada
4
Brasileira de Qumica - 2015
I
118
.
2

47th IChO

Exame Terico

2. Determine a frmula qumica do gs e mostre as equaes para todas as


reaes na forma inica ou no inico.
Clculos:
Frmula qumica do gs ________
Dissoluo de X1
Formao de X2
Formao de X2 e X3
Adio de NaOH e formao de X4
3. Na unidade estrutural de X1 todos os tomos de posies equivalentes so
de um mesmo elemento. Desenhe a estrutura de X1.
4. Preveja os produtos da interao X1 com:
a) excesso de oxignio;
b) excesso de cido sulfrico concentrado quente;
c) slido KClO3 moido.
Escreva abaixo as respectivas equaes das reaes.

Problema 5

indispensvel glucose

Questo

8 pontos

Pontuao 1.1 1.2 1.3 1.4 1.5


2

Total

1
4

1.6

2.1

2.2

2.3

2.4

2.5

34

Os carboidratos so as mais importantes fontes de energia para clulas vivas.


O monossacardeo glicose uma fonte de energia de clulas vivas, mas para
pessoas que sofrem de diabetes a glicose pode causar danos. Alto nvel de
glicose pode levar a doenas cardiovascular e at a morte. As pessoas devem evitar o consumo de muito carboidrato e de particularmente a glicose.

Olimpada Brasileira de Qumica - 2015

119

47redutores
IChO no suco de fruta
terminao de acares
th

Exame Terico

1. Determinao
de acares
redutores
no suco
de fruta
das tcnicas
para a determinao
de acares
redutores
em diferentes

Uma
das do
tcnicas
para
a determinao
de acares
redutoras incluem
o uso
reagente
de Fehling.
Uma alquota
de 10,00mL
de
res em diferentes amostras incluem o uso do reagente de
de fruta (considerando uma amostra inicial contendo somente glicose e
Fehling. Uma alquota de 10,00mL de suco de fruta (cone) foi transferida
parauma
um amostra
frasco deinicial
titulao
e o reagente
de Fehling
siderando
contendo
somente
glicosefoie
frutose)
foi
transferida
para
um
frasco
de
titulao
e o reanado. Este reagente foi preparado pela mistura de 50,00 mL de soluo
de
gente de Fehling foi adicionado. Este reagente foi preparao de cobre
M (soluo
A) emL
tartarato
de potssio
e hidrxido
de
do0,04000
pela mistura
de 50,00
de soluo
de sulfato
de cobre
0,04000
M
(soluo
A)
e
tartarato
de
potssio
e
hidrxido
de
(soluon B). A soluo C assim obtida, foi ento aquecida e um
sdio (soluo B). A soluo C assim obtida, foi ento aquepitado vermelho
foi formado.
cida e um
precipitado vermelho foi formado.

Glucose

1.1. Escreva a equao inica balanceada das reaes qumicas que ocorparaocorrem
soluosob
de aquecimento
cobre inicial.da
sob inica
aquecimento
da das
soluo
C. Use
Cu2+ que
screva a rem
equao
balanceada
reaes
qumicas

o C. Use Cu2+ para soluo de cobre inicial.

Depois disso, 10 mL de soluo de iodeto de potssio 10% e cido sulfrico


1 M foram adicionado ao frasco. A mistura foi coberta com vidro de relgio e
s disso, 10
mL de
soluo de
iodeto
de potssio
10%
cido sulfrico
1M
adicionado
ento
colocada
em
um lugar
escuro.
Umeexcesso
de iodo
foiforam
ento
titulado
com
uma
soluo
de
tiossulfato
de
sdio
0,05078
M
.
11,87
mL
do
titulante
sco. A mistura foi coberta com vidro de relgio e ento colocada em um lugar escuro. Um
foram necessrios para atingir o ponto final.
so de iodo foi ento titulado com uma soluo de tiossulfato de sdio 0,05078
. 11,87 mL
1.2. Escreva a equao ou equaces balanceadas na forma molecular ou
ulante foram
necessrios
paraas
atingir
o ponto
inica
para todas
reaes
quefinal.
ocorreram no frasco.

screva a equao ou equaces balanceadas na forma molecular ou inica para todas as reaes

correram no frasco.
1.3. Considere que toda frutose foi transformada em glicose sob as condies experimentais; Calcule a massa total de acar contida no suco de fruta
(em g/L). Massa molar = 180,16 g/mol.
massa contida =

Considere que toda frutose foi transformada em glicose sob as condies experimentais;

le a massa total de acar contida no suco de fruta (em g/L). Massa molar = 180,16 g/mol.
contida =

120

Olimpada Brasileira de Qumica - 2015

47th IChO

Exame Terico

Uma nova alquota de 10,00mL do mesmo suco foi tratada com uma poro
de 10,00 mL de soluo de iodato(V) de potssio acidificada 0,01502 M e
10 mL de soluo de iodeto de potssio 10 % . Aps a mistura ficar marron,
um excesso de soluo de hidrxido de sdio foi adicionada. O frasco foi
ento coberto com um vidro de relgio e colocado em um lugar escuro. A
soluo obtida foi acidificada e titulada com soluo de tiossulfato de sdio
0,01089M . A mdia dos volumes usados nas titulaes foi 23,43 mL. Note
que a frutose nao convertida em glicose sob essas condies.
1.4. Escreva todas as equaes balanceadas para descrever as reaes na
forma inica ou molecular.

1.5. Calcule a massa contida de cada acar no suco (em g/L) .


massa contida de glicose =
massa contida de frutose =
1.6. Uma unidade de troca de po (bread exchange unit = 1 BEU) corresponde a 12 g de carboidrato digervel no produto. Quantos BEU esto em
um copo de suco (200 mL) ?

2. Diagnstico de doenas
O derivado da glicose, 2-deoxi-2-(18F)fluoro-D-glicose (FDG), o mais comum radiofrmaco para diagnstico de cncer usando tomografia por emisso de psitron. A primeira etapa da preparao do FDG produzir um radionucldeo flor-18 pela reao nuclear em um ciclotron. A prxima etapa
uma sntese radioqumica. Flor-18 introduzido na molcula D-glicose por
substituio nucleoflica. O 2-deoxi-2-(18F)fluoro-D-glicose uma vez injetado
no paciente acumulado ativamente nas clulas de tumores malgnos; este
processo acompanhado pela decomposio do flor-18. Este radionucldeo um emissor +, emite um psitron (anti-eltron). O psitron interage
I

Olimpada Brasileira de Qumica - 2015

121

47th IChO

Exame Terico

com o eltron e depois que ocorre a aniquilao, pode ser detectado. Esta
sequncia determina precisamente o tamanho e o tipo de tumor.
2.1. Complete as reaes nucleares que produzem os vrios istopos de
flor.

a)

18

b) ...

c)

19

d)

16

.. +
2
1

2
1

+ ......

18

18

20

+ ..

18

+ n

2.2. O modo de decaimento de ncleos instveis leves depende da razo


2.2. O modo de decaimento de ncleos instveis leves depende da razo entre os seus nme
entre os seus nmero de nutrons e prtons. Se esta razo for maior do que
nutronsento
e prtons.
Se esta razo
forem
maior
domodo
que a doistopo
estvel,
entaono
o ncleo decai e
a do istopo estvel,
o ncleo
decai
um
, se for
menor,

+
+
,
se
for
menor,
no
modo
.
modo
modo .
Determine o tipo de decaimento para os ncleos na tabela:
Ncleos

Determine o tipo de decaimento para os ncleos na tabela:


Ncleos

11

20

11

Modo de Modo de
decaimento

20

F17

17

14
14

decaimento

Quando a reao nuclear (a) usada para a preparao do flor-18, o material identificado como gua enriquecida com H218O. A presena da gua
comum H216O Quando
leva a uma
reao nuclear paralela com 16O, levando formaa reao nuclear (a) usada para a preparao do flor-18, o material identificado
17
o do istopo F.
18
16

gua enriquecida com H2 O. A presena da gua comum H2 O leva a uma reao nuclear pa

16
17
O, levando
formao
do istopo
F.
com nos
2.3. Sabe-se que
5 minutos
seguintes
concluso
da radiao do alvo
18
17
5
a razo das radioatividades de F e F 10 . Considerando que o tempo
de radiao 2.3.
curto
e a radioatividade de cada istopo proporcional ao
Sabe-se que nos 5 minutos seguintes concluso da radiao do alvo a razo
rendimento da reao nuclear
e a frao
molar de um componente no
e 17Fmassa
105.Considerando
tempot de(18radiao
curto e a radioativ
de 18F em
alvo irradiado,radioatividades
calcule a frao
de H218O que
no oalvo.
F) = 109,7
1/2
17 de cada istopo proporcional ao rendimento da reao nuclear e a frao molar de um compo
minutes, t1/2( F) = 65 segundos. A razo entre os rendimentos das reaes
16 -17
nucleares 18noO-18
144,7.
F / irradiado,
O
F =
alvo
calcule
a frao em massa de H218O no alvo. t1/2(18F) = 109,7 minutes, t1/2(

65 segundos. A razo entre os rendimentos das reaes nucleares

122

Olimpada Brasileira de Qumica - 2015

= 144,7

47th IChO

Exame Terico

2.4. Calcule o rendimento da D-glicose marcada com flor-18, se a radioatividade inicial de uma amostra de flor-18 era 600,0 MBq e a radioatividade
da 2-deoxi-2-(18F)fluoro-D-glicose obtida 528,2 MBq. Tempo de sntese
3,5 minutos.

=
2.5.

2.5. A meia-vida biolgica (atravs dos rgos excretores) do 2-deoxi-2-(18F)


fluoro-D-glicose 120,0 minutes. Quanta radioatividade (emn MBq) permaAnecer
meia-vida
biolgica (atravs dos rgos excretores) do 2-deoxi-2-(18F)
no paciente aps 10 horas da injeo de FDG com radioatividade
inicial de 450,0 MBq.

120,0 minutes. Quanta radioatividade (emn MBq) permanecer no paciente ap


injeo de FDG com radioatividade inicial de 450,0 MBq.
Problema 6

Po a essncia da vida

1 da2 vida
3
PROBLEMA 6. Po Questo
a essncia
Pontuao

8 pontos

(8 pontos)

Total
10

Questo

Quando voc passa por uma padaria,


voc parado pelo aroma do po feito
Pontuao
7
na hora. O heri de uma novela disse em
Quando
passa
por uma
padaria,
voc parado pelo aroma do po feit
umavoc
ocasio
parecida:
Se voc
me disser que isto no perfeito, voc ser meu
uma inimigo
novela para
disse
em uma
ocasiocomparecida: "Se voc me
sempre.
O principal
ponente flavorizante foi identificado em
disser1969
quecomo
isto ono
perfeito,
ser meu inimigo para
composto
X que voc
est em
equilbrio com o seu tautmero Y em uma
sempre".
O principal componente flavorizante foi identificado
razo de 2:1. Infelizmente, ambas as formas so lbeis e, depois de algumas
horas,
o poono
tem o mesmo
aroma.
em 1969
como
composto
X que
est em equilbrio com o seu
Esta mistura tautomrica de X e Y foi sintetizada em 1993 a partir da piperidi-

tautmero
em uma
2:1.
Infelizmente,
ambas
na pelaY
sequncia
de razo
reaesde
dada
pelo
esquema 1. digno
de as
nota que a

razo inicial de X e Y que era 1:4 muda gradualmente at atingir o equilbrio.

formas so lbeis e, depois de algumas horas, o po no tem o


mesmo aroma.

Esta mistura tautomrica de X e Y foi sintetizada em 1993 a par

sequncia de reaes dada pelo esquema 1. digno de nota que a razo inicia
I

Olimpada Brasileira de Qumica - 2015

muda gradualmente at atingir o equilibrio.

123

Esta mistura tautomrica de X e Y foi sintetizada em 1993 a partir da piperidina

ncia de reaes dada pelo esquema 1. digno de nota que a razo inicial de X e Y que era

a gradualmente at atingir
o equilibrio.
th

47 IChO

uema 1. Esquema 1.
abaixo as frmulas

Exame Terico

estruturais dos compostos A-E, X, Y.

atamento do composto E com o complexo CH3Li-LiBr em (C2H5)2O a 0oC

as desejadas
X e Y.
Em vez
disso, um
amarelo
F foi
formado i
O composto
B que
caracterizado
por precipitado
eixo de simetria
de ordem
3 (ou

O composto B que caracterizado


por eixo de simetria de ordem 3 (ou seja, uma rota
o
seja, uma rotao de 120 resulta numa molcula no distinguvel da original)

o resulta
aquoso
deste
precipitado
a mistura
dos
composto
Edestas
e seu tautme
ocorre
em
equilbriono
comoriginou
o seu diastereoismero
C. A interconverso
numa
molcula
distinguvel
da original)
ocorre
em equilbrio
com o
duas formas acontece via o intermedirio A que tambm intermedirio na

tereoismero
C. A
duas
formas acontece
viacompostos
o intermedirio A
formao
de interconverso
B e C bem comodestas
nas suas
transformaes
em D. Os

A, B e C tem a mesma composio centesimal: C = 72,24%, H = 10,91%, N


bm
intermedirio
formao de
e C bem como
baixo
as frmulas na
estruturais
dosBcompostos
F e nas
G. suas transformaes em D.
= 16,85%.

72,24%,
postos A,
e C tem
a mesma
composio
centesimal:
C =A-E,
H = 10,91%,
1. .BEscreva
abaixo
as frmulas
estruturais
dos compostos
X, Y.
5%.
O tratamento do composto E com o complexo CH3Li-LiBr em (C2H5)2O a 0oC

no produz as substncias desejadas X e Y. Em vez disso, um precipitado


amarelo F foi formado inicialmente. O tratamento aquoso deste precipitado
aminhooriginou
para aa obteno
de X baseado
no usoG.do derivado do cido
mistura dos composto
E e seu tautmero

pipe

Escreva abaixo as frmulas estruturais dos compostos F e G.


X pode2.ser
sintetizado pela sequncia de reaes apresentada no Esquema 2
Um outro caminho para a obteno de
19X baseado no uso do derivado do
cido pipecolnico H. Sabe-se que X pode ser sintetizado pela sequncia de
reaes apresentada no Esquema 2.
Esquema 2.

3. Escreva abaixo as frmulas estruturais dos compostos I e J.

baixo as frmulas estruturais dos compostos I e J.


124

Olimpada Brasileira de Qumica - 2015

47th IChO

Exame Terico

Problema 7

No apenas pelo po

8 pontos

Questo

Total

Pontuao

24

16

50

A rom chamado no Azerbaijo (famoso


por seus vegetais) como a rainha de todas as
frutas. Rom intitulado em vrias religies
como uma fruta do paraso, smbolo de justia, sade e garantia da vida eterna.
Em 1878 o alcalide pelletierina foi isolado
a partir da casca da rvore da rom (Punica
Este alcalide
tradicionalmente
usado
como um medicamento
anti-helmntico. Inicialmente, Xw
granatum
L., Lythraceae).
Este
alcalide
tra(3-(piperidin-2-il)propional)
foi incorretamente
proposto para anti-helmntico.
pelletierina. Mas agora
aceito que a
dicionalmente
usado como
um medicamento
Inicialmente,
foi incorretamente
proposto para pelletierina.
Xpelletierina
w (3-(piperidin-2-il)propional)
(S)-1-(piperidin-2-il)propan-2-ona
(XS).
Mas agora aceito que a pelletierina (S)-1-(piperidin-2-il)propan-2-ona (XS).
XS informaes
(o ltimoestereoqumicas)
com informa1.1.Escreva
abaixo
as frmulas
estruturais
deltimo
Xw ecom
Escreva abaixo
as frmulas
estruturais
de XW e XS (o
es estereoqumicas)
XW
XS
(3-(piperidina-2-il)propanal)
Xw
(3-(piperidina-2-il)propanal)

(S)-1-(piperidin-2-il)propan-2-ona
XS
(S)-1-(piperidin-2-il)propan-2-ona

sntese da
natural
(XS) (X
baseada
na transformao
do nortropanoldo
A nortropafoi descrita
na transformao
AAsntese
dapelletierina
pelletierina
natural
S) baseada
nol
A
foi
descrita
recentemente.
recentemente.

2. Escreva abaixo as frmulas estruturais dos compostos B-G com as informaes estereoqumicas.
I

Olimpada Brasileira de Qumica - 2015

125

47th IChO

Exame Terico

2. Escreva abaixo as frmulas estruturais dos compostos B-G com as informaes estereoqumicas.
3. Nortropanol A foi usado nesta reao como um nico estereoismero.
Quantos estereoismeros existem para o composto A (incluindo A). Ignore a
quiralidade do nitrognio.
O nmero de possveis estereoismeros de A
O enantimero de XS foi sintetizado usando tert-butanosulfinamida quiral (H):

4. Escreva abaixo as frmulas estruturais dos compostos I a L com as informaes estereoqumicas.


4. Escreva abaixo as frmulas estruturais dos compostos I a L com as informaes estereoqumicas.

Problema 8. Petrleo para a vida e vida aps o petrleo


Questo
Pontuao

(8 pontos)
1

1a

1b

1c

1d

1e

12

Total

13

13

55

O Azerbaijo conhecido por seus vastos campos de petrleo. A


primeira perfurao para extrao de petrleo foi feita em BibiHeybat, em 1846, 13 anos aps o estabelecimento do primeiro
poo de petrleo comercial na Pensilvnia (USA). Esta marcante
data
na histria
do Azerbaijo
considerada
como o ponto de
I
Olimpada
Brasileira de Qumica
- 2015
I
126
partida da indstria contempornea de petrleo que lidera o setor da economia mundal de hoje.

47th IChO

Exame Terico

Problema 8
Questo
Pontuao

Petrleo para a vida e vida aps o petrleo


1
1a

1b

1c

1d

1e

12

13

13

8 pontos
Total
55

O Azerbaijo conhecido por seus


vastos campos de petrleo. A primeira
perfurao para extrao de petrleo
foi feita em Bibi-Heybat, em 1846, 13
anos aps o estabelecimento do primeiro poo de petrleo comercial na
Pensilvnia (USA). Esta marcante data
na histria do Azerbaijo considerada
como o ponto de partida da indstria
contempornea de petrleo que lidera
o setor da economia mundial de hoje. Atualmente a produo de petrleo
no continente e no mar est sendo desenvolvida no Azerbaijao. Embora srias precaues sejam tomadas, sempre h o risco de poluio do ambiente
por hidrocarbonetos, durante a produo, o transporte e o processamento
do petrleo. Neste problema voc ir considerar diversas tecnologias empregadas na limpeza de vazamentos e aspectos especficos de caminhos
metablicos envolvidos.
A aplicao de solventes complexos (dispersantes) levando a captura de
leo derramado no mar est entre as mais promissoras abordagens de limpeza de derramamentos. A substncia orgnica X (11,94% de H em massa)
um componente tpico de tais dispersantes. A segurana de X para humanos
fortemente debatida. X1 (54,53% de carbono em massa) constituido de
trs elementos e excretado na urina o principal metablito de X em humanos. Os nmeros de tomos de diferentes elementos em X1 so trs termos
consecutivos de uma progresso geomtrica (n, nq, nq2), enquanto a soma
destes nmeros no excede 25.
1a. Decida sobre a relao entre o nmero de tomos de carbono e de oxignio em X1 (marque a opo correta).
n(C) > n(O)

n(C) < n(O)

n(C) = n(O)

Data insufficie

Olimpada Brasileira de Qumica - 2015

127

47th IChO

Exame Terico

1b. Deduza a frmula emprica de X1 (em seguida sempre mostre as suas


dedues quando requerida). Tenha certeza de ter provado sua resposta por
clculo.
Suas dedues
Frmula emprica de X1:

A biotransformao de X em X1 ocorre em duas etapas enzimaticamente catalisadas, de acordo com as equaes balanceadass abaixo (NAD+ e NADH
so, respectivamente, as formas oxidada e reduzida da nicotinamida adenina
dinucleotdeo):
X + NAD+ X0 + NADH + H+

(1)

X0 + NAD + H2O X1 + NADH + H


+

(2)

1c. Deduza a frmula molecular of X.


Suas dedues
Frmula emprica de X:

Uma menor transformao de X catalisada pelo citocromo P450-monooxigenase dependente. Esta reao leva a dois compostos X2 (51,56% de oxignio e 9,74% de hidrognio em massa) e X3.
1d. Deduza a frmula molecular de X2 e desenhe sua estrutura.
Suas dedues
Frmula molecular de X2:

128

Olimpada Brasileira de Qumica - 2015

Estrutura de X2:

47th IChO

Exame Terico

X contm somente tomos de carbono primrios e secundrios. X0 e X3 contm grupos funcionais comuns.
1e. Desenhe as frmulas estruturais de X, X1 e X3.
Em um estudo mdico, uma pessoa exposta a solventes contendo X, sem
a proteo apropriada, apresentou uma concentrao estacionria de X no
sangue.
2. X1 excretado na urina. Escolha o grfico da variao do contedo em
massa de X1,durante o dia, no sangue de um participante voluntrio deste
experimento. Escreva abaixo o nmero do grfico correto.

Nmero do grfico:
O uso de diferentes bactrias , tambm, considerado como um caminho
promissor para a remoo de hidrocarbonetos (mesmo aromtico) contamiO uso
de diferentes
bactrias
, tambm,
como um caminho
promissor para
a remoo bionantes da
gua
do mar
e do
solo.considerado
Sob condies
aerbicas,
benzeno
de hidrocarbonetos (mesmo aromtico) contaminantes da gua do mar e do solo. Sob condies
degradado
como segue (as primeiras trs etapas esto balanceadas):
aerbicas, benzeno biodegradado como segue (as primeiras trs etapas esto balanceadas):

Sob as mesmas condies, um hidrocarboneto aromtico monocclico P (91,25% de carbono em


I
Olimpada Brasileira de Qumica - 2015
I
massa) sofre a seguinte transformao (as primeiras trs etapas esto balanceadas):

129

O uso de diferentes bactrias , tambm, considerado como um caminho promissor para a remoo
O uso de diferentes bactrias , tambm, considerado como um caminho promissor para a remoo
de hidrocarbonetos (mesmo aromtico) contaminantes da gua do mar e do solo. Sob condies
de hidrocarbonetos (mesmo aromtico) contaminantes da gua do mar e do solo. Sob condies
aerbicas, benzeno biodegradado como segue (as primeiras trs etapas esto balanceadas):
aerbicas, benzeno
biodegradado como segue (as primeiras trs etapas esto balanceadas):
th

47 IChO

Exame Terico

Sob as mesmas condies, um hidrocarboneto aromtico monocclico P


(91,25%
carbono
em massa)
sofre aaromtico
seguinte
transformao
(as primeiras
Sobde
as mesmas
condies,
um hidrocarboneto
monocclico
P (91,25% de carbono
em
Sob as mesmas condies, um hidrocarboneto aromtico monocclico P (91,25% de carbono em
trs etapas
esto
balanceadas):
massa) sofre
a seguinte
transformao (as primeiras trs etapas esto balanceadas):
massa) sofre a seguinte transformao (as primeiras trs etapas esto balanceadas):

P3 d teste de iodofrmio positivo. Uma amostra de 100 mg de P3 requer 6,41 mL de soluo de


P3 d teste de iodofrmio positivo. Uma amostra de 100 mg de P3 requer 6,41 mL de soluo de
KOH 0,100 M para completa neutralizao.
P3 d KOH
teste
deMiodofrmio
positivo. Uma amostra de 100 mg de P3 requer
0,100
para completa neutralizao.
3.
Deduza as estruturas de PP3. D o tautmero mais estvel de P3.
3. de
Deduza
as estruturas
de PP3.
D o tautmero
estvel de P3.
6,41 mL
soluo
de KOH
0,100
M paramais
completa
neutralizao.
Suas dedues
Suas dedues

3. Deduza as estruturas de PP3. D o tautmero mais estvel de P3.


Suas dedues
P

P
P

P1
P1P1

P2
P2
P2

P3
P3

P3

Os microorganismos Alicycliphilus so capazes de realizar a biodegradao


at mesmo de hidrocarbonetos aromticos no solo. O processo requer acepOs eltrons
microorganismos
Alicycliphilustais
so como
capazes o
de nion
realizar inorgnico
a biodegradao Y1
at (as
mesmo
de
tores de
adequados
primeiras
Os microorganismos Alicycliphilus so capazes de realizar a biodegradao at mesmo de
hidrocarbonetos
aromticos
no solo. O processo requer aceptores de eltrons adequados tais como o
trs etapas
esto
balanceadas).
hidrocarbonetos aromticos no solo. O processo requer aceptores de eltrons adequados tais como o
nion inorgnico Y1 (as primeiras trs etapas esto balanceadas).
nion inorgnico Y1 (as primeiras trs etapas esto balanceadas).

O nion intermedirio Y2 decomposto enzimaticamente de acordo com a


equao de reao balanceada:

O nion intermedirio Y2 decomposto enzimaticamente de acordo com a equao de reao


Y3(aq) + Y3(g),
O nion intermedirio Y2 Y2(aq)
decomposto enzimaticamente
de acordo com a equao de reao
balanceada:
balanceada:
Y2(aq)cada
Y3(aq)
+ Y4(g),
qual, Y3 e Y4 so constitudos,
um,
de tomos de um nico elemenY2(aq) Y3(aq) + Y4(g),

Na
to. T2 no contm dois grupos funcionais oxigenados idnticos. T2 d um
precipitado quando tratado com soluo Ag2O em amnia, enquanto que
Y3 no d.
25
25

4. Deduza e d as frmulas de Y1-Y4. Desenhe as estruturas de T1-T2. D o


tautmero mais estvel de T2.

130

Olimpada Brasileira de Qumica - 2015

47th IChO

Exame Prtico

47a Olimpada
Internacional
de Qumica
25 de julho de 2015
Baku - Azerbaijo
INSTRUES GERAIS
s Regras de segurana: siga as instrues fornecidas no caderno de Problemas Preparatrios.
s No coma ou beba no laboratrio.
s Sempre use o seu jaleco e os culos de segurana enquanto estiver no laboratrio.
Pea para o assistente luvas do seu tamanho.
s Violao das regras de segurana: voc receber apenas um aviso; na reincidncia:
voc ser desqualificado.
s Caderno de Experimentos e respostas: 28 paginas (incluindo a capa e a tabela peridica de elementos) com 3 Experimentos.
s Tempo: 5 h; voc ter 30 min para leitura antes do incio e um aviso ser dado 30 min
antes do final.
s Seu cdigo de estudante: escreva-o na parte superior de cada pgina!
s Respostas: Escreva as suas respostas apenas nos espaos destinados e nas clulas do
arquivo no seu pen drive. Respostas escritas em qualquer outro lugar no sero computadas. Clculos relevantes devem ser mostrados.
s Use apenas a caneta, o lpis e a calculadora disponveis.
s Obtenha a leitura da bureta com o mximo de preciso.
s So necessrios mais reagentes ou vidrarias? Pergunte ao seu assistente de laboratrio. A troca de cada tem ser penalizada com 1 ponto do total de 40 da prova prtica.
Isto no se refere gua destilada, gelo e papel toalha. No haver troca de manmetro,
equipamentos do Experimento 3 e nem de pen drive.
s Questes sobre: segurana, aparatos, reagentes, paradas para ir ao banheiro ou beber
gua: consulte o assistente de laboratrio.
s Descarte de reagentes lquidos: ponha-os apenas no frasco de 1L rotulado WASTE.
s A verso original em ingls est disponvel e pode ser requisitada para esclarecimentos somente. Pergunte a seu assistente de laboratrio.
I

Olimpada Brasileira de Qumica - 2015

131

47th IChO

Exame Prtico

s Depois do sinal de parar coloque o caderno no envelope (no lacre), e deixe sobre sua mesa.
s No saia do laboratrio at que seja dada a permisso pelo assistente de laboratrio.
s Voc deve encerrar o seu trabalho imediatamente aps o sinal de parada. Um atraso de 1 min resultar em zero pontos no Experimento em que estiver trabalhando.
s Durante o exame prtico, algumas vidrarias e utenslios de plstico sero usados
vrias vezes. Limpe-os cuidadosamente.
s No recomendamos a realizao simultnea dos Experimentos 1, 2 e 3. Faa um
Experimento de cada vez.

INSTRUES GERAIS
Nome

Estado

Concentrao

Quant.

Colocado em

Rotulado

Experimento 1
3-Metiltiofeno

Soluo em CCl4

4g/8 mL

4g

Frasco plstico,
30 mL

3-methylthiophene in CCl4

1-Bromo-2,5-pirrolidinediona (NBS)

Slido

7,3g

Frasco plstico,
30 mL

NBS
7.3 g

Tetracloreto de
carbono

Lquido

24 mL

Frasco plstico,
125 mL

CCl4

Catalisador desconhecido

em CCl4

Frasco plstico,
4 mL

Catalyst

Carbonato de potssio

Slido

Frasco plstico,
4 mL

K2CO3

Soluo contendo
VO2+ e Cr3+

Soluo aquosa

A ser
determinada

100 mL

Frasco plstico, 100 mL

Test solution

cido sulfrico

Soluo aquosa

1M

~ 500
mL

Frasco de vidro, 1000


mL

1M H2SO4

Permanganato de
potssio

Soluo aquosa

0,03 M

15 mL

Frasco plstico, 30 mL

0.03 M KMnO4
0.03 M H2C2O4

0,02 g

Experimento 2

cido oxlico

Soluo aquosa

0,03 M

30 mL

Frasco plstico, 50 mL

cido fenilantranlico

Soluo aquosa

0,1 %

5 mL

Conta-gotas, 6 mL

Indicator

Sulfato de ferro (II)


amoniacal

Soluo aquosa

Ver no rtulo

100 mL

Frasco de reagente,
100 mL

Mohrs salt

Nitrato de prata

Soluo aquosa

0,3 %

5 mL

Conta-gotas, 8 mL

0.3 % AgNO3

Persulfato de amnio

Soluo aquosa

10 %

70 mL

Frasco plstico, 100 mL

10 % (NH4)2S2O8

132

Olimpada Brasileira de Qumica - 2015

47th IChO

Exame Prtico

Experimento 3
Diclofenaco contido
no medicamento

Soluo aquosa

A ser determinado

5 mL

Frasco plstico, 30 mL

Control

Permanganato de
potssio

Soluo aquosa

6x10-3 M

~ 30 mL

Frasco de reagente,
100 mL

KMnO4
610-3 M

cido sulfrico (no


mesmo frasco do
Experimento 2)

Soluo aquosa

1M

~ 500
mL

Frasco de reagente com


tampa de vidro, 1L

1M H2SO4

Diclofenaco de sdio

Soluo aquosa

~ 600 mg/L

~ 20 mL

Frasco de reagente,
100 mL

DCF 600 mg/L

LISTA DE MATERIAL DE LABORTRIO E EQUIPAMENTOS


Item

Quantidade

Localizado

Na mesa para uso comum


Refratmetro Refracto 30GS

1-2 / 1 lab

Na capela

Papel-toalha para limpar o refratmetro cleaning

Na capela

Frasco de lavagem Cleaning solvent para o refratmetro

Na capela

Folha de alumnio para invlucro

1-2 rolls / 1 lab

Na mesa do assistente de lab.

1-3/ 1 lab

Em mesas separadas

Balanas
Luvas (S, M, L)

Na mesa do assistente de lab.

Frasco grande rotulado H2O dist.

Perto da pia

Papel toalha para diversos propsitos

1 Pacote / 1 row

Perto da pia

Item

Quantidade

Como marcado
nas Figs. 1, 2, 5

Em cada local de trabalho, para ser usado em mais de um Experimento


Placa aquecedora com agitao magntica

Frasco para descarte rotulado Waste

Luvas de algodo

1 par

Frasco lavador, 500 mL, rotulado H2O distilled

Pro-pipeta, 10 mL, verde

Pro-pipeta, 2 mL, azul

Proveta, 25,0 mL para H2SO4 somente

1
I

Olimpada Brasileira de Qumica - 2015

133

47th IChO

Exame Prtico

culos de seguranca

Papel toalha para diversos propsitos

1 pacote
Experimento 1

Suporte universal

Balo de trs bocas, 100 mL

6 (um deles marcado com seu cdigo


de estudante)

Funil de adio, 50 mL

Barra magnetica oval (grande)

para destilao, 50 mL

Adaptador de Claisen para destilao

Termmetro com junta fixada

Funil de placa porosa

10

Adaptador de borracha para filtrao a vcuo

11

Condensador de Liebig

12

Junta para destilao tipo teta de vaca

13

Balo receptor, 10 mL

4 (um deles marcado com seu cdigo


de estudante)

14

Balo receptor, 50 mL

15

Elevador ajustvel (macaco)

16

Barra magntica oval (pequena)

17

Bquer plstico, 50 mL, rotulado For the receiver with the


product

Fita de teflon para vedar juntas esmerilhadas

12

Funil grande, 65 mm, com haste curta

Grampo para junta

18

Garra cinza

19

Garra vermelha

20

Marcador permanente

Bquer de vidro, 25 mL

Container de plstico rotulado Used glassware

Container de plstico rotulado Ice bath

Condensador de refluxo, conectado ao suprimento de gua


Tampas (rolhas) de vidro esmerilhadas

Balo de fundo redondo, no formato de pera,

134

Olimpada Brasileira de Qumica - 2015

47th IChO

Exame Prtico

Manmetro digital

Chumao de algodo

Esptula

Basto de vidro

Rgua

Lpis

1
Experimento 2

Suporte universal

Garra para bureta

Bquer plstico, 100 mL, rotulado Waste

Bquer de vidro, 150 mL

Balo volumtrico com tampa, 100 mL

Funil pequeno, 45 mm

Funil mdio, 55 mm

Vidro de relgio

Bureta, 25,00 mL, fixada no suporte universal

Pipeta volumtrica, 10,00 mL

Pipeta graduada, 5,00 mL

Erlenmeyer, 150 mL

Proveta, 100,0 mL

Pipeta de Pasteur

Folha de papel branca

1
Task 3

Fotmetro, 525 nm

Termostato com adaptador

Clula para espectrofotmetro de 3,5 cm de caminho ptico

Agitador magntico

Barra de agitao magntica (tamanho mdio)

Notebook com adaptador e mouse

Balo volumtrico com tampa, 100 mL

Pipeta graduada, 2 mL

Pendrive de 8 Gb rotulado com o seu cdigo de estudante

Barra magntica preta

1
I

Olimpada Brasileira de Qumica - 2015

135

47th IChO

Exame Prtico

AJUSTANDO A SELETIVIDADE DA BROMAO


POR CATLISE (15 PONTOS).

EXPERIMENTO 1
Questo #

Q1

Q2

Q3

Q4

Q5

Q6

Total

pontuao

39

50

A seletividade das reaes qumicas um dos experimentos mais desafiadores da pesquisa contempornea. Em muitos casos, as condies reacionais
e os catalisadores utilizados so chaves para alcanar uma elevada seletiviO
3-metiltiofeno
pode,
teoricamente,
ser experimento,
transformado
em vai
quatro
derivados
monobrom
feno
pode,
teoricamente,
ser transformado
em
derivados
T1-T4
das reaes
orgnicas.
voc
estudar
um
desmetiltiofeno
pode,
teoricamente,
ser Neste
transformado
em quatro
derivados
monobromados
casos. Odade
3-metiltiofeno
pode,
teoricamente,
serquatro
transformado
emmonobromados
quatro
derivados
mon
ses casos. O 3-metiltiofeno pode, teoricamente, ser transformado em quatro
T1-T4
edos
os ndices
valores
dos
ndices
desintetizados
refrao
dados
rizados
em
detalhe.
devalores
T1-T4
e os
de
refrao
so
dados
naso
Tabela
1 na
detalhe.
Estruturas
deEstruturas
T1-T4
e osdevalores
de
refrao
so
Tabe
s em detalhe.
Estruturas
deEstruturas
T1-T4
e osndices
valores
dos
ndices
de dados
refrao
da
caracterizados
em monobromados
detalhe.
derivados
T1-T4,
que
ja dos
foram,
na verdade,
e na so
caracterizados em detalhe. Estruturas de T1-T4 e os valores dos ndices de
refrao so dados na Tabela 1

Tabela
1.e Estruturas
ndices
de tiofenos
refrao
de tiofenos
m
TabelaTabela
1. Estruturas
ndices
de erefrao
de
monobromad
1. Estruturas
e ndices
de refrao
de tiofenos
monobr
de tiof

e ndices de refrao
Tabela 1. Estruturas e ndices deTabela
refrao1.deEstruturas
tiofenos monobromados.

ao
A
Designao

Designao
Br
Br
Estrutura

Br

B
A
A

Br

T3 T3

BT3

T4
T3
T4

T3

T4 T4

Br

ra
Estrutura

nD20

S
S

Br

Br

BrS

Br
S
Br

Br
S
Br

S
Br

Br

Br

S S

Br
S

20
1,5961
1,5706 1,5706
1,5786 1,57861,5786
1,5795 1,57951,57951
1,5961
1,57061,57061,5706
1,57861,5786
1,5795
nD
1,59611,59611,5961

A sntese seletiva de cada um dos compostos T1-T4 pode ser realizada utili-

se
seletiva
deum
cada
dosum
compostos
T1-T4
pode
ser
realizada
utilizando
3-metiltiofen
e cada
dos
compostos
T1-T4
pode
ser
realizada
utilizando
3-metiltiofeno
como
mate
etiva
deum
cada
dos
compostos
pode
ser
realizada
utilizando
3-metiltiofeno
com
A
sntese
seletiva
deum
cada
dosT1-T4
compostos
T1-T4
pode
ser
realizada
utilizando
3-meti
zando
3-metiltiofeno
como
material
de partida.
T1
e T2
podem
ser
obtidos
por bromao
direta
utilizando
diferentes
enquanto
T3
e sntese
T4 em uma
mao
direta
utilizando
diferentes
catalisadores,
enquanto
T3
e T4
so
obtidos
sp
utilizando
diferentes
catalisadores,
enquanto
T3catalisadores,
e T4
obtidos
em
"one
oapor
direta
utilizando
diferentes
catalisadores,
enquanto
T3so
e T4
so
obtidos
em
sntese
bromao
direta
utilizando
diferentes
catalisadores,
enquanto
T3
e uma
T4
souma
obtidos
em
so obtidos em uma sntese one pot de passos mltiplos (ver Esquema 1).
T2

CCl4 CCl4NBS, CClNBS, CCl4


NBS, CCl4NBS, CClNBS, NBS,
4
4
NBS, CCl4
NBS, CCl4
T1
T2
T1
T1
T2
T2 HClO
cat. 4 cat.
AIBN cat. AIBN cat.AIBN
HClO4 cat.
HClO
cat.
AIBN cat. 4
HClO cat.

136

T4

Olimpada Brasileira de Qumica - 2015

1. 3.5
eq. Br2,
, eq. Br
1. 3.5 eq. Br
1.23.5
S2,
1. 3.5 eq. S
Br2,
S
1. BuLi, S
TMEDA,
NaOAc, NaOAc, NaOAc,
1. BuLi, TMEDA,
1.
BuLi, TMEDA,
NaOAc,
1. BuLi, TMEDA,
H
O,
100
C
H2O, 100 C
Et
O,
rt
2
2
H2O, 100 C
Et2C
O, rt Et2O, rt Et O, rt
H2O, 100
2 T3
T4
T3
T3
T4
Zn
dust
2. Zn dust 2. Zn dust2.T4
I
2. CBr4
2. CBr
4 2. 2.
ZnCBr
dust4
2. CBr
4

961

1,5706

1,5786

1,5795

Exame T1-T4
Prtico pode ser realizada utilizando 3-metiltiofeno
m dos compostos
como material de pa
47th IChO

o diferentes catalisadores, enquanto T3 e T4 so obtidos em uma sntese "one pot" de pas


T2

T4

NBS, CCl4

NBS, CCl4

AIBN cat.

HClO4 cat.

1. 3.5 eq. Br2,


NaOAc,
H2O, 100 C

2. Zn dust

T1

1. BuLi, TMEDA,
Et2O, rt
2. CBr4

T3

O
NBS =

N Br

AIBN = NC

N N

CN

O
TMEDA =

Scheme 1. Sntese seletiva de tiofenos monobromados.

Q1. Associe os compostos indicados no Esquema 1 como T1 e T2 s estruturas dadas na Tabela 1. Preencha cada caixa abaixo como A ou B.
T2

T1

Neste Experimento, voc ir:


-

Sintetizar um derivado de tiofeno monobromado usando um dos catalisadores a partir da lista abaixo;

Medir o ndice de refrao do produto (nD)

Comparar os resultados obtidos com os dados da literatura e decidir sobre a estrutura do produto e dos catalisadores dados.

Lista de possveis catalisadores


-

HClO4 em CCl4

AIBN em CCl4
I

Olimpada Brasileira de Qumica - 2015

137

47th IChO

Exame Prtico

PROCEDIMENTO
Nota!
s!PPARATOSUSADOSNESTAQUESTOESTOMOSTRADOSNAS&IGSE
3EMPREUSEAlTADETEmONEMTODASASJUNTAS#OLOQUEIMEDIAMENTENOCORRESPONDENTE
CONTAINER TODASASPEASDEVIDROUTILIZADAS
s6OCDEVEUSARASLUVASDEALGODOQUANDOMANUSEARCOISASQUENTES

Passo 1. Prenda o balo de 3 bocas no suporte universal e sobre a placa de


aquecimento com agitador magntico (Fig.1). Inserir o funil de adio e o
condensador de refluxo nas bocas correspondentes e coloque a barra magntica oval dentro do balo atravs da boca aberta. Pea a seu assistente
de laboratrio que ligue o fluxo de gua no condensador de refluxo (No
faa isso sozinho!). Transfira quantitativamente o NBS para o balo usando
esptula e o funil de plstico grande. Transfira ~15 mL de CCl4 para o bquer
de vidro de 25 mL. Coloque ~2/3 deste volume de solvente do bquer no
balo. Agite o catalisador e o adicione quantitativamente usando o mesmo
funil de plstico no balo. Adicione ao balo o resto do solvente que est no
bquer. Feche a boca do balo com a tampa. Coloque o balo no recipiente
de plstico para banho de gelo preenchido com gua e gelo em ~ 2/3 do
seu volume. Comece a agitar a mistura.

Fig. 1. Aparato necessrio para realizar os Passos 1-4 da


sntese. Os nmeros se referem lista das pginas 4 e 5

Passo 2. Usando o funil de plstico grande, transfira quantitativamente a soluo de 3-metiltiofeno para o funil de adio com a torneira fechada. Passe
um pedao de algodo na abertura final do funil de adio e do condensador de refluxo. Sob agitao vigorosa, adicione a soluo de 3-metiltiofeno
gota a gota durante ~ 3 min. Substitua o funil de adio pela tampa de vidro.

138

Olimpada Brasileira de Qumica - 2015

47th IChO

Exame Prtico

Use a fita de Teflon. Remova o banho de gelo. Seque o placa de aquecimento


e a parte inferior do balo com o papel toalha.
Passo 3. Envolva o balo com uma folha de alumnio. Ligue o aquecedor
(posio 3). Aquea a mistura at a fervura e mantenha fervendo por 10 min.
Prepare o banho de gelo (~2/3 do volume) enquanto a mistura ferve.
Passo 4. Desligue o aquecedor e remova cuidadosamente (quente!) a placa
de aquecimento com agitao magntica. Mergulhe o balo conectado ao
condensador e tampas no banho de gelo por 3-5 min. Agite suavemente o
balo para que o resfriamente seja mais rpido. Ento remova o condensador de refluxo e coloque 0,02 g de K2CO3, usando o funil grande, atravs
da boca aberta. Feche a boca com uma tampa e agite o balo vrias vezes.
Desligue o fluxo de gua e desconecte as mangueiras do condensador de
refluxo. Deixe a gua residual sair e coloque-o dentro do container rotulado
used glassware. Remova a garra mantendo o balo no banho de gelo.
Passo 5. Pese o balo receptor de 10 mL para o produto com a tampa de
vidro, ambos marcados com o seu cdigo de estudante. Escreva o valor na
tabela de respostas. Coloque o agitador magntico pequeno no balo de
destilao com formato de pera de 50 mL. Encaixe as mangueiras nos bicos
de entrada e sada do condensador de Liebig e fixe-o no suporte com a garra vermelha. Ligue voc mesmo, o fluxo de gua e certifique-se se no h
vazamento.
Passo 6. Monte o aparato de destilao como mostrado na Fig. 2 suprindo as
juntas com a fita de teflon e grampos. Primeiro, encaixe dois frascos receptores de 10 mL e um de 50 mL na junta tipo teta de vaca para destilao. Em seguida, conecte a trompa de vcuo na teta de vaca e complete a montagem.
Fixe o aparato sobre o agitador magntico e ajuste a altura. Use o elevador
ajustvel (macaco) como suporte.

Fig. 2. Aparato necessrio para realizar os Passos 5-10 da


sntese. Os nmeros se referem lista das pginas 4 e 5
I

Olimpada Brasileira de Qumica - 2015

139

47th IChO

Exame Prtico

Passo 7. Remova a placa de aquecimento com agitao magntica. Insira


o funil de placa porosa no adaptador de Claisen para destilao usando o
adaptador de borracha para vcuo. Ligue a trompa de vcuo e tambm o
manmetro digital. Remova o balo de 3 bocas do banho de gelo e seque-o
com o papel toalha. Transfira, cuidadosamente, a mistura reacional do balo
de 3 bocas para o filtro (Ateno! Se voc fizer muito rpido, a mistura pode
ser parcialmente sugada para dentro da parte curva do adaptador). Quando
terminar, desligue a trompa e substitua o filtro por uma tampa de vidro e use
a fita de teflon.
Passo 8. Envolva firmemente o balo e o adaptador para destilao com folha de papel alumnio at a junta do termmetro. Traga de volta o agitador
magntico e ligue a agitao e aquecimento (posio 6). No ligue a trompa
vcuo! Colete o solvente destilado no balo receptor de 50 mL . Quando
o solvente destilado acabar, desligue o aquecimento e a agitao e, cuidadosamente (quente!), remova a placa de aquecimento com agitao magntica. Substitua o balo receptor por um novo de 10 mL. Feche o balo de
50 mL com uma tampa de vidro e entregue para o assistente de laboratrio.
Passo 9. Remova a folha de alumnio e ponha o balo de fundo redondo no
banho de gelo por 2 a 3 min para que a temperatura fique menor que a ambiente. Remova o banho de gelo; seque o balo com uma toalha de papel.
Traga de volta o agitador magntico sob o balo (Ateno! A placa ainda
pode estar quente!). Ligue a agitao. Envolva o balo firmemente com uma
folha de alumnio. Ligue a trompa de vcuo. Quando o vcuo estiver estabilizado (siga a leitura do manmetro digital), ligue o aquecimento (posio 6).
Observe o estgio inicial da destilao do produto desejado e colete as primeiras 3-5 gotas em um balo receptor conectado no rotulado com o seu
cdigo de estudante. Ento gire a junta tipo teta de vaca e colete o produto
desejado no balo receptor com o seu cdigo de estudante. Escreva a temperatura de ebulio do produto e a presso indicada no manmetro digital
na tabela de resposta.
Passo 10. Quando o produto desejado coletado, desligue o aquecimento,
remova a folha e, cuidadosamente (quente!), remova a placa de aquecimento com agitao magntica. Resfrie o aparato para a temperatura ambiente
usando o banho de gelo. Pea ao seu assistente de laboratrio para desconectar a linha de vcuo. Desconecte o frasco receptor com o produto desejado e, imediatamente, feche-o com a tampa de vidro rotulada com o seu
cdigo de estudante. No tente arrancar a fita de teflon se ela permanecer
no balo receptor. Coloque o balo no bquer de plstico de 50 mL rotulado

140

Olimpada Brasileira de Qumica - 2015

47th IChO

Exame Prtico

For the receiver with the product. Imediatamente encaixe um novo receptor
no lugar do removido e coloque o grampo para junta. Deixe o aparato do
jeito que ele est.
Passo 11. Mea o ndice de refrao (antes de pesar) seguindo as instrues
abaixo. Anote a temperatura do refratmetro.
Pese o receptor com o produto desejado fechado com a tampa rotulada. Calcule a massa e o rendimento do produto (considere a massa da fita de teflon
igual a 149 mg). As massas molares do 3-metiltiofeno e do produto igual a
98 e 177 g.mol-1, respectivamente.

Q1. Escreva todos os resultados na Tabela abaixo.


#

Parmetro /Caractersticas

Valor

Unidades

Massa do balo receptor com a tampa de vidro, ambos


marcados com o cdigo do estudante

Massa do produto

Rendimento do produto

ndice de refrao para o produto

Temperatura anotada do refratmetro

Temperatura de ebulio do produto

Presso no ponto de ebulio

mmHg

Entregue o produto para o assistente de laboratrio e pegue a sua assinatura.


O produto desejado entregue: ________________________________________

Assinatura do estudante

Assinatura do assistente do laboratrio

Olimpada Brasileira de Qumica - 2015

141

47th IChO

Exame Prtico

REFRACTO 30GS INSTRUES DE OPERAO

Fig. 3. Usando o Refracto 30GS

1. Para ligar o Refratmetro 30GS , pressione e segure o boto ESC (1) at o display aparecer.
O instrumento est pronto para operar. Ele desligar automaticamente se no for operado em
10 min.
2. Limpe a clula e o basto de vidro com o papel toalha molhado com solvente do frasco de lavagem rotulado cleaning solvent. Seque ambos com outro papel toalha .
3. Tenha certeza que a amostra a ser medida alcanou a temperatura ambiente e est homognea.
4. Adicione 2-3 gotas da amostra dentro da clula (2) usando o basto de vidro.
5. Para iniciar a medida, pressione e segure o boto OK (3) at o bip.
6. Anote o valor do ndice de refrao e a temperatura do display digital (4) e escreva abaixo o
resultado na tabela de respostas.
7. Limpe a clula e o basto de vidro.

Q3. Compare os dados obtidos com os dados da literatura e desenhe as


estruturas do produto e do catalisador dado.
Produto obtido

142

Olimpada Brasileira de Qumica - 2015

Catalisador dado

47th IChO

Exame Prtico

Q4. Desenhe a estrutura dos intermedirios derivados do 3-metiltiofeno,


baseado na seletividade que leva aos produtos T1 e T2.
T1

T2

Q5. Escreva abaixo o produto (T1 ou T2) formado como resultado da bromao direta do 3-metiltiofeno com NBS, conforme as condicoes dadas /
catalisador usado.
ZnBr2
Perxido de dibenzoila
LiBr em AcOH
Luz visvel ou UV
Q6. Nas rotas sintticas para T3 e T4, desenhe as estruturas dos compostos formados na primeira etapa de cada rota mostrada no Esquema 1.
T3

T4

ANLISE DA SOLUO DE UMA LIGA DE CROMO


VANDIO (12 PONTOS)

EXPERIMENTO 2
Questo #

Q1

Q2

Q3a

Q3b

Q4a

Q4b

Q5a

Q5b

Q6

Total

pontuao

32

32

10

90

Materiais antiferromagnticos mostram ser bons no desenvolvimento de dispositivos de memria de alta-densidade para armazenamento de dados. O
menor dispositivo de memria bit do mundo utiliza apenas 12 tomos e um

Olimpada Brasileira de Qumica - 2015

143

47th IChO

Exame Prtico

dos exemplos mais destacados. A liga cromo-vandio, exibe propriedades


antiferromagnticas mesmo a temperaturas abaixo de zero. bvio que a
composio destas ligas, que so usadas em tecnologias de ponta devem
ser devidamente controladas.
Neste experimento voc ir analisar uma soluo aquosa que simula o produto de digesto de uma liga cromo-vandio. O experimento composto
pelas seguintes partes:
I.

Oxidao de vanadil (VO2+) a vanadato (VO3-) na soluo teste usando


permanganato de potssio, seguida pela determinao do vandio (note
que o cromo(III) no oxidado nestas condies).

II. Oxidao da soluo teste com persulfato de amnio seguida de titulao com o sal de Mohrs (sulfato de ferro(II) amoniacal) para determinar o
contedo total de vandio e cromo.
PROCEDIMENTO
Nota!
s !QUANTIDADEDEVANDIOECROMODEVERSERCALCULADAEREGISTADAEMMGPORM,DE
SOLUOTESTE
s )NICIEESTEEXPERIMENTOPELAPARTE!UMAVEZQUEVOCIRNECESSITARDETEMPOPARA
OXIDARASOLUOTESTEASERANALISADANAPARTE#
s !PIPETAVOLUMTRICADE  M,APRESENTATRAOSDEGRADUAO$EVEPIPETAROVOLU
MEENTREOSTRAOS

144

Olimpada Brasileira de Qumica - 2015

47th IChO

Exame Prtico

Parte A - Preparao da soluo para a determinao


do contedo total de vandio e cromo
1. Transfira 10,00-mL da soluo teste para o bquer de 150 mL e adicione
20 mL da soluo de cido sulfrico 1 M (utilize a proveta de 25 mL).
2. Adicione 6 a 8 gotas da soluo de nitrato de prata a 0,3% (o catalisador)
e usando a placa de aquecimento (posio 3), aquea a mistura a 70-80
C at aparecer condensao nas paredes do bquer.
3. Usando a proveta de 100 mL, adicione 20 mL da soluo de persulfato de
amnio a 10% mistura aquecida.
4. Continue o aquecimento at observar o aparecimento da cor amarela,
indicativa da formao do dicromato.
Nota! 0ODEFAZERADETERMINAODOVANDIO0ARTE" A ENQUANTOESTAMISTURA
ESTSENDOAQUECIDA
5. Aps o aparecimento da cor amarela, mantenha o aquecimento da mistura durante 10-15 min (posio 3), para decompor o excesso de persulfato de amnio (a decomposio estar terminada quando deixar de se
observar pequenas bolhas na soluo).
6. Deixe resfriar a soluo at temperatura ambiente.
7. Transfira quantitativamente a soluo do bquer de 150 mL para o balo
volumtrico de 100 mL. Ajuste o volume at a marca com gua destilada,
coloque a rolha e agite convenientemente a soluo.

Parte B - Determinao volumtrica do Vandio


1. Transfira com a pipeta graduada uma alquota de 5,00-mL da soluo
teste para o Erlenmeyer.
2. Cuidadosamente e gota a gota adicione a soluo de permanganato de
potssio 0,03 M. Agite, a cada gota adicionada, at aparecer uma colorao rosa. Certifique-se de que a colorao rosa persistente. Remova
o excesso de permanganato de potssio adicionando gota a gota uma
soluo de cido oxlico 0,03 M. Agite o Erlenmeyer aps a adio de

Olimpada Brasileira de Qumica - 2015

145

47th IChO

Exame Prtico

cada gota at a colorao rosa mudar para azul claro. Deixe a soluo
repousar durante 1 min para garantir que a colorao rosa desapareceu
completamente.
3. Adicione ao Erlenmeyer 10 mL da soluo de H2SO4 1M, usando a proveta de 25 mL.
4. Adicione ao Erlenmeyer 2 a 3 gotas (no mais!) de indicador e agite vigorosamente. Deixe o Erlenmeyer repousar por 2 a 3 min e observe o
aparecimento da colorao prpura.
5. Encha a bureta com a soluo de sal de Mohr. Utilize o bquer de plstico
de 100 mL etiquetado Waste para retirar o excesso de soluo de sal de
Mohr da bureta. Anote o volume inicial.
6. Titule a soluo que se encontra no Erlenmeyer com a soluo de sal de
Mohr at visualizar a mudana de cor para castanho acinzentado e em
seguida para um puro verde claro.
7. Anote o volume final. Repita quantas vezes necessrias.

Q1. Preencha a Tabela 2.


Tabela 2. Determinao de Vandio
Titulao N
Leitura inicial, mL
Leitura final, mL
Volume gasto, mL
Volume aceito, V1 _____mL

Parte C - Determinao volumtrica do contedo total


de vandio e cromo na soluo teste
1. Lave a pipeta volumtrica de 10,00 mL com guas destilada, e em seguida leve-a com a soluo contida no balo volumtrico de 100 mL (obtida
na parte A).
2. Pipete para um Erlenmeyer 10,00 mL desta soluo e adicione 10 mL de
soluo H2SO4 1 M usando a proveta de 25 mL.

146

Olimpada Brasileira de Qumica - 2015

47th IChO

Exame Prtico

3. Adicione 3 a 4 gotas do indicador. Agite vigorosamente o Erlenmeyer e


deixe repousar por 3 a 4 minutos. Observe o aparecimento de uma cor
vermelha.
4. Encha a bureta com a soluo de sal de Mohr. Utilize o bquer de plstico
de 100 mL etiquetado Waste para retirar o excesso de soluo de sal de
Mohr da bureta. Anote o volume inicial.
5. Titule a soluo que se encontra no Erlenmeyer com a soluo de sal de
Mohr at visualizar a mudana de cor para uma cor amarelo-esverdeada
clara.
6. Anote o volume final. Repita quantas vezes forem necessrias.

Q2. Preencha a Tabela 3.


Tabela 3. Determinao do contedo total de vandio e cromo
Titulao No
Leitura inicial, mL
Leitura final, mL
Volume gasto, mL
Volume aceito, V2 _____mL

Parte D - Questes e anlise de dados


Q3. Escreva as equaes qumicas devidamente balanceadas das reaes
que ocorreram na:
a) Oxidao da soluo teste com a soluo de permanganato de potssio
b) Titulao do vanadato com o sal de Mohr
a)
b)

VOSO4

Olimpada Brasileira de Qumica - 2015

147

47th IChO

Exame Prtico

Q4. Escreva as equaes qumicas devidamente balanceadas das reaes


que ocorreram na:
a) Oxidao da soluo teste com a soluo de persulfato de amnio
b) Titulao do soluo teste com o sal de Mohr.

Q5. Calcule as concentraes na soluo teste de a) V(IV) e b) Cr(III). Calcule a quantidade dos metais em mg por 100 mL de soluo teste.

Q6. Este procedimento no pode ser aplicado para a determinao de vandio e cromo em ao, se este for digerido com HCl concentrado. Escreva
as equaes de duas reaes que explicam a afirmao anterior.

EXPERIMENTO 3

DETERMINAO CINTICA DO DICLOFENACO (DCF) (13 PONTOS)

Questo #

Q1

Curvas DCF

Controle DCF

Ordem de Reao

pontuao

10

40

20

10

Total
80

A utilizao de espectrofotometria para determinao de princpios ativos


por mtodos cinticos tem sido largamente desenvolvida na ltima dcada
devido ao grande nmero de vantagens, incluindo a simplicidade de utilizao, o baixo custo, a disponibilidade na maioria dos laboratrios de controle de qualidade e a sua maior seletividade. Neste experimento pretende-se
que:
s Determine cineticamente a quantidade de Diclofenaco (DCF) presente no
medicamento, seguindo o progresso da sua reao de oxidao.
s Determine a ordem de reao em relao ao DCF.

148

Olimpada Brasileira de Qumica - 2015

47th IChO

Exame Prtico

Q1. As alteraes no espectro durante a oxidao do DCF com o KMnO4


esto representadas na figura 4 (1 a 10 reflete o progresso da reao).
Na tabela representada abaixo, indique dos comprimentos de onda os
que acha poderem ser aplicados na determinao cintica fotomtrica do
DCF. Em cada caso, indique a direo da alterao que a absorvncia sofre
(se aumentar use e se diminuir use ).

Fig. 4. Oxidao do DCF com KMnO4

#
1
2
3
4
5

Comprimento de onda, nm
420
480
520
580
610

Sim (Yes) ou No (No) e direo da alterao

Olimpada Brasileira de Qumica - 2015

149

47th IChO

Exame Prtico

Procedimento
Parte A. Montagem do equipamento
Monte o equipamento de acordo com o que mostrado na Fig. 5. Ligue o
fotmetro (1), (comprimento de onda fixo, 525 nm) e o termostato (2) ao notebook via porta USB. Ligue o termostato ao cabo etiquetado Thermo e em
seguida ligue este atravs do adaptador de corrente a uma das tomadas que
tem na sua bancada. Coloque a clula (3) em cima da agitador magntico (4),
faa deslizar a clula atravs do fotmetro (no possvel coloc-la de outra
maneira) e coloque o termostato em cima da clula (ver Fig. 5b).

Fig. 5. Equipamento

Sugestes!
s ,IGUEtodos os cabos ao computador porttil antes de o ligar
s -ONTEe ligue todo o equipamento laboratorial (fotmetro e termostato)
antes de ligar no notebook. Ligue tambm o mouse usando o receptor
que est dentro do compartimento da pilha).
s 1UANDOabrir o programa s abrir uma janela (mais adiante designada
por Pattern) em vez de duas, feche e volte a abrir o programa.
s .Odesligue NENHUM dispositivo da porta USB enquanto estiver efe-

150

Olimpada Brasileira de Qumica - 2015

47th IChO

Exame Prtico

tuando a experincia. Se isso acontecer surge um aviso na tela. Feche e


volte a abrir o programa.
s 3Eo notebook desligar automaticamente dos dispositivos, clique no boto Setup existente na janela de leituras da absorvncia (Measurements
window on the absorbance plot pattern).
s Se voc observar mudanas caticas de tempetura, pare e reinicie as medidas.

Parte B. Registro da curva de calibrao


Todas as leituras necessria para registrar a curva de calibrao devem ser
efetuadas a 30 C com concentraes iniciais de KMnO4 e H2SO4 constantes.
A concentrao do DCF variada usando 4 alquotas diferentes (de 0,2, 0,4,
0,6 e 0,8 mL) da soluo de DCF fornecida.
1) Transfira para o balo volumtrico de 100 mL, 5 mL da soluo de H2SO4
1 M usando a proveta e 0,2 mL da soluo de DCF fornecida usando a pipeta graduada de 2 mL. Acerte o volume at marca com gua destilada,
tampe o balo e agite bem.
2) Coloque o contedo do balo na clula juntamente com a barra magntica de tamanho mdio e ligue a agitao magntica. Regule a agitao
de acordo com a posio mostrada na Fig.5a para ter uma boa agitao.
3) Abra o programa Chemistry-Practicum disponvel no notebook. O software detectar automaticamente os dispositivos externos ligados (sensores). Voc ver o aparecimento n tela de dois grficos (o da absorbncia/
extino/densidade ptica, D versus t, s ; e o da temperatura, T C versus
t, s ).
4) Selecione os seguintes parmetros na barra Menu do grfico correspondente (Fig. 6):
s .Ogrfico da absorbncia click no boto
que est prximo do boto
, para fixar o mximo do eixo do X como sendo a tela inteira. Usualmente os grficos ajustam-se tela.
(seleciona a varias !INDAno grfico da absorvncia, click no boto
o de Y) e ajuste a variao da absorbncia (no eixo das ordenadas)
entre -0.1 e 1.1.
I

Olimpada Brasileira de Qumica - 2015

151

47th IChO

Exame Prtico

s %SCREVA2 (em vez de 1) na caixa de intervalo de leituras no grfico da


absorbncia.
s .A janela do grfico da temperatura selecione Precisely em vez de
Roughly e em seguida click no boto T = X e selecione a temperatura
de 30 C.
s 5TILIZE o boto Setup na janela de leitura do grfico da absorbncia
para calibrar o fotmetro.

Fig. 6. Interface do software Chemistry-Practicum

Nota! A escolha dos parmetros anteriores (passo 4) s necessria antes


da primeira leitura.
5) Para ligar o termostato e observar o aquecimento da soluo na clula pela
(Start measure for chosen sensors). Controlmpada, click no boto
le a temperatura reportada na linha acima do grfico. Espere at que a
lmpada do termostato se desligue, indicativo de que a temperatura selecionada foi atingida. Pare as leituras clicando no boto
(est ativado e
adquire a cor laranja avermelhado quando a leitura est sendo feita).
6) Para ativar o grfico da absorbncia click em qualquer ponto do mesmo.
Utilizando a pipeta de 2 mL, encha-a com 2 mL da soluo KMnO4. Na barra
de menu da janela de leituras, click no boto
(Start measure for chosen
sensors) e rapidamente despeje a soluo de permanganato na clula.

152

Olimpada Brasileira de Qumica - 2015

47th IChO

Exame Prtico

Nota! Garanta que antes de adicionar a soluo de KMnO4 a temperatura


da clula de 30 C!
7) Observe na tela o progresso da curva cintica. Continue as leituras durante 50 s aps a adio da soluo de KMnO4, e termine as leituras pressioStop measurements.
nando o boto
(Export all the data collected
8) Guarde os dados pressionando o boto
in an external file) na barra de menu da janela de leituras do grfico de
absorbncia, selecione gravar no Desktop com o nome DCF2 (nas experincias seguintes grave os arquivos com o nome DCF4, ou DCF6,
ou DCF8).
Note! Utilize apenas os nomes no formato sugerido!
s Grave sempre os dados no Desktop antes de iniciar a prxima experincia, uma vez que os dados sero perdidos assim que pressionar o
boto
s #ERTIlQUE SEque o grfico da absorbncia est ativo quando exportar os
dados. De outra forma estar a exportar resultados invlidos. Se nenhum
grfico estiver selecionado surgir uma mensagem de aviso.
9) Despeje o contedo da clula para o frasco etiquetado Waste, e lave
bem a clula com gua destilada. Utilize o man preto do lado de fora
da clula para impedir que a sua barra magntica se perca durante as
lavagens da clula. Limpe cuidadosamente com o papel toalha as superfcies externas da clula bem como as gotculas presentes na lmpada do
termostato.
10) Repita os passos 1), 2, e de 5) a 9) para outros volumes de soluo fornecida de DCF.
Parte C
1. Determinao da quantidade de DCF no medicamento (Control)
1) Lave bem o balo volumtrico e prepare a mistura como anteriormente descrita, mas usando agora 0,4 mL da soluo de medicamento (Control) em vez da soluo fornecida de DCF.
2) Repita os passos 1), 2), e de 5) a 9) descritos na Parte B. Salve os
arquivos com os dados desta experincia com o nome DCFmed.
3) Repita as leituras da soluo de medicamento Control se necessrio.
I

Olimpada Brasileira de Qumica - 2015

153

47th IChO

Exame Prtico

2. Anlise dos dados experimentais


1) Abra o arquivo Excel existente no seu pen drive. Com duplo click abra
um a um os arquivos que gravou no Desktop (usando o Notepad).
Selecione Edit/Select All na barra Menu, e com o boto direito copie
os dados selecionados e na folha Excel correspondente ao mesmo
nome (DCF e volume adicionado ou DCFmed) escolhendo Edit/
Paste na barra Menu ver colado na folha Excel os resultados experimentais (time, s, na coluna A, e absorbance na coluna B).
2) Ignore os valores antes do mximo. Selecione as colunas A e B, e desenhe o grfico. Utilize para tal o cone Insert Scatter mostrado na
Fig. 7.

Figure 7. Posio do cone Insert Scatter

3) Selecione a seo linear inicial curva remanescente (15 a 20 pontos experimentais), aplique a aproximao linear adicionando a curva de tendncia linear e mostre esses parmetros na rea do grfico. Certifique-se
que o valor de R2 superior a 0,98. Se necessrio diminua o nmero de
dados experimentais da parte final da experincia.. Mesmo assim tente
garantir o maior nmero de dados experimentais para a obteno do
valor adequado de R2. Determine o valor da velocidade inicial, v0, para a
variao da absorbncia.
Nota! Ter zero pontos se usar para obter a reta menos de 12 pontos experimentais..
4) Analise do mesmo modo todas as outras experincias feitas com solues de DCF e da soluo do medicamento Control (DCFmed file).
5) Calcule as concentraes de DCF (em mg/L) nas solues que preparou
e registre na folha Excel Results, nas clulas apropriadas, as concentraes de DCF e as respectivas velocidades iniciais.

154

Olimpada Brasileira de Qumica - 2015

47th IChO

Exame Prtico

6) Insira o grfico com a reta de calibrao na folha Results e utilize-o para


determinar a concentrao de DCF presente na soluo de medicamento analisada (Control). Preencha nas clulas apropriadas da folha Excel
Results o coeficiente de linearidade da reta de calibrao. Calcule a
concentrao de DCF presente na soluo do medicamento.
7) Escreva na clula F10 da folha Excel Results o valor encontrado.
8) Na folha Excel Results determine graficamente a ordem de reao em
relao ao DCF e escreva na clula I3 exatamente o valor obtido.
9) Assim que terminar, salve o seu arquivo e chame o assistente de laboratrio para lhe mostrar que gravou os dados experimentais no arquivo Excel.
Assine e pea a assinatura ao assistente de laboratrio.
Nota! Somente os dados guardados na pen drive sero considerados para a
avaliao deste experimento.
Os arquivos com os dados Excel encontram-se no pen drive (deve ser assinado pelo assistente de laboratrio)
Sim

No

Estudante

Assistente de Laboratrio

SUBSTITUIES COM PENALIDADES


tem

Quant.

Assinatura do
estudante

Assinatura do assistente de
Laboratrio

"V firme na direo das suas metas. Porque o pensamento cria, o


desejo atrai e a f realiza."
(Desconhecido)
I

Olimpada Brasileira de Qumica - 2015

155

Iberoamericana

Exame Experimental

XX Olimpada
Iberoamericana
de Qumica
04 a 14 de setembro de 2015
Teresina - Brasil
EXAME EXPERIMENTAL
7 de setembro de 2015
Instrues gerais para o exame prtico (TOTAL DE PONTOS = 40)
1. Durante todo o tempo em que estiver no laboratrio, voc dever estar equipado com a
bata e culos de segurana que lhe so fornecidos. Voc pode usar a sua bata e os seus
culos se lhe derem autorizao. Sugere-se a utilizao de luvas, quando necessrio.
2. Espera-se que os estudantes trabalhem de forma segura e socivel e que conservem limpo
o material e a bancada de trabalho. Pergunte ao assistente de laboratrio se tiver alguma
dvida relativa s normas de segurana.
3. Este exame prtico tem a durao de 4:30 horas e constitudo por 2 problemas experimentais (cada um valendo 20 pontos). Voc pode realizar os problemas prticos na ordem que desejar.
4. Antes de iniciar o seu trabalho experimental leia cuidadosamente todo o exame prtico e
analise os espaos para respostas. Identifique onde est todo o material que vai utilizar.
Voc tem 30 minutos para ler o exame e planejar o trabalho experimental.
5. Comece a prova quando for dado o sinal de incio.
6. Voc tem 4 horas para executar todo o exame e registrar os seus resultados nos espaos
reservados para respostas.
7. Para medir com as pipetas voc dever utilizar unicamente a pra de borracha.
8. proibido comer no laboratrio. Voc pode pedir autorizao ao assistente de laboratrio
para ir ao banheiro ou para beber gua.
9. Escreva todas as suas respostas com caneta esferogrfica (No use o lpis).
10. No cabealho de cada folha de respostas escreva o seu nome e o seu cdigo de identificao (este est indicado no seu local de trabalho).
11. Voc s poder usar o material que lhe for fornecido e a sua calculadora.

156

Olimpada Brasileira de Qumica - 2015

Iberoamericana

Exame Experimental

12. Em caso de necessitar de mais reagentes ou de repor algum material de vidro ou outro
dirija-se ao assistente de laboratrio. Isto ser penalizado com 1 (um) ponto por cada
solicitao.
13. O nmero de algarismos significativos nas respostas numricas deve estar de acordo
com o erro experimental.
14. Voc dever escrever todas as suas respostas nos espaos reservados para esse fim nas
folhas de respostas. O que for escrito fora desses espaos no ser qualificado. Tambm
no escreva nada na parte de trs das folhas. Se necessitar de folhas de rascunho para
clculos ou substituir alguma(s) folha(s) de resposta por outra(s) solicite ao assistente de
laboratrio.
15. Ser dado um aviso 15 minutos antes do final do tempo previsto para a prova.
16. Quando ouvir o sinal de terminar, voc dever parar de trabalhar imediatamente, caso
contrrio, o problema que estiver fazendo ser anulado.
17. Quando terminar o exame voc deve colocar todas as folhas no envelope que recebeu
e somente dever fech-lo na presena do assistente de laboratrio que lhe entregar
um recibo. S as folhas que estiverem dentro do envelope fechado sero corrigidas e
classificadas.
18. Voc no pode sair do laboratrio sem que lhe seja dada autorizao.
19. imprescindvel que entregue o enunciado do exame com o seu nome e o seu cdigo.

Resduos qumicos e material de vidro.


1. Os resduos qumicos devem ser colocados nos recipientes apropriados.
2. O material de vidro quebrado deve ser colocado no recipiente para descarte de vidro.
Limpeza.
1. Ao terminar deixe o seu local de trabalho limpo e arrumado.
SOLUES DISPONVEIS
01
02
03
04

Acido clordrico ~ 0,1 mol/L ( HCl)


Acido clordrico 1 M (HCl)
Hidrxido de sdio ~ 0,1M (NaOH)
Hidrxido de sdio 1M (NaOH)

250 mL
500ml
250ml
250ml
I

Olimpada Brasileira de Qumica - 2015

157

Iberoamericana
05
06
07
08
09

Exame Experimental

Fenolftalena
Verde de bromo cresol
4 tubos Falcon com NaOH(s)
1 tudo Falcon com amostra desconhecida
Cloreto de brio 10%

5ml
5ml
50ml

MATERIAL DISPONVEL NA BANCADA (1 kit completo para cada estudante)


ITENS
01
02
03
04
05
06
07
08
09
10
11
12
13
14
15
16
17
18
19

MATERIAL
Erlenmeyer de 250 mL
Erlenmeyer de 125 mL
Balo volumtrico de 250 mL
Becker de 50 mL
Funil analtico de 30 mL
Funil analtico de 50 mL
Pipeta volumtrica de 10 mL
Pipeta volumtrica de 25 mL
Pipeta volumtrica de 50 mL
Proveta de 100 mL
Basto de vidro
Bureta de 50 mL
Suporte universal
Garra dupla tipo borboleta
Pipetador automtico (pera)
Garrafa trmica (calormetro)
Termmetro
Pisseta para gua destilada
Conta gotas

QUANTIDADE
03
03
01
02
01
01
01
01
01
02
01
01
01
01
01
01
01
01
02

Experimento 1: Verificao experimental da Lei de Hess


A Termoqumica o ramo da Qumica que estuda a troca de energia que
acompanha transformaes, tais como, processos de misturas, transies
de fases e reaes qumicas. Uma das grandezas calculadas e/ou medidas
experimentalmente, em Termodinmica, a variao de entalpia (H), que
corresponde energia absorvida ou liberada numa transformao qumica
sob presso constante. Portanto, as medidas comuns de calor, realizadas em
laboratrio (em geral, sob presso atmosfrica), so variaes de entalpia.

158

Olimpada Brasileira de Qumica - 2015

Iberoamericana

Exame Experimental

A variao de entalpia uma grandeza denominada de funo de estado.


Em outras palavras, dizemos que a variao de entalpia de uma dada transformao independe do caminho sob o qual a transformao realizada.
Essa regra conhecida como Lei de Hess.
Nesta questo vamos fazer uma verificao experimental da Lei de Hess utilizando a entalpia de dissoluo e a entalpia de neutralizao.
As medidas de calor so realizadas num calormetro adiabtico. Este dispositivo um recipiente contendo o sistema sob transformao e que perfeitamente isolado, de modo que no h escoamento nem de calor ou matria,
seja para dentro ou para fora do sistema. Para o calormetro usado neste experimento considere que a capacidade calorfica, Ccal, igual a 17,6 cal/C.
A entalpia calculada pela variao de temperatura que ser convenientemente medida com um termmetro adequado.
PARTE EXPERIMENTAL
Procedimento experimental
A) Determinao do calor (entalpia) de dissoluo do NaOH(s) em gua
(realize o experimento em duplicata):
-

Medir com uma proveta 96 mL de gua destilada a temperatura ambiente e despejar no calormetro vazio. Aps o sistema entrar em
equilbrio trmico, medir a temperatura da gua no calormetro;

Transfira quantitativamente a massa de NaOH, disponvel em um


dos frascos, para dentro do calormetro com gua e agitar levemente para dissolver todo o NaOH. Aps o equilbrio trmico, medir a
temperatura do sistema; Observao: anote o nmero do frasco e a
massa correspondente de NaoH).

QA1) Determine a variao de temperatura (Tdiss): (3 pontos)


Nmero do Frasco Massa de NaOH / g

T inicial / oC

T final / oC

Tdiss / oC

QA2) Calcule a variao de entalpia de dissoluo do hidrxido de sdio


(vamos denomin-la Hdiss). (3 pontos)
I

Olimpada Brasileira de Qumica - 2015

159

Iberoamericana

Exame Experimental

Frmula:
Onde m a massa de soluo, c o calor especfico da soluo.
Observaes: Expresse esse valor em kcal/mol.
Calcule a mdia.
Considere que o calor especfico da soluo independente da temperatura
e possui valor de 0,94 cal g-1 C-1.
Utilize a massa de soluo.
Se por alguma razo voc no obteve Tdiss, utilize o valor de 10 C.
B) Determinao do calor (entalpia) de reao (neutralizao) do NaOH(aq) e
HCl(aq) (realize o experimento em duplicata):
-

Aps esvaziar, lavar e secar convenientemente o calormetro e o


termmetro, medir com uma proveta 100 mL da soluo de NaOH 1
mol/L e a seguir despejar dentro do calormetro. Mea a temperatura
aps o sistema entrar em equilbrio trmico.

Medir 100 mL da soluo de HCl 1 mol/L com uma proveta, coloc-la


num bquer e aps o equilbrio trmico, medir a temperatura; Faa a
medida com o termmetro previamente lavado e seco;

Adicionar a soluo de HCl soluo de NaOH de dentro do calormetro. Fechar o calormetro imediatamente e medir a temperatura
aps o sistema entrar em equilbrio trmico;

QB1) Determine a variao de temperatura (Tneut): (3 pontos)


T inicial / oC

T final / oC

Tneut / oC

QB2) Calcule a variao de entalpia de neutralizao (vamos denomin-la


Hneut). (3 pontos).
Frmula:
Onde mNaOH a massa de soluo de NaOH, cNaOH o calor especfico da
soluo de NaOH, mHCl a massa de soluo de HCl, cHCl o calor especfico da soluo de HCl.

160

Olimpada Brasileira de Qumica - 2015

Iberoamericana

Exame Experimental

Observaes:
Expresse esse valor em kcal/mol.
Calcule a mdia.
Considere que o calor especfico das solues so independentes da temperatura e possuem valor de 0,94 cal/gC para a soluo de NaOH e 1,00 cal/C
para a soluo de HCl.
Considere que ambas as solues possuem densidade igual a 1,00 g/mL.
Se por alguma razo voc no obteve Tneut, utilize o valor de 15 C.
C) Determinao do calor (entalpia) de reao (neutralizao) entre o
NaOH(s) e HCl(aq) (realize o experimento em duplicata):
-

Aps esvaziar, lavar e secar convenientemente o calormetro e o termmetro, medir com uma proveta 100 mL da soluo de HCl 1 mol/L
e a seguir despejar dentro do calormetro. Mea a temperatura aps
o sistema entrar em equilbrio trmico;

Transferir quantitativamente a massa de NaOH disponvel para dentro do calormetro com a soluo de HCl e agitar levemente para dissolver todo o NaOH. Aps atingir o equilbrio trmico, medir a temperatura do sistema;

QC1) Determine a variao de temperatura (Tdn): (1,5 pontos)


Nmero do Frasco

Massa de NaOH / g

T inicial / oC

T final / oC

Tdn / oC

QC2) Calcule a variao de entalpia nesta transformao (vamos denomin-la Hdn). (1,5 pontos).
Frmula:
Onde m a massa de soluo, c o calor especfico da soluo.
Observaes:
Expresse esse valor em kcal/mol.
I

Olimpada Brasileira de Qumica - 2015

161

Iberoamericana

Exame Experimental

Calcule a mdia.
Considere que o calor especfico da soluo final independente da temperatura e possui valor de 1,00 cal/gC.
Considere que a soluo possui densidade igual a 1,00 g/mL.
Se por alguma razo voc no obteve Tdn, utilize o valor de 25 C.
QC3) Escreva abaixo as equaes qumicas envolvidas na parte A e B: (1,0
pontos)

QC4) Calcule a variao de entalpia que resulta da soma de Hdiss com Hneut
(vamos denomin-laHdncal): (1 ponto)

QC5) Calcule o erro percentual associado verificao experimental da lei


de Hess pela frmula: (1,0 pontos)
Frmula:
QC6) Assinale V para verdadeiro e F para falso nas seguintes afirmaes a
respeito de detalhes do experimento e das possveis fontes de erro: (2 pontos)
( ) A falta de homogeneidade na temperatura pode acontecer devido a
uma troca rpida de calor dentro do calormetro.
( ) Quanto maior o intervalo de tempo utilizado para a leitura do termmetro, aps cada procedimento, melhor ser a preciso na medida do correto T.
( ) Um mau isolamento trmico do calormetro ocasionando perdas de calor
para o exterior uma importante fonte de erro.

162

Olimpada Brasileira de Qumica - 2015

Iberoamericana

Exame Experimental

( ) Um mecanismo que permitisse uma leve de agitao do sistema poderia


acelerar as trocas de calor e assim proporcionar medies mais adequadas para o correto T.
Experimento 2: Anlise De Uma Mistura De Carbonato E Bicarbonato
Este procedimento envolve duas titulaes. (a) Primeiro, a alcalinidade total
(bicarbonato + carbonato) medida titulando-se a mistura com HCl padro
at o ponto final verde, indicado pelo verde de bromocresol. (b) Uma alquota separada da amostra desconhecida e tratada com NaOH padro em
excesso para converter bicarbonato em carbonato. A seguir todo o carbonato precipitado com BaCl2. O excesso de NaOH titulado imediatamente
com HCl padro para determinar quanto bicarbonato estava presente. A partir da alcalinidade total e da concentrao de bicarbonato possvel calcular
a concentrao original de carbonato.
Procedimento
s Transfira a quantidade total da mistura de carbonato e bicarbonato, contida no frasco rotulado Amostra desconhecida, para um balo volumtrico de 250 mL com o auxlio de um funil. Para uma transferncia quantitativa, coloque um pouco de gua contida na pisseta dentro do tubo e
transfira para o balo volumtrico. Repita este procedimento por mais
duas vezes. Lave o funil algumas vezes com pequenas pores de gua
para dissolver a amostra. Remova o funil, dilua at a marca de aferio e
homogeneze bem.
s Anlise da alcalinidade total. Pipete uma alquota de 25,00 mL de soluo da amostra desconhecida para um erlenmeyer de 125 mL, adicione
trs gotas do indicador verde de bromocresol (contido no tubo plstico
rotulado verde de bromocresol) e titule com HCl padro, (contido no
frasco rotulado Soluo HCl 0,094 mol/L), onde a concentrao exata
est descrita no rtulo, at mudana da cor do indicador para esverdeado. Repita este procedimento com pelo menos mais uma alquota de
25,00 mL.
s Teor de bicarbonato. Pipete 25,00 mL da soluo da amostra desconhecida e 50,00 mL de NaOH padro (contido no frasco rotulado Soluo
NaOH 0,100 mol/L) para um frasco de 250 mL. Misture e adicione, por
meio de uma pipeta, 10,00 mL de BaCl2 10% m/m (contido no frasco rotulado Soluo BaCl2 10%. Misture novamente de modo a precipitar o
I

Olimpada Brasileira de Qumica - 2015

163

Iberoamericana

Exame Experimental

BaCO3. Em seguida, adicione duas gotas do indicador fenolftalena (contido no tubo plstico rotulado fenolftalena) e titule imediatamente a
mistura com soluo HCl 0,094 mol/L. Repita este procedimento com
pelo menos mais uma alquota de 25,00 mL da amostra desconhecida.
Questionrio:
s Escrever todas as equaes qumicas balanceadas, envolvidas nas duas
titulaes utilizadas, indicando os estados de agregao. (2 pontos)
s Calcule a alcalinidade total mdia na amostra. (7 pontos)
Volume/mL (5 pontos)

Concentrao/mol L-1
(1,5 pontos)

Concentrao mdia/
mol L-1 (0,5 ponto)

s Calcule a concentrao mdia de bicarbonato na amostra. (7 pontos)


Volume/mL (5 pontos)

Concentrao/mol L-1
(1,5 pontos)

Concentrao mdia/
mol L-1 (0,5 ponto)

s Calcule a concentrao mdia de carbonato na amostra. (2 pontos)


Concentrao/mol L-1 (1,5 pontos)

Concentrao mdia/ mol L-1 (0,5 ponto)

s Expresse a composio da amostra desconhecida, em % (m/m), para


cada uma das espcies analisadas (bicarbonato de sdio, 84 g/mol e carbonato de sdio, 106 g/mol). (2 pontos)

164

Olimpada Brasileira de Qumica - 2015

Iberoamericana

Exame Terico

XX Olimpada
Iberoamericana
de Qumica
04 a 14 de setembro de 2015
Teresina - Brasil
EXAME TERICO
10 de setembro de 2015
Instrues gerais para o exame terico (TOTAL DE PONTOS = 90
que so equivalentes a 60% da nota final)
1. Este exame contm um total de 45 pginas, incluindo um conjunto de equaes universais e constantes que podem ser teis.
2. Escreva seu nome e o cdigo que tenha sido atribuda a voc na capa do exame e em
todas as folhas.
3. Voc tem 5 horas para completar todo o exame e registrar suas respostas nos locais apropriados. Voc deve parar o trabalho imediatamente aps o recebimento do sinal de parada.
4. Voc deve escrever todos os seus resultados nas caixas apropriadas em cada pgina.
Nada escrito em outros lugares ser considerado para a pontuao, mas voc pode usar o
verso das folhas como papel de rascunho.
5. Faa os clculos relevantes nos espaos apropriados quando necessrio. Voc receber a
mais alta pontuao apenas se o processo para obter o resultado final estiver explcito.
6. Use somente a caneta, a tabela peridica e a sua calculadora.
7. Observe que para os dados numricos, o ponto separa milhares e a vrgula separa a parte
inteira da parte dos decimais.
8. Para ir ao banheiro, pea permisso ao supervisor.
9. Aps a concluso do exame e, quando solicitado, voc entregar o exame e seguir as
instrues dadas pelo supervisor. No deixe o gabinete sem ser previamente autorizado.

Olimpada Brasileira de Qumica - 2015

165

Constantes Fsicas e Equaes

N = 6,0221 x 1023 mol

Constante de Avogadro

Iberoamericana
kB = 1,3807 x 10 23 JK 1

Exame Terico

Constante de Boltzmann

R = 8,3145 JK 1mol 1 = 0,08205 atmLK 1mol

Constante dos gases


Constante de Faraday

Constantes
Fsicas
F = 96485,3
Cmol 1 e Equaes

Velocidade
Constantedadeluz
Avogadro

c N= =
2,9979
6,0221x x10108 23ms
mol11

Constante
Boltzmann
Constante
dede
Planck

1,3807x 10
x 1034 JsJK
hk=B =
6,6261

Carga do eltron

e = 1,602 x 10

Constante dos gases


Constante de Faraday
Permissividade
Velocidade dado
luzvcuo
Constante
de Planck
Massa
do eltron
Carga do eltron
Presso padro
Permissividade do vcuo
Presso
Massaatmosfrica
do eltron
padro
ZeroPresso
da escala
Celsius
Presso
atmosfrica
nanmetro (nm)
Zero da escala Celsius
picmetro (pm)
nanmetro (nm)
ngstrm
()(pm)
picmetro
ngstrm ()

4
4

23

R = 8,3145 JK19 mol = 0,08205 atmLK1mol1


C
F = 96485,3 Cmol1
2 1
1
108 19
0c =
=8,8541
2,9979 xx 10
msC1J m
x 1034x Js
mhe =
= 6,6261
9,10938215
10 31 kg
19
e = 1,602 x 10 C
P = 1 bar = 105 Pa19 2 1 1
0 = 8,8541 x 10 C J m
5
Pm
760 mmHg = 760 Torr
atm = 1,01325 x 10 Pa
= 9,10938215 x 1031= kg
e
5
P = 1 bar
273,15
K = 10 Pa
=
1,01325
x 105 Pa = 760 mmHg = 760 Torr
P
atm = 10 9 m
1 nm
273,15 K
1 pm = 10 12
m
1 nm = 109 m
10 12
11pm
= 10
mm
= 10
10
m
1

=
10
3,1416
3,1416
4
= [Zn(OH)422] / ([Zn2+2+] [OH
])
= [Zn(OH)4 ] / ([Zn ] [OH]4)
1

ParaPara
tcnica
ESR:
tcnica
ESR: Idade =

Cintica
de primeira
Cintica
de primeiraordem:
ordem: mfinal = minicial e

kt

ln2

onde m = massa, k = constante de velocidade, t = tempo decorrido e t 1/2 = tempo de meia-vida


onde m = massa, k = constante de velocidade, t = tempo decorrido
e t1/2 = tempo de
2
Entalpia de fuso: Hfuso = m.Lfuso, onde m = massa e Lfuso = calor latente de fuso.
meia-vida

Entalpia de fuso: Hfuso = m.Lfuso, onde m = massa e Lfuso = calor latente de fuso.

Problema 1 - O B-R--Br

Questo
Pontos (mximo)
Pontos (correo)

1.1.1
3,0

1.1.2
2,0

1.2.1
3,0

1.2.2
2,0

Total
10

01- Quase
os moradores da cidade de Teresina conhecem a expresProblema
1 - Otodos
B-R--Br
so B-R--BR que corresponde a um perodo do ano caracterizado por
Questo
1.1.1
1.1.2por volta
1.2.1 de 1.2.2
dias com altas
temperaturas
(s vezes,
40 C) eTotal
baixa umidade
relativa do ar. A expresso se origina da terminao dos nomes dos meses
Pontos (mximo)
3,0
2,0
3,0
2,0
10

166

I
Olimpada
Brasileira de Qumica - 2015
Pontos
(correo)

Iberoamericana

Exame Terico

(em portugus) correspondentes ao perodo, ou seja, Setembro, Outubro,


Novembro e Dezembro. Portanto, estamos em pleno B-R--BR e como
dito popularmente na cidade, nesta poca um aparelho de ar condicionado
no um luxo e sim, uma necessidade.
Um aparelho de ar condicionado uma mquina que destri trabalho num
ciclo, resultando na retirada de calor de uma fonte fria (o ambiente em que
se deseja resfriar) para uma fonte quente (o meio externo). Apesar dos diferentes arranjos e modos com que diferentes aparelhos de ar condicionados
so construdos, se focarmos no que eles fazem e no em como eles fazem
podemos entender estes dispositivos como refrigeradores de Carnot e assim
aplicando a primeira e a segunda lei da Termodinmica estabelecer um limiaos seus
desempenhos.
Qh,teento
a primeira
lei da Termodinmica diz que: Qh = Qc + W. Ainda, impondo o
Para um refrigerador
de Carnot,temos
o coeficiente
deo desempenho,
, calculado
segundo
da Termodinmica
que:que:
1) se
reversivelmente,
Q
a primeira
lei da Termodinmica
diz
Qh =ciclo
Qc +operar
W. Ainda,
impondo o
h, entoprincpio
atravs da razo entre benefcio e o custo, ou seja,
o benefcio
o calor retirado
Q
,
ento
a
primeira
lei
da
Termodinmica
diz
que:
Q
=
Q
+
W.
Ainda,
impondo
h
h
c
a variao
de
entropia
zero,
seja,
Shdestrudo,
= Sc ou
QhW.
= Qoperar
Tnum
as o
h Este
c/T
c, onde
h e Tar
c so
, eTermodinmica
oser
custo
o ou
trabalho
trabalho,
condida fonte
fria,
Qda
segundo
princpio
temos
que:
1) se
o/Tciclo
reversivelmente,
c
cionado,
adas
energia
eltrica
gasta
seu funcionamento.
oSecalor
cedido
para
segundo princpio
da Termodinmica
temos
que: 1) se o cicloSe
operar
fontes
quente
e no
fria,
oreversivelmente,
opera
atemperaturas
variao de entropia
ser zero,
ou seja,
Sh =respectivamente;
Sc ou Qh/Th = Qc/T2)c, onde
Thciclo
e Tc so
as
a fonte quente Qh, ento a primeira lei da Termodinmica diz que: Qh = Qc + W.
a
variao
de
entropia
ser
zero,
ou
seja,
S
=
S
ou
Q
/T
=
Q
/T
,
onde
T
e
T
so
as
h
c
h
h
c
c
h
c
> opera
Qoc/T
irreversivelmente
real)
de entropiatemos
e assim
h/T
c.
temperaturas
das(transformao
fontes
quente
e h
fria,aumento
2)
Se que:
oQciclo
Ainda, impondo
o segundo
princpio
darespectivamente;
Termodinmica
1)h se
ciclo
temperaturas
das escrever
fontesa variao
quente
edefria,
respectivamente;
Se So =ciclo
operar
reversivelmente,
entropia
ser zero, ou2)seja,
S ouopera
Q/
Diante
disso
podemos
que:
irreversivelmente
(transformao
real) h aumento de entropia e assim Qhh/Th >c Qc/Tc. h
Th = Qc/Tc, onde Th e Tc so as temperaturas das fontes quente e fria, respectivairreversivelmente (transformao real) h aumento de entropia e assim Qh/Th > Qc/Tc.
Diante
disso2)podemos
escrever
que:
mente;
Se o ciclo
opera irreversivelmente
(transformao real) h aumento de
Diante disso
podemos
que:
entropia
e assim
Qh/Tescrever
>
Q
/T
.
Diante
disso
podemos
escrever que:
h
c c
Mas como:
Mas como:
Mas como:
Mas
como:

ou
ou
ou

Finalmente:
Finalmente:
Finalmente:
Finalmente:

ou

ou
A desigualdade da equao acima mostraou
que o mximo desempenho possvel
obtido operando reversivelmente. Na prtica, para aparelhos de ar condicionaA desigualdade
acima
mostra que
o mximo
desempenho

do e freezers da
(ouequao
geladeiras)
a operao
reversvel
impossvel,
uma possvel
vez que tomaria
um tempo
infinito (aacima
operao
reversvel
tem
modificaes
infinitesimais).
operando
Na
prtica,
aparelhos
de ar condicionado
Aobtido
desigualdade
dareversivelmente.
equao
mostra
quepara
o mximo
desempenho
possvel e
A
desigualdade
da
equao
acima
mostra
que
o
mximo
desempenho
possvel

freezersoperando
(ou geladeiras)
a operaoNa
reversvel
impossvel,
umadevez
tomaria
um
obtido
reversivelmente.
prtica, para
aparelhos
ar que
condicionado
e
I
Olimpada Brasileira de Qumica - 2015
I
167
obtido
operando
reversivelmente.
Na modificaes
prtica, parainfinitesimais).
aparelhos de ar condicionado e
tempo
infinito
(a operao
reversvelreversvel
tem
freezers
(ou geladeiras)
a operao
impossvel, uma vez que tomaria um

Iberoamericana

Exame Terico

1.1. Considere um aparelho de ar condicionado de 30000 Btu/h, operando


numa sala em Teresina, com razo de eficincia energtica (EER, do ingls,
Energy Efficiency Ratio) de 8,5 Btu h1 Watt-1. A EER definida como o nmero
de Btu/h extrada do ambiente dividida pelo consumo de energia em Watt.
(1 Watt = 1 J/s e 1 Btu = 1,055 kJ).
1.1.1. Calcule o coeficiente de desempenho do ar condicionado
Coeficiente de desempenho:
1.1.2. Se a temperatura externa 40 C e a temperatura interna 20 C, qual a
porcentagem do valor mximo terico ser o coeficiente de desempenho?
Porcentagem:
1.2. Em Teresina, to importante quanto os aparelhos de ar condicionados
so as geladeiras. Necessitamos de muita gua gelada por aqui.
1.2.1. Suponha que um motor eltrico fornea o trabalho para operar um
refrigerador de Carnot. O interior do refrigerador se encontra a 0 C e gua lquida tambm a 0 C convertida em gelo a 0 C. Sabendo que Hfuso = 334
J/g para o gelo, se a temperatura do lado de fora do refrigerador for 40 C,
qual a massa de gelo que pode ser produzida em 10 minutos por um motor
de HP (186,5 Watt) operando continuamente? Admita que o refrigerador
perfeitamente isolado e que a eficincia envolvida tem o seu mximo valor
possvel.
Massa de gelo:
1.2.2. Em um refrigerador com uma potncia de HP (186,5 Watt), se a temperatura no lado interno for mantida em -20 C contra uma temperatura exterior mxima de 45 C, qual a quantidade de calor mxima que se pode
extrair (em Watt) se o motor opera continuamente? Assuma que o coeficiente
de desempenho 75 % do valor para a operao reversvel.
Calor extrado:

Problema 2 - A cajuna cristalina em Teresina


Questo
Pontos (mximo)
Pontos (correo)

168

2.1.1 2.2.1
10

60

Olimpada Brasileira de Qumica - 2015

2.3.1

2.3.2

Total

10

10

90

Total
Padronizado
15

Iberoamericana

Exame Terico

02 - A cajuna cristalina em Teresina. Esta frase est presente numa msica


de autoria do artista baiano, Caetano Veloso, e segundo ele prprio a msica
foi composta aps um encontro, em Teresina, com o pai de um amigo falecido (Pai do artista piauiense Torquato Neto). Na ocasio, os dois, enlutados,
conversavam e tomavam cajuna.
Os atrativos da bebida so ( claro) o seu sabor peculiar, em geral, doce com
um sabor residual levemente azedo, e a sua aparncia ou colorao, que varia de amarelo a mbar. Em geral, a cajuna obtida artesanalmente, pelas
seguintes etapas: 1) O suco de caju, obtido da prensagem do pednculo,
inicialmente filtrado para remoo de resduos grosseiros; 2) A seguir ocorre
a precipitao dos taninos mediante a formao de complexos com gelatina
(colgeno); 3) Uma nova filtrao retira os complexos formados e fornece um
lquido quase incolor; 4) Os frascos nos quais este ltimo lquido colocado
so submetidos a aquecimento em banho-maria, numa temperatura de cerca de 90 C. Nesta ltima etapa, a cajuna adquire sua colorao tpica, graas
caramelizao do acar naturalmente contido no suco de caju, a frutose.
2.1. Taninos so compostos polifenlicos com sabor amargo e adstringente
que podem precipitar protenas e outros compostos orgnicos, como alcalides. Os taninos hidrolisveis so derivados do cido glico (cido 3,4,5-triidrxibenzico), onde unidades deste cido se encontram esterificadas com
polialcoois. O cido tnico um exemplo desta classe e tem como uma de
suas possveis estruturas, a que mostrada a seguir.

cido Tnico (C76H52O46)


Tannicacid de en:User_talk:Ronhjones - Trabajopropio. Disponible bajo la licencia Dominio
pblico va Wikimedia Commons - https://commons.wikimedia.org/wiki/File:Tannic_acid.svg#/
media/File:Tannic_acid.svg

Olimpada Brasileira de Qumica - 2015

169

cido Tnico (C76H52O46)

Iberoamericana

Exame Terico

Tannicacid de en:User_talk:Ronhjones - Trabajopropio. Disponible bajo la licencia Dominio pblico va Wikimedia Commons -

https://commons.wikimedia.org/wiki/File:Tannic_acid.svg#/media/File:Tannic_acid.svg

2.1.1. Qual o2.1.1.


nmero
de carbonos
notnico
cido
tnico
mostrado
Qual o nmero
de carbonosassimtricos
assimtricos no cido
mostrado
abaixo
e qual
abaixo e qual seu
seundice
ndice
de defidecincia
de hidrognio?
de deficincia
hidrognio?
Nmero de carbonos assimtricos:
Nmero de carbonos
assimtricos:

IDH:

IDH:

2.2. A partir do cido glico, a mescalina (C11H17NO3), um alucingeno com importncia


2.2. A partir do
cido glico, a mescalina (C11H17NO3), um alucingeno com
pode serpode
sintetizada.
Em 1951, Tsao props
a sntese
da mescalina
importnciafarmacolgica,
farmacolgica,
ser sintetizada.
Em 1951,
Tsao
props
(J.
Am.
Chem.
Soc.
5495

5496)
a
partir
do
cido
glico
correspondendo
esquema
a sntese da mescalina (J. Am. Chem. Soc. 5495 5496) a partiraodo
cido
que parcialmente
abaixo:
glico correspondendo
aomostrado
esquema
que parcialmente mostrado abaixo:
1) LiAlH/Et2O

1) NaOH/Na2CO3

cido 3,4,5Trimetoxibenzico

2) (CH3O)2SO2

(C11H14O5)

2) H2SO4

C
(C10H14O4)

3) HCl

cido Glico

C
(C10H14O4)

KCN/H2O

Cloreto de 3,4,5Trimetoxibenzila

1) LiAlH/Et2O

(C11H13NO3) 2) H SO
2
4

F
Mescalina
(C11H17NO3)

2.2.1. Desenhe as estruturas dos compostos A, B, C, D, E e F (mescalina). (60 pontos,


2.2.1. Desenhe
as estruturas dos compostos A, B, C, D, E e F (mescalina). (60
10 pontos
porestrutura).
estrutura).
pontos, 10 pontos
por

2.3. Na caramelizao do acar (para dar7 a colorao cajuna) ocorrem


diversas reaes complexas, sendo ao final a produo das chamadas melanoidinas que so polmeros coloridos (amarelos) cujo monmero o hidroximetilfurfural (HMF). O HMF uma molcula resultante da transformao da
frutose e da glicose. No caso da cajuna, supostamente, h a presena apenas da frutose. A estrutura da D-frutose (uma ceto-hexose) mostrada abaixo, em notao de Fischer, ao lado da estrutura do HMF. mostrada tambm
uma estrutura em projeo de Haworth da -D-frutopiranose, que decorre
da ciclizao da frutose em um anel de seis membros.
CH2OH
O
HO
H

H
OH

Hidroximetilfurfural (HMF)
CH2OH

D-frutose

-D-frutopiranose

170

Olimpada Brasileira de Qumica - 2015

Iberoamericana

Exame Terico

2.3.1. Desenhe a estrutura da alfa-D-frutofuranose, na notao de Haworth e


d a configurao (R ou S) para o carbono anomrico.
ESTRUTURA:

CONFIGURAO:

2.3.2. Marque, com um X, a nica opo que completa corretamente os


espaos em branco do texto abaixo:
Na sequncia, a partir da -D-frutofuranose seguem-se __________ desidrataes, ______________ o anel de cinco membros, at a obteno do HMF,
que um composto _________________, tendo os intermedirios de sua formao compostos com frmulas moleculares ________ e ________.
(
(
(
(
(
(

) Trs quebrando-se aliftico C6H10O5 C5H6O3


) Duas mantendo-se aromtico C6H10O5 C6H8O4
) Duas quebrando-se aliftico C5H8O4 C5H6O3
) Trs mantendo-se aromtico C6H10O5 C6H8O4
) Duas quebrando-se aliftico C5H8O4 C5H6O3
) Trs mantendo-se aromtico C5H8O4 C6H8O4

Problema 3 Produtos Naturais no Piau


Questo
Pontos (mximo)
Pontos (correo)

3.1
4,0

3.2
4,0

3.3
4,0

3.4
3,0

Total
15

03 - Cenostigma macrophyllum Tul. var. acuminata Teles Freire uma rvore conhecida popularmente por caneleiro, sendo bastante comum no Piau.
Sua beleza, decorrente de flores amarelas, faz com que esta seja empregada como planta ornamental, sobretudo na regio da grande Teresina PI,
onde foi escolhida, atravs de decreto municipal, como a rvore smbolo da
I

Olimpada Brasileira de Qumica - 2015

171

Iberoamericana

Exame Terico

cidade (Silva, H. R., et al., Qum. Nova, 30, 8, 2007). Estudos com extrato de
folhas desta espcie revelaram a presena de atividades antioxidante, anti-inflamatria, antinociceptiva, antibacteriana, antiulcerognica e hepatoprotetora. Adicionalmente, esta planta teve a composio qumica das cascas
do seu caule estudada por pesquisadores da Universidade Federal do Piau,
sendo observada, por exemplo, a ocorrncia de colesterol, um esteroide
muitas vezes considerado apenas com origem animal. Os demais constituintes identificados so mostrados na Figura abaixo (Nas duas estruturas mais
abaixo da Figura, o grupo R indicado para diferentes constituintes com um
esqueleto bsico comum).
3.1. Identifique TODAS as famlias de compostos orgnicos (por exemplo,
aldedo, cetona, ster, etc.) nas estruturas 2, 4, 6 e 15 presentes na figura a
seguir, as quais foram identificadas nas cascas do caule do caneleiro:

172

Olimpada Brasileira de Qumica - 2015

Iberoamericana

Exame Terico

OBSERVAO: A notao CX:Y (a,b) representa radical acila, com cadeia hidrocarbnica contendo X-1 carbonos e Y ligaes duplas que esto localizadas nos carbonos a e b.
2:
4:
6:
15:
Protium heptaphyllum March, pertencente famlia Burseraceae e, comumente conhecida por almcega, outra planta encontrada no Piau cuja resina exsudada a partir do tronco possui cerca de 45,3% de uma mistura de
-amirina (20) e -amirina (21) que so compostos naturais pertencentes
classe dos triterpenides pentacclicos. Geralmente a - e -amirina (20+21)
so isoladas em mistura de difcil separao e em cromatografia em camada
delgada (CCD) de gel de slica, eluda com hexano-acetato de etila (8:2) a
mistura mostra uma nica mancha (Vieira Jnior et al., Qum. Nova, 30, 2,
2007). Estes compostos possuem vrias atividades farmacolgicas.
A reao da mistura de -amirina (20) e -amirina (21) com anidrido actico
em presena de piridina como catalisador, temperatura ambiente, origina a
mistura de 22+23 (sendo 22 resultante de 20 e 23 resultante de 21), conforme esquematizado a seguir:

3.2. Desenhe a estrutura dos compostos 22 e 23.

Olimpada Brasileira de Qumica - 2015

173

Iberoamericana

Exame Terico

22

23

3.3. Sobre os constituintes mostrados acima e os princpios da CCD em gel


de slica, analise as afirmaes abaixo e assinale V para VERDADEIRO ou F
para FALSO.
- As substncias - e -amirinas possuem o mesmo Rf: ( ) V ( ) F
- Sobre os constituintes da resina de almcega e seus derivados, o composto 23 possui menor Rf do que o composto 21: ( ) V ( ) F
- Sobre os constituintes da casca do caule do caneleiro, o composto 2 possui
maior Rf do que o composto 1: ( ) V ( ) F
- Sobre os constituintes da casca do caule do caneleiro, para os compostos
17, 18 e 19, observa-se que no h diferenas apreciveis de Rf: ( ) V ( ) F
3.4. Ainda sobre CCD em gel de slica, considere uma mistura composta de
bifenil, cido benzico e lcool benzlico (mistura 1) e uma mistura dos compostos meta-hidroxi-fenol, meta-dimetil-benzeno e meta-metoxi-fenol (mistura 2).
Desenvolvendo a CCD, em condies adequadas para cada mistura, qual
deve ser a ordem relativa dos compostos aps o desenvolvimento da placa
cromatogrfica?
Para sua resposta utilize as Figuras abaixo, onde as posies so apenas hipotticas:
Mistura 1:

174

Olimpada Brasileira de Qumica - 2015

Iberoamericana

Exame Terico

Mistura 2:

Problema 4 - Breaking Bad


Questo
Pontos (mximo)
Pontos (correo)

4.1
4,0

4.2.1
3,0

4.2.2
0,5

4.2.3
2,5

Total
10

04 - O seriado Breaking Bad foi um dos TV shows de maior sucesso nos ltimos anos na televiso americana e mundial. O personagem principal Walter
White um professor de Qumica de ensino mdio e ajudante numa lavadora de carros nas horas vagas.

Olimpada Brasileira de Qumica - 2015

175

Iberoamericana

Exame Terico

Dentre as diversas situaes em que Heisenberg (Walter White) se envolve,


junto com o seu ex-aluno e ajudante no mundo do crime, Jesse Pinkman, h
referncias explcitas do seu timo conhecimento de qumica.
4.1. Walter White no s qumico orgnico. No episdio 9 da segunda
temporada, ele e seu pupilo vo para o meio do deserto produzir (cozinhar)
a meta-anfetamina dentro de um trailer (Nos EUA trailers so chamados de
RVs, ou seja, Recreational Vehicles) que usado como um laboratrio ambulante. Por descuido ou burrice, Jesse Pinkman esquece a chave no contato
com a ignio ligada e a bateria do (velho) veculo descarrega, impedindo
que eles dem a partida no motor para ir embora. Estando no meio do
nada os personagens entram em pnico e na iminncia de morrerem de
sede, e num lampejo de inteligncia, Walter White usa os seus conhecimentos qumicos para fazer uma bateria. Usando esponjas embebidas com uma
soluo eletroltica, metais galvanizados (parafusos, porcas, arruelas, moedas
de dlar, etc) e as pastilhas de freio do trailer, ele monta um sistema de clulas eletroqumicas em srie, o qual ligado convenientemente aos polos da
bateria no motor do trailer permite que seja dada a ignio.
Os constituintes de cada clula eletroqumica so delineados no dilogo entre os personagens. Entende-se que: 1) A soluo eletroltica uma soluo
de hidrxido de potssio; 2) Os metais galvanizados fornecem o zinco; 3) A
partir das pastilhas de freio retirado grafite revestido com xido de mercrio (por simplicidade admitiremos o xido de mercrio II HgO); 4) Fios
de cobre so utilizados nas ligaes entre os eletrodos e as clulas; e 5) S
existia material disponvel para a construo de seis clulas.
Sabendo que os potenciais padres de reduo das semi-reaes de Zn2+/
Zn e Hg2+/Hg so, respectivamente, - 0,76 V e 0,85 V, calcule a diferena de
potencial padro de uma clula construda por Walter White e a diferena de
potencial mximo da bateria (sistema em srie). Escreva as semi-reaes que
possivelmente ocorrem, bem como a reao global e identifique o ctodo, o
nodo e as suas polaridades.
Potencial da clula:
Potencial do sistema em srie (bateria):
Semi reao no ctodo:
Polaridade:
Semi reao no nodo:
Polaridade:
Reao Global:

176

Olimpada Brasileira de Qumica - 2015

Iberoamericana

Exame Terico

4.2. Em outro episdio marcante da srie Breaking Bad, um j debilitado


pelo tratamento de cncer, Walter White, ou melhor, Heisenberg como ele se
autodenominou pela primeira vez neste mesmo episdio, procura reaver um
pacote de meta-anfetamina que foi roubado do seu parceiro Jesse Pinkman
por um traficante extremamente violento chamado Tuco Salamanca. Alm
do roubo, Tuco espancara Jesse brutalmente e assim Heisenberg claramente
tambm busca vingana. Ele visita o traficante no seu escritrio e apresenta um pacote com cristais que so supostamente meta-anfetamina. Quando
Tuco o ironiza e tenta amea-lo, ele revela que os cristais so de fulminato
de mercrio (Hg(CNO)2) e em seguida atira um dos cristais ao cho, resultando numa exploso que destri o escritrio. Heisenberg deixa claro que os
demais cristais tambm so de fulminato de mercrio e com isso Tuco colabora prontamente, pagando o carregamento anterior e se comprometendo
a comprar carregamentos futuros.
O Hg(CNO)2 explode prontamente com a formao de vapor de mercrio,
monxido de carbono e gs nitrognio, em resposta ao impacto, frico ou
aquecimento: Hg(CNO)2 (s) 2 CO (g) + N2 (g) + Hg (g)
A estrutura cristalina do fulminato de mercrio no foi explicada at 2007
(Beck W et al., Z. Anorg. Allg. Chem. 2007, 633, 1417-1422). Hoje, sabe-se
que o fulminato de mercrio cristaliza em uma cela ortorrmbica (Figura
abaixo, em que cada ponto representa uma molcula) com face centrada
e parmetros de cela: distncias a = 5,355 ; b = 10,459 e c =7,558 . As
distncias de ligao na molcula so HgC = 2,029 , CN = 1,143 , NO =
1,248 . Os ngulos na molcula so CHgC = 180,0, HgCN = 169,1, CNO
= 179,7.
= = = 90

Retculo de Bravais ortorrmbico de face centrada.


Orthorhombic-face-centered by Original PNGs by User:Rocha, traced in Inkscape by
User:Stannered - Crystal structure. Licensed under CC BY-SA 3.0 via Commons - https://commons.wikimedia.org/wiki/File:Orthorhombic-face-centered.svg#/media/File:Orthorhombic-face-centered.svg
I

Olimpada Brasileira de Qumica - 2015

177

Iberoamericana

Exame Terico

( ) Angulares maior 5s

4.2.1.
Calcule
a densidade
do fulminato de mercrio em g/cm3.
( ) Lineares
maior
6s
4.2.2.
Qualomaior
nmero
( ) Angulares
6s de tomos em cada cela unitria, no fulminato de mercrio?
4.2.3. Sobre a estrutura do fulminato de mercrio em comparao com o
complexo tetradrico [Hg(CNO)4]2-, marque com X, na lista abaixo, a nica
opo que completa a expresso de forma quimicamente coerente.
As ligaes Hg-C nas molculas de Hg(CNO)2, que so molculas _________,
so mais curtas do que aquelas no complexo tetradrico, uma vez que h
uma ________ contribuio do orbital _______ na ligao Hg-C do Hg(CNO)2.
Problema 5 - Equilbrio de solubilidade
( ) Lineares maior 5s
( ) Lineares menor 6s
Questo
5.1
5.2
5.3
5.4
5.5
5.6
5.7
5.8
Total
( ) Angulares maior 5s
( ) Lineares maior 6s
Pontos
( ) Angulares maior
1,0
6s 2,0
1,0
2,0
2,0
2,0
3,0
2,0
15
(mximo)
Pontos
(correo)

Problema 5 - Equilbrio de solubilidade

Questo
5.1 5.2 5.3 5.4 5.5 5.6 5.7 5.8
Total
Pontos
(mximo)
1,0
1,0
2,0
2,0
2,0
2,0
3,0
2,0
05. ons zinco podem reagir com ons hidrxido para formar uma espcie insolvel, o 15
Pontos (correo)
hidrxido de zinco. Entretanto, a adio de um excesso de molculas do ligante (neste
caso, OH-) pode resultar na formao de espcies solveis, tal como os ons complexos
2+
05.
ons
zinco
ons hidrxido
para formar
uma espcie
; Zn(OH)
; Zn(OH)3-;reagir
Zn(OH)4com
, aumentando
a sua solubilidade.
Em relao
Zn(OH)
2(aq)podem
insolvel,
o
hidrxido
de
zinco.
Entretanto,
a
adio
de
um
excesso
de mol-17
ao hidrxido de zinco, seu produto de solubilidade 1,80 x 10 .
culas do ligante (neste caso, OH ) pode resultar na formao de espcies solveis, tal como os ons complexos Zn(OH)+; Zn(OH)2(aq); Zn(OH)3-; Zn(OH)42-,
5.1 Calcule a solubilidade
do hidrxido de zinco
em meio aquoso.
aumentando
a sua solubilidade.
Em relao
ao hidrxido de zinco, seu produto de solubilidade 1,80 x 10-17.

5.1
dosaturada
hidrxido
de zinco
meio
aquoso.a
5.2. Calcule
Calcule o a
pHsolubilidade
de uma soluo
de hidrxido
deem
zinco,
desprezando
formao
de algum
meio aquoso.
Casode
no
tenha obtido
valor da
5.2.
Calcule
o pHcomplexo
de umaem
soluo
saturada
hidrxido
deozinco,
desprezan-6
do
a formao
de algum
complexo
aquoso.
Caso no tenha obtido
esta.
solubilidade
na questo
5.1, considere
o valor em
2,0 xmeio
10 para
o valor da solubilidade na questo 5.1, considere o valor 2,0 x 10-6 para esta.

Os
potenciais
padres
das seguintes
reaes
Os potenciais
padres
das seguintes
reaes abaixo
so: abaixo so:
[Zn(OH)4]2- + 2e2+

Zn + 2e

Zn(s) + 4 OH-

Zn(s)

E = -1,285 V
E = -0,762 V

5.3. Calcule a constante de formao global do complexo (4) [Zn(OH)4]2-.

178

Olimpada Brasileira de Qumica - 2015

17 I

de complexos.
5.5. Calcule a solubilidade do hidrxido de zinco em pH 9,58
Exame Terico

a formao do complexo [Zn(OH)4]2-. (Obs.: Caso no tenha

Iberoamericana

5.5. Calculeno
a solubilidade
do hidrxido4 de
pH; 9,58
em
item 5.3, considerar
= zinco
5,0 xem
1017
caso levando
no tenha

2. (Obs.: Caso
no
a formao do complexo
4] considerar
no[Zn(OH)
item
5.4,
S =desprezando
2,0 xtenha
10-8).encontrado
5.4. Calcule a solubilidadesolubilidade
do hidrxido
de
zinco
em pH 9,58
no item 5.3,
considerar 4 = 5,0 x 1017; caso no tenha encontrad
a formao de complexos.
Resposta:

-8 levando em
5.5. Calcule a solubilidade
doitem
hidrxido
de zincoS em
9,58
solubilidade no
5.4, considerar
= 2,0pH
x 10
).
2
considerao a formao do complexo [Zn(OH)4] -. (Obs.: Caso no tenha
Resposta: A concentrao de cromato em uma17 soluo deve ser regul
encontrado o valor de 4 no
item 5.3, considerar 4 = 5,0 x 10 ; caso no tenha encontrado o valor da solubilidade
no item
5.4, considerar
S=
10-8). ser visto
equilbrio de ons
cromato,
dependente
do2,0
pH,x pode

Resposta:

A concentrao de cromato em uma soluo deve ser regulada control


A concentrao equilbrio
de cromato
soluo deve
ser regulada
controlando
+ uma
15
de2Hem
ons
no equilbri
+ 2cromato,
CrO42- dependente
Cr2O72- + H2do
O pH,
K = pode
1,50 xser
10visto
o pH. O equilbrio de ons cromato, dependente do pH, pode ser visto no
equilbrio abaixo:
2215
Cr2O7de
+solubilidade
H2O K = 1,50
x 10de
2H+ + 2 CrOO
4 produto
do sal
cromato de prata :

O produto de solubilidade do sal de cromato de prata :


-12 cromato de prata :
O produto de
do xsal
Agsolubilidade
10de
2CrO4; KPS = 1,2

5.6. Calcule a solubilidade do Ag2CrO4, em meio bsico, onde somente existam ons cromato,
que
no
haja formao
de CrO
outros
slidos.
Agconsiderando
= 1,2
x 10-12
2CrO4; KPS5.6.
Calcule
a solubilidade
do Ag
2
4, em meio bsico, ond
Resposta:
cromato, considerando que no haja formao de outros slid
-1
K2Cr
O ajustado a
5.7. O pH de uma
0,10
2 7 bsico, onde somente
5.6. soluo
CalculeResposta:
acontendo
solubilidade
domol.L
Ag2CrOde
meio
4, em
3,00 atravs de um tampo cido actico / acetato de sdio. Calcular as conconsiderando
no haja
formao
de outros
slidos.
centraes de Crcromato,
O 2- e de
CrO42- nestaque
soluo.
Alm
disso, calcular
o menor
2 7
+
valor da concentrao de Ag , na qual a precipitao do respectivo cromato
Resposta:
inicia.
Resposta:
5.8. No preparo de 1 litro de soluo tampo cido actico/acetato, com
concentrao final de cido actico 0,1 mol.L-1, calcule a massa de acetato
de sdio necessria para atingir o pH 4,00. A constante de acidez do cido
actico 1,78 x 10-5.
Resposta:

18

18

Olimpada Brasileira de Qumica - 2015

179

Iberoamericana

Exame Terico

exemplo, quando uma amostra contendo ferro dissolvida, normalmente a soluo


resultante contm uma mistura de ons Fe(II) e Fe(III). Se utilizamos um oxidante

Problema 6 Equilbrio de oxidao-reduo

padro para determinar o ferro, primeiro precisaremos tratar a soluo contendo a

Questo
6.1 6.2
6.3 para
6.4converter
6.5 6.6
6.8 Fe(II).Total
amostra
com um agente redutor
auxiliar
todo o6.7
ferro para
Pontos (mximo)

1,0 1,0 2,0 2,0 2,0

3,0

1,0

3,0

15

Em
uma anlise
do teor de ferro na magnetita, 4,00 g de p de magnetita foram
Pontos
(correo)

06
- A magnetita
um dos concentrado.
mais importantes
minrios
de foi
ferro
em funo
tratados
com cidoclordrico
O material
insolvel
separado
por
da sua abundncia e alto teor de ferro. A anlise quantitativa do ferro exfiltrao sendo lavado cuidadosamente. No material filtrado, todos os ons de ferro
pressa na forma de xido de ferro(III) e pode ser feita usando a titulometria
foram
reduzidos para ferro
A soluo
(pH precisa
= 0) foi estar
titulada
com
dicromato
de
oxidao-reduo.
Para(II).
isso,
o analito
em
umons
nico
estado
de
oxidao.
Geralmente,
as
etapas
que
precedem
a
titulao,
tais
como
-1
(CM = 0,100 mol.L ), como agente oxidante. O volume utilizado na titulao foi de a
dissoluo da amostra e a separao de interferncias, convertem o analito
a33,74
umamL.
mistura de estados de oxidao. Por exemplo, quando uma amostra
contendo ferro dissolvida, normalmente a soluo resultante contm uma
mistura de ons Fe(II) e Fe(III). Se utilizamos um oxidante padro para determinar o ferro, primeiro precisaremos tratar a soluo contendo a amostra com
um agente redutor auxiliar para converter todo o ferro para Fe(II).
Em uma anlise do teor de ferro na magnetita, 4,00 g de p de magnetita foram
cido
clordrico
concentrado.
O material
insolvel
foi sepa6.1.tratados
Escreva acom
reao
qumica
balanceada
para a reao
do xido
de ferro(III)
com
rado por filtrao sendo lavado cuidadosamente. No material filtrado, todos
cido
os
onsclordrico.
de ferro foram reduzidos para ferro (II). A soluo (pH = 0) foi titulada
com ons dicromato (CM = 0,100 mol.L-1), como agente oxidante. O volume
utilizado na titulao foi de 33,74 mL.
6.1. Escreva a reao qumica balanceada para a reao do xido de ferro(III)
6.2. Dentre os pares redox abaixo, marque com um X quais espcies poderiam ser
com cido clordrico.
agentes redutores adequados para a reao de reduo para obter-se Fe(II)? Use as

6.2. Dentre os pares redox abaixo, marque com um X quais espcies podeinformaes
(potenciais
de reduo)
contidas
na tabela
abaixo.
riam
ser agentes
redutores
adequados
para
a reao
de reduo para obter-se
Fe(II)? Use as informaes (potenciais de reduo) contidas na tabela abaixo.
Ox/Red

E (V)

2+

Zn /Zn
4+

3+

-0,76

2+

Sn /Sn

0,15

SO42-/SO32-

( ( )) Br
Br--

Ox/Red
Fe /Fe
2-

) Cr 3+

3+

Cr2O7 /Cr
Cl2/Cl-

0,17

E (V)

2+

) SO3 2-

( ) Cr 3+

Ox/Red
-

0,77

E (V)
-

1,44

2+

1,51

BrO3 /Br
3+

1,33

Mn /Mn

1,36

MnO4-/Mn2+

) Zn(s)

1,51

) Sn2+

6.3. Escreva a reao qumica balanceada para a reao de titulao dos ons
2( ) SO
3 com ons dicromato.
ferro
(II)
( ) Zn(s)

180

Olimpada Brasileira de Qumica - 2015

Iberoamericana

Exame Terico

6.4. Calcule o percentual, em massa, do contedo de ferro na amostra de


magnetita, expresso como xido de ferro(III).
Percentual:
6.5. Calcule a constante de equilbrio para a reao de titulao. (Se necessrio, utilizar a tabela de potenciais de reduo do item 6.2)
Constante de equilbrio:
6.6. Calcule o potencial no ponto de equivalncia para a reao de titulao
nas condies citadas. (Se necessrio, utilizar a tabela de potenciais do item 6.2).
Resposta:
6.7. Dentre os indicadores listados abaixo, marque com um X o melhor indicador redox para ser utilizado para esta titulao. (Caso no tenha obtido o
valor do potencial no item 6.6, utilizar o valor 1,32 V)
( ) Fenantrolina (azul / vermelho; E = 1,14V);
( ) Ferroina (azul / vermelho laranja; E = 1,06V);
( ) Carmim de ndigo (azul / amarelo; E = 0,29V);
( ) Azul de metileno (azul / incolor; E = 0,53 V);
( ) Sulfato Azul do Nilo (violeta / incolor; E = 0,41V)
Outra possibilidade para a determinao quantitativa de ferro (II) seria a titulao com soluo de permanganato de potssio. O inconveniente de utilizar
este oxidante com solues que contm ons cloreto a oxidao deste on
a cloro gasoso.
6.8. Calcule o pH mnimo no qual cloro poder ser gerado, a partir de ons
cloreto, em uma titulao com ons permanganato. (Obs.: Considerar as atividades de todas as espcies iguais a 1 e utilizar a tabela de potenciais de
reduo do item 6.2).
Resposta:

Olimpada Brasileira de Qumica - 2015

181

Iberoamericana

Exame Terico

Problema 7 - O Homem Americano do Piau


Questo
Pontos (mximo)
Pontos (correo)

7.1
1,0

7.2
1,0

7.3
5,0

7.4
3,0

Total
10,0

07- A ocupao do homem e o seu espalhamento no novo mundo uma das


questes mais polmicas na paleoantropologia. O bem conhecido paradigma de Clovis sugere que a chegada do Homo Sapiens ao continente americano ocorreu no Alasca por volta de 12000 anos atrs, com provenincia
da Sibria, utilizando-se para isso o acesso dado pelo estreito de Bering. O
nome do paradigma vem da cidade do Novo Mxico na qual foram encontradas pontas de lana fabricadas por uma populao que ocupou parte das
Amricas do Norte e Central.
Human Evolution,
77 de
(2014),
187-195),
alm da
da Tiraum
Peiagrande
no municpio
de de
Entretanto,
o modelo
Clovis
incapaz
deToca
explicar
nmero
stiosCoronel
arqueolgicos
muitoaoantigos
encontrados
em vrios
doofcontiJos Dias, prxima
mesmo parque
nacional (Lahaye,
C. et.pontos
al., Journal
nenteArchaeological
americano.Science,
Exemplos
destes
stios
ocorrem
no
estado
do
Piau,
na Toca
40 (2013) 2840-2847) onde existem evidncias de ocupao
do Serrote das Moendas que se situa no parque nacional da Serra da Capivara
humana pr-Clovis.
(Kinoshita, A. et al., Journal of Human Evolution, 77 (2014), 187-195), alm da
Toca da Tira Peia no municpio de Coronel Jos Dias, prxima ao mesmo parque nacional
(Lahaye,
C.so
et.utilizadas
al., Journal
of Archaeological
40 (2013)
Duas tcnicas
principais
na datao
de sedimentos e Science,
outros materiais
2840-2847)
onde
existem
evidncias
de
ocupao
humana
pr-Clovis.
arqueolgicos, a saber, a Ressonncia de Spin Eletrnico (ESR, do ingls, Electron Spin

DuasResonance)
tcnicas eprincipais
so Estimulada
utilizadasOpticamente
na datao(OSL,
de do
sedimentos
e outros
Luminescncia
ingls, Optically
materiais arqueolgicos, a saber, a Ressonncia de Spin Eletrnico (ESR, do
Stimulated Luminescence). Estas tcnicas cobrem dataes que se encontram alm dos
ingls, Electron Spin Resonance) e Luminescncia Estimulada Opticamente
da tcnica
de dataoStimulated
por Carbono-14
e, portanto, so boas
alternativas.
(OSL,limites
do ingls,
Optically
Luminescence).
Estas
tcnicas cobrem
dataes que se encontram alm dos limites da tcnica de datao por Carbono-14
e, portanto,
so boas
alternativas.
De modo
geral, em ambas
as tcnicas,
a idade do material determinada atravs da
De modo
geral,
em de
ambas
asionizante
tcnicas,
a idade
material
determinada
razo entre
a dose
radiao
acumulada
pelado
amostra
e a velocidade
de
atravs
da
razo
entre
a
dose
de
radiao
ionizante
acumulada
pela
amostra
absoro desta radiao. Esta radiao ionizante corresponde a emisses (alfa),
e a velocidade de absoro desta radiao. Esta radiao ionizante
corres(beta) e (gama) proveniente principalmente dos radioistopos do Urnio (238U), Trio
ponde
a emisses (alfa), (beta) e (gama) proveniente principalmente
K), alm dos
raiosTrio
csmicos
a superfcie(40
terrestre.
(232Th) e do Potssio
dos radioistopos
do(40Urnio
(238U),
(232que
Th)atingem
e do Potssio
K), alm dos
raios csmicos que atingem a superfcie terrestre.

182

Quando a radiao ionizante absorvida pelo material isolante (ou semicondutor),


I

Olimpada Brasileira de Qumica - 2015

como os cristais de quartzo dos sedimentos, eltrons so promovidos da banda de

Iberoamericana

Exame Terico

Quando a radiao ionizante absorvida pelo material isolante (ou semicondutor), como os cristais de quartzo dos sedimentos, eltrons so promovidos
da banda de valncia para a banda de conduo, deixando buracos (stios
positivos) na primeira.
Eventualmente, tanto os eltrons quanto os buracos podem ser aprisionados
em defeitos nas estruturas dos cristais. No caso especfico da ESR, os centros
paramagnticos, que so os eltrons aprisionados, so medidos e a intensidade do sinal , portanto, relacionada com a dose de radiao ionizante
absorvida. Na OSL, uma excitao prvia com luz visvel promove os eltrons
aprisionados para a banda de conduo, o que por sua vez seguida de uma
recombinao destes eltrons com os buracos, quando estes retornam ao
estado fundamental. A luz emitida neste retorno medida e assim a radiao
ionizante absorvida pode ser determinada.
No exemplo do material arqueolgico piauiense proveniente da Toca
do Serrote das Moendas, as amostras correspondiam a dentes de cervos
(Blastocerus dichotomus) associados a um material humano (ossos humanos).
O estudo por ESR revelou que a dose de radiao absorvida pelas amostras
era de 37 Gy (Gy = Gray unidade de dose de radiao ionizante absorvida,
no sistema internacional) e atravs da quantificao dos radioistopos
estabeleceu-se que a velocidade de absoro de radiao ocorreu com taxas
de 305 Gy/ano para radiaes e 970 Gy/ano para radiaes . A absoro
da radiao foi considerada desprezvel, tendo em vista a remoo das
camadas mais externas das amostras. Pela latitude e longitude da localizao
do stio arqueolgico sabia-se que a radiao csmica respondia por uma
taxa de 0,934 Gy/ano.
7.1. Utilizando os resultados da anlise por ESR, qual a idade determinada
para os dentes dos cervos (material arqueolgico)?
Idade do Material arqueolgico:

7.2. Assinale a opo que explica razoavelmente o porqu da dose de radiao ser desprezvel no estudo em questo:
( ) Por possuir maior capacidade ionizante e menor poder de penetrao, a
radiao deve sempre ser desprezada em estudos deste tipo.
( ) No h razo para considerar a radiao , uma vez que no decaimento
dos radioistopos em questo, este tipo de radiao no est envolvida.
I

Olimpada Brasileira de Qumica - 2015

183

Exame
Terico permite qu
Iberoamericana
( ) O maior poder de penetrao da
radiao

(quase
) O nula
maiornas
poder
de penetrao
da radiao
permite qu
camadas
externas das
amostras analisadas,
( ) A radiao tem um
menor
poder
de
penetrao,
portanto,
a remo-analisadas,
quase
nula
nasremovidas.
camadas
externas
das amostras
camadas
foram

o das camadas externas das amostras praticamente removeu material


camadas
foram removidas.
onde h absoro desta
radiao.
( ) O maior poder de penetrao da radiao permite que est tenha ab7.3. O potssio-40 possui tempo de meia vida igual a 1,27 x 10
soro quase nula nas camadas externas das amostras analisadas, por
7.3.
O foram
potssio-40
possui tempopossa
de meia
vida igual a 1,27
x 10
este motivo estas camadas
removidas.
decaimento
desse
radioistopo
ser representado
pelas
re
9
7.3. O potssio-40 possui
tempo
de
meia
vida
igual
a
1,27
x
10
anos.
Sudecaimento desse radioistopo possa ser representado pelas re
ponha que o decaimento desse radioistopo possa ser representado pelas
40
reaes:
K 40Ca +
(Reao 1)
40

40

Ca +

40

(Reao 1)

40

K+
40
K+

Ar (Reao 2)
Ar
(Reao 2)
40

40

Onde 40Ca um istopo do clcio e Ar um istopo do argnio. Suponha


tambm que seja possvel, por 40
algum mtodo, medir as massas de argnio e
Onde Ca um istopo do clcio e 40Ar um istopo do argnio
de clcio num material geolgico. Se o processo na reao 1 responsvel
40
Onde
Ca do
um
istopo
do 10%
clciorestantes
e 40Ar as
um
istopo
argnio
por 90 % do decaimento seja
radioativo
potssio
os
decorrem
possvel,
por
algume mtodo,
medir
massas
dedo
argnio
e
da reao 2, calcule a idade do material geolgico se a razo entre as massas
seja
possvel,
por
algum mtodo,
medir
as massas de
e
geolgico.
Se o
processo
na reao
1 responsvel
porargnio
90 % do
do argnio e do clcio igual
a 0,95.

geolgico.
processo
na reaodecorrem
1 responsvel
por 2,
90 calcu
% do
do potssioSeeoos
10% restantes
da reao
do
potssio
os 10%
restantes
decorrem
da reao
2, calcu
geolgico
se ae razo
entre
as massas
do argnio
e do clcio
igu

Idade do material geolgico:

geolgico se a razo entre as massas do argnio e do clcio igu

7.4. Na emisso da radiao ionizante natural a partir dos radioistopos 238U


e 232Th, tem-se duas famlias radioativas. A partir do 238U, o ltimo elemento
da srie o chumbo-206 (206Pb), e a partir do 232Th o ltimo elemento o
208
Pb. Quantas emisses ra radiaes e 970 Gy/ano para radiaes . A
absoro da radiao e so observadas em cada uma das sries citadas?

Idade do material geolgico:


Idade do material geolgico:

7.4. Na emisso da radiao ionizante natural a partir dos rad


7.4.
Na duas
emisso
da radiao
ionizante
natural
rad
tem-se
famlias
radioativas.
A partir
do a238partir
U, o dos
ltimo
tem-se duas (famlias
ATh
partir
do 238
U, o ltimo
206
chumbo-206
Pb), e radioativas.
a partir do 232
o ltimo
elemento
o2

chumbo-206
(206Pb), eem
a partir
do 232das
Th sries
o ltimo
elemento o 2
e so observadas
cada uma
citadas?
e so observadas em cada uma das sries citadas?

184

Olimpada Brasileira de Qumica - 2015

OBQ 2015

Destaques
Destaques Olmpicos

EOQ Escola Olmpica de Qumica


A Escola Olmpica de Qumica (EOQ) um curso com durao de uma
semana realizado no IQ-USP e organizado por ex-olmpicos de qumica com
apoio do Programa Nacional Olimpadas de Qumica. Realizado anualmente
no ms de julho, contabiliza centenas de estudantes beneficiados por essa
iniciativa do ex-olmpico de qumica, Andr Franco.

Alunos participantes da EOQ em 2015.


I

Olimpada Brasileira de Qumica - 2015

185

OBQ 2015

Destaques

Treinamento laboratorial na UNICAMP


De 28 de junho a 3 de julho de 2015 os estudantes selecionados para
representar o Brasil nas olimpadas internacionais de 2015 participaram de
treinamento laboratorial ministrado no Instituto de Qumica da UNICAMP.
O curso tem por objetivo familiarizar os estudantes selecionados com
procedimentos laboratoriais comumente utilizados nos exames prticos
aplicados nas duas olimpadas internacionais de qumica.
Na foto, da esquerda para a direita, os participantes do curso Vitor Gomes
Pires, Giovanni Elson Rafael de Souza, Pedro Teotnio de Sousa, Gabriel
Ferreira Gomes Amgarten e Joo Martins Cortez Filho.

Alunos participantes do treinamento laboratorial na UNICAMP.

186

Olimpada Brasileira de Qumica - 2015

OBQ 2015

Destaques
Destaques Olmpicos

Destaques Olmpicos
Em 2015, o Brasil participou de duas olimpadas internacionais, em julho,
no Azerbaijo, a equipe composta por trs professores e quatro estudantes
juntou-se a outras 72 delegaes na 47th International Chemistry Olympiad.
Neste certame, a equipe brasileira esteve representada por Vitor Gomes
Pires, do Colgio Etapa de So Paulo, agraciado com medalha de prata, e os
cearenses Gabriel Ferreira Gomes Amgarten e Pedro Teotnio de Sousa, do
Colgio Ari de S premiados com medalhas de bronze e por Giovanni Elson
Rafael de Souza, do Colgio Farias Brito.
Em setembro, em Teresina, organizada pela Universidade Federal do
Piau, a XXI Olimpada Iberoamericana de Qumica que reuniu quinze pases
dessa comunidade. A equipe brasileira ficou composta pelo estudante
paulista Vitor Pires, premiado com medalha de ouro, por dois cearenses,
Gabriel Amgarten, medalhista de ouro, e Pedro Teotnio, que conquistou
medalha de prata, e, ainda, o estudante Joo Martins Cortez Filho, do Instituto
Dom Barreto, em Teresina premiado com medalha de prata.

Gabriel, medalha de ouro na XXI OIAQ,


medalha de bronze na 47th IChO

Vitor, medalha de ouro na XXI OIAQ, medalha


de prata na 47th IChO
I

Olimpada Brasileira de Qumica - 2015

187

OBQ 2015

Destaques

Pedro, medalha de prata na XXI OIAQ,


medalha de bronze na 47th IChO

Joo Cortez, medalha de prata na XXI OIAQ,

Giovanni, participou na 47th IChO

188

Olimpada Brasileira de Qumica - 2015

OBQ 2015

Destaques
Destaques Olmpicos

Equipe brasileira na XXI OIAQ

Olimpada Brasileira de Qumica - 2015

189

OBQ 2015

Destaques

Equipe brasileira em Baku - Azerbaijo, ladeada pelo Prof. Dr. Jos Arimatia Lopes, Reitor da UFPI, e o Embaixador do Brasil
no Azerbaijo, Santiago Lus Fernndez Alczar.

Programa Nacional
Olimpadas de Qumica,
20 anos revelando
talentos para a
academia e a indstria.

190

Olimpada Brasileira de Qumica - 2015

OBQ 2015

Destaques
Destaques Olmpicos

Finalistas da XX OIAQ, em Teresina, ladeados pelo Secretrio de Fazenda do Piau, ex-olmpico de Qumica, Rafael Fonteles,
Prof. Srgio Melo, mentor da Delegao Brasileira e Prof. Arimateia Lopes, Reitor da UFPI.

Encerramento do curso de qumica ministrado por licenciandos em qumica, ex-olmpicos, para estudantes de ensino mdio
em escolas pblicas do Cear.

Olimpada Brasileira de Qumica - 2015

191

Depoimento

Fabiano Gomes

Dos certames olmpicos


para a docncia na academia
Durante os anos de ensino mdio, fui apresentado ao fascinante mundo
da qumica. Achava fantstica a possibilidade de compreender, sob o olhar
da qumica, como a natureza atua a nvel molecular, e ficava extasiado quando meus professores me ensinavam alguns truques utilizando a qumica,
como por exemplo, o uso de pequenas quantidades de gua sanitria para
purificar frutas e legumes, e o uso de acetona para remover a cola de Super
Bonder. Nesse perodo, eu era apenas um adolescente encantado com a qumica, mas meus professores viram algum tipo de talento dentro de mim e me
estimularam a estudar cada vez mais essa matria. Nesse sentido, eles me
inscreveram para participar da 1 Olimpada de Qumica do meu estado. Estava na ltima srie do ensino mdio e resolvi realizar as provas da Olimpada
de Qumica como uma preparao para o vestibular, que naquela poca era
praticamente o nico meio de se entrar em uma universidade pblica. Mas,
para minha surpresa e felicidade, obtive a primeira colocao nesta olimpada. A partir de ento, comecei a acreditar naquilo que meus professores
diziam: eu tinha algum tipo de talento para a qumica. Nesse mesmo ano,
fui convocado para fazer a prova da Olimpada Norte-Nordeste de Qumica,
mas dessa vez no fui muito bem. Mesmo assim, no desanimei e continuei
acreditando nesse talento escondido dentro de mim, que meus professores
tanto me falavam.
Mas se aproximava o vestibular e eu ainda estava com dvida para qual
curso prestar: qumica ou engenharia qumica. Conversei com professores e
orientadores pedaggicos de minha escola, li inmeras matrias de revistas
sobre vocao profissional, fui at a universidade e conheci professores e laboratrios de ambos os cursos, mas no fim, decidi seguir o que meu corao
dizia e optei pelo curso de qumica bacharelado. Passei no vestibular e, logo
no incio do curso, j comecei a me envolver com trabalhos de pesquisa e a
participar de eventos de qumica. De fato, era aquilo mesmo que eu queria.
Terminei a graduao antes do tempo previsto e entrei logo para o mestrado
e depois para o doutorado. Durante o meu doutorado, tive a oportunidade
de atuar como professor substituto na universidade em que terminei minha
graduao, e logo aps defender minha tese, tive a felicidade de passar em
concurso pblico para professor desta mesma universidade que me formou.

192

Olimpada Brasileira de Qumica - 2015

Depoimento

Fabiano
Gomes Olmpicos
Destaques

Hoje sou professor de Qumica Orgnica na Universidade Federal do


Rio Grande do Norte (UFRN). Assim que entrei para a UFRN, convidaram-me para coordenar a Olimpada de Qumica em meu estado. Atualmente,
tenho a enorme satisfao de poder contribuir para a descoberta de jovens
talentos para a rea da qumica e de poder retribuir tudo aquilo que o Programa Olimpadas de Qumica me proporcionou. Como atual coordenador
do curso de graduao de Qumica da UFRN, tenho ainda outra satisfao,
que a de poder receber, no curso de qumica, estudantes que, assim como
eu, optaram por este curso aps participarem da Olimpada de Qumica
do Rio Grande do Norte. Esta umas das maiores qualidades e virtudes da
Olimpada de Qumica: ajudar jovens a descobrirem sua vocao. E sinto-me
bastante grato e honrado por ter sido fruto da Olimpada de Qumica, e de
estar tendo a oportunidade de contribuir, voluntariamente e com muita dedicao, para a sua execuo em meu estado.
Fabiano do Esprito Santo Gomes
Coordenador da Olimpada de Qumica
no estado do Rio Grande do Norte

Formando pesquisadores
Ingressei nas olimpadas de Qumica em meados de 2004, era por volta
da antiga stima srie. Embora a gente s fosse ver Qumica na sala de aula
na oitava srie, quem quisesse j podia ter aulas nos grupos interessados
em olimpada. Eu tinha algumas noes vagas dela por gostar bastante de
ler enciclopdias e de ver minha irm estudando para as provas dela (minha
irm cinco anos mais velha que eu). Foi com o auxlio de minha primeira
professora de olimpadas, professora Eliene, que eu me apaixonei pela Qumica. Ela me emprestava livros e inspirava a todos os seus alunos com experimentos bastante visuais, como o experimento do vulco (decomposio
trmica do dicromato de amnio).
A partir de ento eu ingressaria em uma vida de olimpadas at o terceiro
ano do ensino mdio. Tive a oportunidade de viajar ao participar da Olimpa-

Olimpada Brasileira de Qumica - 2015

193

Depoimento

PedroNakasu

da Ibero Americana de Qumica no Rio de Janeiro em 2007 e o intercmbio


cultural que tive com outros estudantes foi de muita valia. At hoje mantenho
contato com eles. 2007 tambm foi o ano em que eu tive o vestibular. Todos
diziam para eu prestar para Medicina, j que uma das especficas era Qumica e um curso mais concorrido para se passar. Mas o que eu realmente
gostava era da prpria Qumica! Resolvi seguir meu sonho e consegui passar
para o curso de Qumica na Unicamp.
Hoje, j finalizei meu mestrado e curso doutorado em Engenharia Qumica. Sou eternamente grato s oportunidades oferecidas pelo meu perodo
de participao nas olimpadas. Eu poderia dizer que elas me prepararam de
vrias formas: como um estudante autnomo, um pesquisador e um cidado
consciente. Ser estudante autnomo significa organizar seus estudos, elaborar rotinas de resoluo de exerccios e de provas de olimpadas anteriores;
voc estuda por espontnea vontade, no h a presso de vestibular, ENEM
e afins. Ser um pesquisador significa investigar profundamente problemas
de complexidade elevada e que requerem a construo inerente do mtodo
cientfico. Em olimpadas internacionais, frequentemente os estudantes deparam com questes com temas atuais e geralmente vistos em dissertaes
de mestrado e teses de doutorado. Ser um cidado consciente entender o
papel da educao na evoluo de nossa sociedade. Somente um pas com
educao forte consegue sobrepujar os mais difceis obstculos, tais como
a corrupo e o subdesenvolvimento. Finalizo esse pequeno texto fazendo
uma citao de Castro Alves:
Oh, Bendito o que semeia,
livros, livros mo cheia,
E manda o povo pensar,
O livro caindo nalma,
germe - que faz a palma,
chuva - que faz o mar!

"Nunca estrague o seu presente


por um passado que no tem futuro".
Dalai Lama

194

Olimpada Brasileira de Qumica - 2015

Consideraes

Srgio Melo

Construindo uma histria


Iniciado em 1995, o Programa Nacional Olimpadas de Qumica atingiu
neste ano duas dcadas de atividades contnuas, um patrimnio imaterial
construdo com muitas lutas e uma histria construda com a tenacidade de
uma equipe de abnegados professores que ao longo desses anos ofereceram trabalho voluntrio em prol da valorizao do estudo e ensino da qumica em seus estados.
Adicionalmente, contamos com preciosos esforos dos professores das
escolas participantes e o apoio dos agentes financiadores, determinantes
para contabilizarmos, ao longo desse caminho, mais de 3 milhes de estudantes envolvidos nas quatro modalidades de certames deste projeto. Fecha-se esta dcada com a criao de uma nova modalidade de premiao
a qual tem por objetivo precpuo destacar e incentivar alunos das escolas
pblicas que se sobressaram na Olimpada Brasileira de Qumica Jnior. Assim, foi criado um novo trofu para registrar o esforo e a competncia dos
estudantes com relevante destaque nesse segmento educacional.
Uma parceria da Associao Brasileira de Qumica ABQ com a Sociedade Brasileira de Fsica SBF, apoiada pela SECIS Secretaria de Cincia,
Tecnologia e Incluso Social do Ministrio da Cincia, Tecnologia e Inovao, resultou na criao da Olimpada Brasileira de Cincias, ideia gestada
h mais de quatro anos, somente agora em vias em execuo. Este certame,
previsto para 2016, mobilizar um eixo integrador para duas disciplinas que
se identificam, qumica e fsica, com o estabelecimento de nexos e vnculos
entre si para alcanar um conhecimento unificado. De uma s vez, estimular
o estudo integrado das cincias qumica e fsica nas escolas de ensino fundamental e mdio objetivando identificar talentos, sobretudo, nas escolas da
rede pblica de ensino.
Um acontecimento de grande destaque em 2015 foi a realizao, no Brasil, da XX Olimpada Iberoamericana de Qumica. Em momento desfavorvel,
a UFPI enfrentou as adversidades e realizou uma das mais primorosas edies na existncia das olimpadas ibero-americanas de qumica, superando
a si prpria quando realizou, h cinco anos passados, similar evento.
A lamentar, o encerramento do projeto Aes Construtivas do Conhecimento Qumica, um projeto que teve trs anos de atividades em prol dos carentes jovens das escola pblicas e resultou, melhora na qualidade do ensino
I

Olimpada Brasileira de Qumica - 2015

195

Consideraes

Srgio Melo

nas escolas envolvidas e, centenas de publicaes em Anais de congressos


direcionados para a qumica. Um compndio publicado pela EDUEPB, com
210 pginas e 7 artigos cientficos, analisados e aprovados por competente
corpo editorial saiu publicado no incio deste ms. Em decorrncia dessa
descontinuidade, nove mil estudantes/ano deixaram de ter esse apoio didtico conduzido por licenciandos de qumica que compartilharam experincias em nosso Programa.
A sobrevivncia das Olimpadas do Conhecimento est intrinsecamente
ligada ao apoio que o governo oferece, desde 2002, quando o primeiro Edital de apoio s olimpadas cientficas foi lanado, esses projetos cresceram
em nmero de participantes e na variedade das cincias envolvidas. Com
respaldo das sociedades cientficas adquiriu corpo e prestgio, agora, ameaados pelos cortes de verbas, prenuncia-se a asfixia desses projetos educacionais.
Atualmente, o Programa Nacional Olimpadas de Qumica apoia a realizao de olimpadas estaduais, regionais e municipais. As olimpadas estaduais formam a base mais larga e mais significativa desse projeto, um grupo
de 27 olimpadas, cada uma delas representado a fora de uma unidade federativa. Manteve, em 2015, o mesmo nvel de participao alcanado no
ano anterior, 301 mil estudantes nos quatro certames nacionais. Seus objetivos no ficam restritos aos resultados e premiaes, mas se ampliam na acentuada capacidade de mapear a qualidade do ensino de qumica posto em
prtica nas diferentes regies do pas. Com essa experincia acumulada foi
capaz oferecer subsdios para estudos acadmicos, j disponveis, em teses
de doutorado, Anais de congressos e compndios editados pelo Programa.
Explicitados nos diversos depoimentos gravados ao longo da existncia dos Anais do Programa, tornamo-nos conscientes da capacidade transformadora das olimpadas cientficas e do nosso compromisso em marchar
cultivando os mais nobres atributos desse certame: promover interaes indissolveis entre estudantes, seus professores e a academia e influenciar em
suas trajetrias de vida.
Parabenizemo-nos todos por termos desfrutado mais essa vitria.
Srgio Melo

196

Olimpada Brasileira de Qumica - 2015

Endereos

Relao

Relao de endereos
ESTADO
ACRE

ALAGOAS
alagoas.obquimica.org/

COORDENADOR
Profa. Iusseny do Nascimento
Soares Vieira
iusseny.vieira@ifac.edu.br
Prof. Joacy Vicente Ferreira
joacyferreira@ifal.edu.br

AMAP
amapa.obquimica.org/

Prof. Roberto Messias Bezerra


messias@unifap.br

AMAZONAS
www.oaq.ufam.edu.br
olimpadasdequimica@
hotmail.com

Prof . Paulo Rogrio da Costa


Couceiro couceiro@ufam.edu.br
couceiro35@gmail.com

BAHIA
www.obaq.ufba.br

Prof . Lafaiete Almeida Cardoso


lafaiete@ufba.br

CEAR
www.necim.ufc.br

Prof. Leonilde Maria Cmara


Jatahy
necim@bol.com.br
leojatahy@ig.com.br
Prof. Cludia Christina B. S. Carneiro
Profa. Elaine Rose Maia
emaia@unb.br petqui@unb.br
elaine.rose.maia@gmail.com

DISTRITO FEDERAL
brasilia.obquimica.org/

ESPRITO SANTO
www.ocq.ufes.br/

Prof. Carlos Vital Paixo de Melo


cvpaixao@globo.com
carlosmelo@ufes.br

GOIS
goias.obquimica.org/

Renato Cndido da Silva


obqgoias@yahoo.com.br

ENDEREO PROFISSIONAL
Instututo Federal do Acre - Campus Xapuri
Rua Cel. Brando1622 - Centro
69.930-000 Xapuri - AC - (68) 3542.2083
IFAL - Campus Macei
Instituto Federal de Alagoas
Rua Mizael Domingues, 75 Poo
57.025-510 Macei - AL
Fone: (82) 2126-7000 2126 .7016
Universidade Federal do Amap
Rod. Juscelino Kubitscheck, Km 02
68.902-280 Macap - AP
Fone: (96)3312-1700
Universidade Federal do Amazonas
Departamento de Qumica/ICE/UFAM - Bloco 10
Setor Norte do Campus Universitrio Sen. Arthur
Virglio Filho
Av . Gal . Rodrigo Otvio Jordo Ramos, 6.200
69.077-000 Manaus - AM Coroado
(92) 3305-2874 (telefax)
Universidade Federal da Bahia
Instituto de Qumica - Depto . Qui . Orgnica
Rua Baro de Geremoabo, s/n (Ondina)
40 .170-115 Salvador - BA
(71) 3283 .6813 3237.4117 (Fax)
Universidade Federal do Cear
NECIM - Ncleo de Ensino de Cincias e
Matemtica
Av. da Universidade, 2470
60020-180 Fortaleza - Cear (85) 3366.7796
Universidade de Braslia - Instituto de Qumica
Campus Universitrio Darcy Ribeiro 70910-970 ICC Sul - Asa Norte Cx.Postal: 04478
(61) - (61) 3107-3895 3107.3893
Universidade Federal do Esprito Santo
Departamento de Qumica - CCE
Av. Fernando Ferrari, 845 Goiabeiras
29.075-015 Vitria ES
(27) 3107.3895 - 3107.3806 - 3107.3893 (PET)
Universidade Federal de Gois,
Instituto de Qumica
Campos II Samambaia Bloco I - 74001-970 Goiania, GO - Telefone: 62 3521.1167

Olimpada Brasileira de Qumica - 2015

197

Endereos

Relao

MARANHO
maranhao.obquimica.org/

Prof. Roberto Batista de Lima


rblimas@gmail.com
obqma@yahoo.com.br

MATO GROSSO

Prof. Luiz Both


luiz_both@ibest.com.br
luiz.both@hotmail.com

MATO GROSSO
DO SUL
olimpiquimica.ms.obq@
gmail.com

Prof. Onofre Salgado Siqueira


olimpiada.quimica.ms@gmail.com
onofre.s.siqueira@gmail.com

MINAS GERAIS
www.qui.ufmg.br/omq/
zeus.qui.ufmg.br/~omq/

Prof. Gilson de Freitas Silva


gilson.freitas@gmail.com
omq.ufmg@gmail.com

PAR
para.obquimica.org/

Prof. Mrcio de Souza Farias


toraqk@yahoo.com.br
Profa. Patrcia da Luz
pdaluz@yahoo.com
Francisco Ferreira Dantas Filho

PARABA
paraiba.obquimica.org/

PARAN
www.oprq.daqbi.ct.utfpr.
edu.br/

Profa. Maurici Luiza Del Monego


mauriciluiza@gmail.com

PERNAMBUCO
pernambuco.obquimica.org/

Prof. Cristiano Marcelino Almeida


Cardoso Filho
cristianomarcelinojr@uol.com.br

PIAU
piaui.obquimica.org/

Prof. Jos Milton Elias de Matos


jmematos@gmail.com

198

Olimpada Brasileira de Qumica - 2015

Universidade Federal do Maranho


Departamento de Qumica
Av. dos Portugueses, 1966 Campus da Bacanga
65.080-580 So Luis - MA
Fone: (98) 3272.9241
IFMT - Campus Bela Vista
Av. Juliano Costa Marques, s/n
78.050-560 Cuiab - MT
Fone: 65 3318.5100 (IFMT) 3663.1374
Universidade Federal do Mato Grosso do Sul
Instituto de Qumica
Rua Filinto Muller, 1555 (Cidade universitria)
79.070-900 Campo Grande - MS
(67) 3345.3556 3345.3552 (FAX)
Universidade Federal de Minas Gerais
Departamento de Qumica
Instituto de Cincias Exatas (ICEx)
Av. Pres. Antnio Carlos, 6627 Pampulha
31.270-901 Belo Horizonte - MG
(31) 3409.5772 - 3409.7558 (omq) 3499.5700 (Fax)
Instituto Federal do Par - IFPA
Departamento de Qumica
Avenida Almirante Barroso, 1155
Bairro do Marco - 66093-020 Belm - PA
Universidade Estadual da Paraba UEPB Centro de Cincias Tecnologia - Depto. Qumica
Rua Juvncio Arruda, s/n Bodocong
58.109-790 - Campina Grande -PB
Telefone: (83) 3315.3356 (83) 99919 -7772
Universidade Tecnolgica Federal do Paran
Campus Curitiba - Sede Ecoville
Rua Dep. Heitor Alencar Furtado, 5000 Bloco C
81280-340 - Curitiba - PR - Brasil
Fone: (41) 3279.4575 / 4522
Universidade Federal Rural de
PernambucoSalgadinho Olinda -PE
Fone: 81-3183.5525 / 3183.5528
Universidade Federal de Pernambuco, CCEN,
Depto. de Qumica Fundamental, Cidade
Universitria, 50 740-521 Recife - PE
Fone: 81-2126.7415 / 2126 .8442 (fax)
Universidade Federal do Piau
Depto de Qumica - SG2 CCN - Campus da Ininga
64049-550 - Teresina - PI - (86) 3215.5620 telefax

Endereos

Relao

RIO DE JANEIRO
sites.google.com/site/
olimpiadadequimicarj/

Prof. Paulo Chagas


paulo.chagas@ifrj.edu.br
Prof. Luis Carlos de Abreu Gomes
luis.quimica@cp2.g12.br

RIO GRANDE
DO NORTE
oqrn.quimica.ufrn.br

RIO GRANDE
DO SUL
http://gaia.liberato.com.br/
olimpiada/index.php
Twitter: http://twitter.com/
oqdors

Prof. Fabiano do Esprito


Santo Gomes
feibi_natal@yahoo.com.br
Profa. Maria de Ftima Vitria
de Moura
mfvmoura@quimica.ufrn.br
Coordenao Colegiada:
Prof. Daniel Jacobus
Prof. Fvio Roberto Becker Dillio
Prof. Sabrina da Silva Bazzan
Profa. Nair CXristina Muller
oqdors@gmail.com
quimica@liberato.com.br
Prof. Renato Cassaro
renatocassaro26@hotmail.com

RONDNIA
www.programa-olimpiadarondoniense-de-quimica.
com/

Prof. Jandi Costa jandi@gmail.com

RORAIMA
roraima.obquimica.org/

Profa. Maria Lcia Taveira


taveiraml@dqui.ufrr.br

SANTA CATARINA
www.ocquimica.com.br

SO PAULO
http://allchemy.iq.usp.br

Profa. Clria Mendona de Moraes


cmdmorae@yahoo.com.br
Prof. Gilson Rocha Reynaldo
gilson@unisul.br
Jos Maximiliano Muller Netto
max@crqsc.gov.br
Jonas Comin Nunes
jonas@crqsc.gov.br
Prof. Ivano G. R. Gutz
abqsp@iq.usp.br
gutz@iq.usp.br

IFRJ - Instituto Federal de Cincia


e Tecnologia do Rio de Janeiro
Rua Senador Furtado, 121 Praa da Bandeira 20.270-021 Rio de Janeiro - RJ
(21) 3978. 5918 3567.0283 (Fax)
CIEP 436 Neusa Brizola - 24.425 - 004 Neves - So
Gonalo - RJ - Fone: 21 9795 0176
Rua Dr. Jos Augusto Pereira dos Santos s/n
Fone: 21 8897-4492
Universidade Federal do Rio Grande do Norte
Centro de Cincias Exatas e da Terra
Instituto de Qumica
Av. Senador Salgado Filho, 3000 Lagoa Nova
Campus Universitrio - 59.072-970 Natal - RN
Fone: (84) 3342.2323 (R.117)
Fundao Escola Tcnica Liberato
Salzano V. da Cunha
Rua Inconfidentes, 395 Primavera
93.340-140 - Novo Hamburgo - RS
(51) 3584.2027

Fundao Universidade Federal de


Rondnia- UNIR
Laboratrio de Qumica Analtica de Solos
Departamento de Qumica BR 364 km 9
78.000-000 Porto Velho - RO
Fone (69) 2182.2193
Universidade Federal de Roraima
Departamento de Qumica - Campus do Paricarana
69.301-270 Boa Vista - RR - Fone: (95) 621.3140
621.3137 623.1581 224.7302
Universidade Estadual de Roraima
Universidade do Sul de Santa Catarina - UNISUL
Av. Jos Accio Moreira, 787 - Caixa postal 370
88.704-900 Tubaro - SC Bairro Dehon Fone: (48)
621.3371 - Fax (48) 621 3000
Conselho Regional de Qumica - 13a Regio
Rua Pref. Osmar Cunha, 126
88.015-100 Florianpolis (48) 3223.7800
ABQ Regional So Paulo
Instituto de Qumica da USP
Av. Prof. Lineu Prestes, 748 sala 1274
05.508-000 - So Paulo - SP
(11) 3091.2159 (Mirian, 8 -12h) 3091.2150

Olimpada Brasileira de Qumica - 2015

199

Endereos

Relao

SERGIPE
http://.osequim.blogpot.
com.br/

Prof. Andr Luiz bacelar Silva


Barreiros
osequim@gmail.com
andrelbbarreiros@hotmail.com

TOCANTINS
tocantins.obquimica.org/

Prof. Jos Expedito


Cavalcante da Silva
mailto:eliete@unitins.br
jecs@mail.uft.edu.br

Universidade Federal de Sergipe Depto. de Qumica


Campus Prof. Jos Alosio de Campos
Av. Mal. Rondon, s/n Jardim Rose Else
49.100.000 - So Cristvo - SE
Fone: (79) 2105.6898
Universidade Federal do Tocantins
Coordenao de Qumica
Rua Paraguai, s/n (esquina com Urixamas)
Setor Cimba - 77.838-824, Araguana-TO
Fone: 63 81112869 - 63 21122201

PROMOTORES
Universidade Federal do Cear
Pr-Reitoria de Extenso

Av. da Universidade, 2932 - Campus do Benfica 60.020


Fortaleza - CE - Fone: (85) 3366.7300

Universidade Federal do Piau


Pr-Reitoria de Extenso

Campus da Ininga - 64.049-550 Teresina - PI


Fone: (86) 3215.5692 Fax: (86) 215.5570

SECITECE - Secretaria da Cincia, Tecnologia


e Educao Superior do Estado do Cear.

Av. Dr. Jos Martins Rodrigues, 150 Edson Queiroz - CEP:


60811-520 - Fortaleza - Cear - Ver localizao no mapa
Telefone: (85) 3101.6400 | Fax: (85) 3101.3675
APOIO

CNPq - Conselho Nacional de Desenvolvimento Cientfico e Tecnolgico

SHIS QI 1 Conjunto B - Bloco D, 2o andar Edifcio Santos


Dumont, Lago Sul 71605-190 Braslia - DF
Fone: (61) 3211-9408

CAPES - Coordenao de Aperfeioamento


de Pessoal de Nvel Superior

Setor Bancrio Norte, Quadra 02, Bloco L, Lote 6, 4o. Andar


70.040-020 - Braslia/DF

Ministrio da Cincia, Tecnologia e Inovao


Secretaria de Cincia e Tecnologia para a Incluso
Social

Esplanada dos Ministrios, Bloco E, 2o. andar


70.067-900 Braslia - DF

REALIZADOR
Associao Brasileira de Qumica
www.abq.org.br

200

Olimpada Brasileira de Qumica - 2015

Av. Presidente Vargas, 633 sala 2208 20071-004 Rio de


Janeiro RJ - Telefone: 21 2224-4480 Fax: 21 2224-6881
E-mail: abqrj@alternex.com.br

Das könnte Ihnen auch gefallen